COLLECTION OF PAST PAPERS BY ANGINA PECTORIS.pdf

May 15, 2018 | Author: Muhammad Omar Akram | Category: Autonomic Nervous System, Multiple Myeloma, Central Nervous System, Hemorrhoid, Heart


Comments



Description

February 2014 Medicine Solved Paper192 MCQs (Mixed – Paper A & B) 1. High Pressure in Glomerular capillaries is due to a. Afferent is long b. Afferent is short c. Efferent is low resistanced. Efferent is down stream 2.Regarding SLE followin are true except a. ANA is +ve b. Fibrinoid necrosis is common histological feature c. Glomerulonephritis is commonest cause of death d. more common in women e. risk is increased by inheritance of HLA B27 3.Edema in nephrotic syndrome a. Na retention b. Hypoproteinemia c. Hyperlipidemia 4.Bronchopulmonary segment a. each has its own blood supply b. different number on both sides 5.Recently diagnosed fatal disease, most appropiate a. tell family but nt patient b tell patient but nt family c. tell family and patient as soon as knwn d. crisp, logical and evidence based accurate information to the patient and family accordin to demand 6.Young lady having anemia, MCV 78 , MCH 25, MCHC 29 a. Hypochromic b.Hypochromic and Normocytic c.Hypochromic and microcytic d. normocytic and normochromic 7.WHICH OF THE FOLLOWING SHIFTS THE O2-HGB DISSOCIATION CURVE TO RIGHT: A.ACIDOSIS B.ALKALOSIS C.INCREASE pH D. NO E. FETAL HEMOGLOBIN 8.A 75 year old man with an acute MI suddenly dies 6 days after the infarct . At autopsy, there is a large amount of blood in the pericardial sac . The most likely cause of death is ; A. Cholesterol pericarditis B. Amyloidosis C. Cardiac tamponade D. Codsackie B pericarditis E. Constrictive pericarditis 9.most active gluconeogenesis results from metabolism of ? a.fatty acid b.protein c.cholesterol d.glycogen e. TG 10. Severe magnesium deficiency a. Hypocalcemia b. Hypercalcemia c. Hypokalemia d.Hypophosphatemia 11. Langerhans giant cells found in a.Tuberculosis b.Sarcoidosis c.Wegners granulomatosis D)Syphlis 12. parasympathetic inervation a.sweat gland b.salivary gland 13. A pt has multiple polyps on colonoscopy.His father died of same disease.These types of polyps are a-Hemartomatous b-Adenomatous polyp c-Metaplastic 14. Dysarthria is due to a lesion in? a.Brocas area b.Cerebellum c.Thalamus d.Sensory motor cortex e.Hypothalamus 15.Hypothyroidism a. Increased cholesterol 16. During 2nd week which is appropriate test for Typhoid fever ? a) Widal+Blood culture b) Blood culture only c) Stool culture only d) Bone marrow 17. A woman havin fever for few days developed chest pain which aggravated by lying down . Reason is? a.Costochondral junction b.Mi c. pericarditis d.due to plurea 18. purpura is most likely asaociated with a. leukemia b. von wilbrand disease c. hiv 19. a lady using diethylesterbestrol what should be most likely in her daughter a. squamous cell carcinoma b. clear cell carcinoma 20. Lady with osteoporosis,having family h/o cardiac disease and breast ca. Treated by a. Calcium plus vit D b. Raloxifene c. bisphosphonates d. HRT 21. Which disease involves a carrier stage ? a.Measles b.Mumps c.Polio d.Typhoid 22. H+ ion secretion in proximal tubule is related with? A. Bicarbonate reabsorption B. Bicarbonate excretion C. K secretion D. Na secretion 23. Hamartoma is ; A. A Metaplasia B. A neoplasia C. Has a capsule D. Same as cholestoma E. Totally benign 24. 40 yr old woman, middle class,3 kids ,pap smear shows cervical dysplasia..cause A HPV B IUCD 25. 10% Formaldehyde is used for sending Biopsy specimens,Why? a. Sterilisation b.To prevent autolysis c. TO stain cells d. TO denature proteins e. to cause lysis of cell 26. Metastatis occur in all except a.Liver b. Brain d. Spleen d. lung 27.Fast, sharp pain from mechanical origin is mediated by which fibers; A. A alpha B. A delta C. C fiber D. High myelinated 28. Fat necrosis occurs in ; A. Acute pancreatitis B. Heart C. Liver 29. Which tumor is derived from all the 3 germ layers; A. Adenoma B. Teratoma C. Rhabdomyosarcoma D. Chancroid 30. A 20yr old male has dyspnea on lying down. The structure most likely to cause this is ; A. Enlarged isthmus of thyroid B. Puberty C. Retrosternal goiter 31. A FISHER MAN PRESENTED WITH SLOWLY DEVELOPING LETHARGY, EASY FATIGUE AND PALPITATIONS. HIS DIET COMPRISES OF FISH AND RICE MOSTLY PHYSICAL EXAM SHOWED PALLOR AND LOSS OF TOUCH SENSATION IN BOTH FEET AND LOWER LIMB. HIS CBC SHOWED HB 7.5G/DL MCV 132, PLATELETS 110 AND WBC 3450 . MOST LIKELY CAUSATIVE PARASITE: A.ANKYLOSTOMA DUODENALE B.DIPHYLLOBOTHRIUM LATUM C.STRONGYLOID STERCOLIS D.ECHINOCOCCUS GRANULOSIS E. TAENIA SAGINATA 32. Long term elevated BP is due to a.cardiac output b.renal function c.TPR 33. The vitelline duct forms; A. Ductus venosus B. Duodenum C. Meckel, s diverticum D. Portal vein E. Urachus 34. Which is correct Regarding Axillary artery a-It Lies behind pectoralis minor b-Extend upto the lower border of pectoralis major 35. a. the components of the cell containing microtubules in there structure are; a. cell membrane b. centrioles c. endoplasmic reticulum d. lysosomes e. mitrochondria 36. a chronic smoker from 30 years working in a tyre factory presented with painless hematuria n on histopathology transitional cell carcinoma of bladder causative agent is ..? a.aromatic amines b.tobaco smoking c.nitrites/nitratesd.shistosomia hematobium 37. most common manifestation of autoimmune disease ; a. Fever b.arthralgia c.haematologic symptoms. d.fatigue 38. ACTH effectively controls ? a. Aldosterone b. Adrenal androgens c. Adrenal estrogen d. Hydrocortisone 39. what defines the acute inflammation ? 1.local response of living tissue to injury 2.systemic response of living tissue to injury 3.migration of leukocytes 40. aortic pressure lowest during which phase of cardiac cycle?? a.atrial systole b.isovolumic contraction c.isovolumic relaxation 41. The vision lost is left temporal and right nasal field of both eyes, the visual pathway involved in this lesion is A. Rt optic nerve B. Left optic tract C. Rt optic tract D. Left optic nerve 42. the affect of L dopa in parkinson pt dcreease gragually due to 1.antibodies develop against l dopa 2.gradual degeneration of neurons in sunstantia nigra 43. Glucose transport across a membrane is directly proportional to concentration graddient . Type of transport is ; A. Simple diffusion B. Facilited C. Active D. Na countertaransport 44. sebacous glands are absent on 1.corner of lips 2.palms 3.glans of penis 45. Blood transfusion reaction is likely to occur when a person having blood group A+ is transfused with blood of group; A. A+ B. A- C. AB- D. O+ E. O46. C0mplicti0n of diabetic? a.Wet gangren b. dry gangren c. ranyod phen0mena d. thrmbroagints 47. true regarding genetic inhertance of SLE is ; a. high incidence in monozygotic twins b. incidence is high in females c. if its in family, more chances to occur to all family members 48. Langhans giant cells are found in; a.Tuberculosis b.Sarcoidosis c.Wegners granulomatosis d. Leprosy 49. most active gluconeogenesis results from metabolism of ? a.fatty acid b.amino acids c.cholesterol d.glycogen/carbohydrates 50. Esophago gastric junction competence is maintained by A. lying supine B. increased intra abdominal pressure C. diaphragm paralysis D. use of metoclopramide E. use of morphine 51. Diminished jaw jerk due to lesion of A. Mesencephalic nucleus of V nerve B. Facial motor nucleus C. Glossophyrngeal injury D. Vagus injury 52. Co-trimoxazole action in.hiv pt on pnumocystc carini? a.Inhibit cellwall syn b.Inhibit folic acid synth c. inhibit DNA synth 53. Which of the following structure doesn't pass posterior to the flexor retinacum A. Ulnar nerve B. Median nerve C. Flexer digitorum superficialis D. Flexer digitorum profundus 54. Cardiac surgeon during open heart surgery found bleeding vessel accompanying LAD in anterior interXventricular groove . Most likely involved vein is ; A.Great cardiac vein B. Anterior cardiac vein C. Oblique vein D. Small cardiac vein E. Marginal vein 55. An upper motor lesion of facial nerve will result in ; A. Excessive lacrimation B. Hyperacusis C. Loss of taste sensation in ant 2 /3 of tongue D. Ability to wrinkle the forehead on the affected side 56. SQUAMOUS CELL CARCINOMA a.Actinic keratosis b. marjulian ulcer c. Bowen Disease 57. Regarding histology of gastrointestinal tract, which is inappropriate ; A. Peneth cells contain Eosinophilic granules in apical cytoplasm B. Peyer's patches are present in submucous layer of duodenum C. Parietal cells of human stomach secretes intrinsic factor D. Striated brush border is composed of microvilli E. The longitudinal coat of muscularis externa is arranged in 3 bands in colon 58. Which one.is.malignant a.Glioma b.Meningioma c.Psedolymphoma d.Chondroma 59. which cranial nerves r parasympathetic A. III, VII, IX, X B. V, VIII, II, XII 60. Severly dehydrated man comes in er what to gv a.5percent glucose b.Ten precent glucose c.Hypertonic albumin d. 0.9% nacl 61. Fibrous dysplasia commonly occurs in a.Maxilla b.Occipital bone c.Frontal bone d.Temporal bone 62. Weight of prostate a.14 gr b.18 gr c.30 gr d.40 gr 63. Edema is caused by; A. ↓ hydrostatic pressure B. ↑ osmotic pressure C. Lymphatic obstruction 64. Growth hormone secretion is increased ; A. Exercise B. Hperglycemia 65. Drug action wont b affected bound to alpha glycoprotein in a. Neoplasm b.Trauma c. Hepatic disease d. MI 66. Clinically GFR is measured by a. Creatinine B. Inulin 67. 16 yr child with cola color urine since one day , Hb 6 , high grade fever , was given antimalarial 1 day back .. a.g6pd b.falciparum(blackwater fever) c. PNH 68. The cause of death in gas gangrene is due to ; A. Bacteremia B. Toxic shock C. Pneumonia 69. which is not the content of mediastinum...? a.lungs b.heart c.trachea d.vagus nerve e.large vessels 70. Diseases transferred thru vertebrate animals to humans: a)zoonosis b)zonoarthosac c)Zoncosis d) anthropozoonosis 71. about calcitonin... a.must b given parenteral b.may b used in intoxication of vid D c.hypoparathroidism d.a polypeptide of 32 aminoacid 72. most common conginital heart dsz? a)asd b)vsd 73. A man with AIDs who has no symptom with CD4 count 800 presented with suppurative otitis media, causative organism is: A. Mycobacterium B. Strep pneumoniae C. Pseudomonas aeruginosa d)stap aureus 74. A pt can survive without mechanical ventilation, if the lesion is ; A. Above C2 B. At C3 C. At C4 D. Below C5 75. Preganglionic sympathetic nerve fibers release ; A. ACH b. Norepinephrine C. Dopamine 76. 5th Post intercostal artery a. Thoracic aorta 77. Which one of the following is a COX II inhibitor ; A. Aspirin B. Indomethacin C. Meloxicam 78. Fasting blood sugar of pt is 6.8 mmol (122 mg/dl) after 1 hr of OGTT it is 10.9 mol and later after 2 hr postprandial it was 10.7 mol ( 194 mg/ dl ) what is the diagnosis A. DM B. impaired glucose tolerance C. Long lag phase 79. Not recorded on ECG normally A. SA NODE b.LEFT ATRIUM c.AV NODE d.RIGHT ATRIUM 80. After 48 hr of starvation, wht will be broken down to provide energy,; A. Carbohydrate B. Fat C. Protein 81. Which of the following causes increased interstitial pressure; A. ↑ capillary permeability B. ↑ colloid osmotic pressure 82. Post op wound causes greenish pus discharge . Which organism is involved ; A. Psudomonas B. Klebseilla C. E. Coli 83. Infection spreads retroperitoneally, which will b infected a) Spleen b) Jejunum c) Transverse colon d. Ascending colon 84. Sprain of ankle leads to eversion of ankle, which ligament is damaged . a. Deltoid b. Talofibular c. peroneal d. tendocalcaneous 85. A pt, wd DK didnt respond to treatment and died, Autopsy show inflammatory lesion in kidney brain and lung, with some fungus. Most likely organism is ; A. Mucormycosis B. Candida 86. Person naked in room, temperature 21 degree centigrade, humidity present, loss of heat by ; A. Insensible perspiration B. Sweating C. Conduction and radiation 87. Microscopic feature of polyarteritis nodosa is ; A. Granuloma B. Fibrinoid necrosis C. Associated with hepatitis c 88. Which of the following structure damages in thyroidectomy ; A. Recurrent laryngeal nerve damage B. External laryngeal nerve damage C. Laryngeal oedema D. Sore throat E. Vagus nerve damage 89. ACTH effectively controls ? a.Aldosterone b.Adrenal androgens c.Adrenal estrogend.Hydrocortisone e.NE 90. which cannot b measured by spirometry 1.residual volume 2.expiratory reserve volume 3.inspiratory reserver vol 4.tidal vol. 91. the following diseas occur in iv abuser 1)infective endocarditis 2)llibmansack endocarditis 92. Normal FEV1/ FVC ratio is ; A. 0.6 B. 0.8 C. 1 D. 5 93. Pt unable to void after fall which of the following section of spinal cord is affected; A. L2, L3, L4, L5 B. L1, L2 C. S1, S2, S3 D. S2, S3, S4 94. osmotic diuretic effects on which part of nephron a.proximal tubule b.collecting duct c.asc loop of henle d.glomerular memb 95. After rapid loss of 2 liter of fluid . person drinks plain water . it ll lead to A. Increase in interstitial fluid only B. Increase in intracellular fluid volume C. Plasms volume increase 96. A patient known smoker in laryngoscopy found laryngal nodule what is most unlikely 1 neoplasm 2 hyperplasia 3 keratosis 4 atrophy 97. secondary center of ossification a.epiphysis b.epiphyseal plate c.diaphysis 98. .liq uefactive necrosis. a.brain b.kidney c.spleen 99. 1st heart sound a.isovolumetric contraction 100. A 40 year old man has pain felt deep in the face and nose between the mouth and orbit to the ear and temporal region . the pain may be due to the lesion of the ; A. auricular nerve b. facial nerve c. hypopharyngeal nerve d. maxillary branch of the trigeminal nerve e. opthalmic branch of the trigeminal nerve 101. EBV nasopharyngeal carcinoma 102. inverse stretch reflex(resulting relaxation) golgi tendon organ 103. fast sharp pain A delta fibers 104. 3rd ventricle bounded laterally by 2/3 thalamus (Rabia Ali MCQ) 105. glycogen breakdown aerobic and anaerobic break down to pyruvate 106. fatty acids co enzyme biotin 107. %age units/total units X 100 108. The diagnostic lab test for the infection by streptococuss beta haemolyticus is ; A. ASO titre B. Blood culture C. ESR D. Leukocytes culture E. Urine for bile pigment 109. disclosing pt's secret info A. only with pt. consent B. Insurance Claim 110. Most common cause of inc bleeding time a.thrombocytopenia b. Aspirin 111. opsonin C3b 112. plasma cells produce antibodies 113. black water fever a.p.falciparum 114.Sulphonylurea?? Don’t remember the options but question was asking about mechanism of action 115. alkalinize urine for removing a.phenobarbitone 116. which one is true for parasympathetic sys on Heart a.inc the AV nodal delay b. Decrease K ion outward current c. Increase Ca++ inward Current d. Decrease Inward Na+ current 117. .parasympathetic action a. sweat glands b.salivary glands 118.ectopic pregnancy most common site a. uterine tube 119. both central n peripheral chemoreceptors respond to a..arterial pCO2 b. Increase pH c. Decrease pO2 120. meckel's diverticulum a.vitelline duct 121. .insulin dec by a.beta blockers 122. most sensitive cardiac marker a.trop T b.Ckmb 123. Hypothyroidism a. increased TSH 124. left adrenal vein drain into a.left renal vein 125. fastest B.P regulation a.baroreceptor 126. ESR inc in a.infections b. polycythemia 127.local factor delaying wound healing a.foreign body b. local infection 128.actin covered by a.tropomyosin 129. turner a.44xo 130.klinefilters syndrome a.46XXY 131. S2,S3,S4 lesion a.rectal incontinence b.no pain in labour c. anorgasmia 132. pain referred to shoulder a.c2c3 b.C3,C4,C5 133. compression of S1a.dec ankle jerk 134. not related to rt. kidney a.descending colon 135. external carotid a.lateral to retromandibulr vein 136.symphisis pubis a.sec. cartilaginous joint 137.fibrous and parietal pericardium supplied by a.phrenic 138. corona radiata a.granulosa cells 139. .cimetidine with sucralfate a.decrease metabolism b. Sucralfate decreases cimetidine absorption, give it after 2 hours 140. Cimetidine may significantly prolong the prothrombin time and causes bleeding in pt who has been treated with warfarin by ; A. decreasing the hepatic clearance of warfarin B. Displacing warfarin from plasma protein C. Increasing the oral bioavailability of warfarin D. Inhibiting renal tubular secretion of warfarin 141. .sternocleidomastoid supplied by a.accessory nerve 142.regarding collagen a.present in all types of connective tissueb.unique protein 145.organ for metabolism a.liver b kidney 146. Regarding Portal vein following statement are correct ; Except A. Develops partly from the cranial part of right vitelline vein B. is connected to veins of anterior abdominal wall through paraumbilical vein C. Is formed by union of superior and inferior mesenteric vein D. lies in the free margin of lesser omentum E. tributaries include left gastric and superior pancreatoduodenal vein 147. compact bone... a.Regularly arranged lamelae 148. An initial left to right shunt before reversal is most commonly a feature of ; A. Coarction of the aorta B. Ebstein' s malformation C. Eisenmenger's complex D. Transpostion of the great arteries E. Truncus arteriosis 149. Digoxine unlikly 1.inc Ca intracellular 2.inc K intracellular3.inc Na intracellular 4.inc sympathetic to heart 150. u wave is associated with A..atrial depo B .atrial repo C. ventricular repor 4. ventricl depo 5. slow repolarization of papillary muscles 151. A 25year old sprinter developed acute leg pain while running. the next day he noticed ecchymosis around ankle . he can stand on his toes through it hurts . the most likely cause is ; a. acute arterial embolism b. deep vein thrombosis MNG 168. controls fiber 1a afferent c. Chi square 161.increased at base 191. Decrease CVP 172. Rt side of mid brain 153. Carotid sinus discharge 170. Decreased postsynaptic receptors 186. Produce IgM D. Pudendal damage 164. something about Gamma motor neuron a. increases T4 to T3 conversion 167.A 30 year male came to a consultant clinic with history of progressive right facial numbness and headach for 3 months. epinephrine given slowly??? a. c waves b.Lipid diet a.graft rejection a. lesser splanchnic nerve MOCK TEST 2014 (held on 22 nd Oct 2014) 1. increase in venous volume. Acute Cellular 190.Renal excretion of drugs??? a. Pons C. Shchistosoma hematobium 175. Graves b. Pregnancy c. Greater splanchnic Nerve b. Cartilagenous joint present in the median plane of a body are called as A) symphysis B) synchondrosis C) syndesmosis D) schindylesis Ans: A exp: symphysis are the secondary cartilageous joint present in the median plane of the body each body surface has a plate of hyaline cartilage finally attached to it which in turn blends with a thick strong deformable plate of fibrocartilage these joints allows restricted but appreciable movement 4. Lichen planus b. wht happens in patient gettin TPN a. Liver fluke 166. TOF 192. aPTT 187.Monocytes likely A. increase in arterial blood volume b. Suspected drug addicted complaining of pain 156. Consensual light reflex c. absent waves are a. hyperphospatemia 158. Cerebello pontine angle B. Relaxes muscle b. Increase in diastolic pressure 159. Truncus Arteriosus b. Celiac disease b. beri beri b. 0.Post Commnicating artery a. recurrent laryngeal nerve 162. Diphtheria toxoid given n thn aftr 2 weeks skin rashes . Alpha blockr c.. Fungus D. hyperthyroidism 180 Labetalol a. Increase in allergy C. Standing from 1 hour a. Can transfer into large multinucleated giant cells in chronic infection B. 16 yr old boy with diarhea. and right sided Intention tremors. Nerve loops around arch of aorta A. ruptured Achilles tendon e. Deep veins accompany arteries except in the A) abdomen B) thorax C) cranium D) lower extremities Ans: C exp: deep veins usually accompany arteries except in the cranium superficial veins also do not accompany the artery 3. Renal failure b. Toxoplasmosis is inection caused by a. ruptured plantaris tendon 152. RBC fragility a. The papillae present on margins of the tongue is A) fungiform papillae B) filliform papillae C) vallate papillae D) folate papillae Ans: A exp: fungiform present on the margins filiform are spread all over the dorsum of the tongue va. Squamous cell Ca 178. Virus C. Cause of Squamous Cell Carcinoma bladder a.Accomodation reflex d. on examination.increase surface tension 183.Oral anticoagulant a. most likely reaction a. Stomach c. thyrotoxicosis a. herinated lumbar disc d. Beta blocker b. Cardiac failure c. AV fistula d.c. facial nerve palsy was confirmed on right side along with VIII nerve involvement of same side.35 % NaCl cause complete hemolysis b. contract the intrafusal fibers 173. tapeworm b. most likely diagnosis a. Dry nasal mucosa 177. Damage to sympathetic efferent pathway C. Ceruloplasmin 154. Hypertonic causes shrinkage 171. Compare with a person who ingest 2L of distilled water a person with water deprivation will have a A) higher free water clearence (Ch20) B) lower plasma osmolarity C) lower circulating level of ADH D) higher rate of H20 reabsorption in the collecting ducts Ans: D exp: the person with water deprivation will have a higher plasma osmolarity and higher circulating levels of ADH these effects will increase the rate of H20 reabsorption in the collecting ducts and create a negative free water clearence (C-H20) 8. Liposarcoma c. Decrease in TPR b. jenunum d. Hemolytic anemia question?? Scenario was given and Correct Answer was Hemolytic anemia 163. Dry mouth only c. Failure of aorta to separate from Pulmonary artery a. the most likely site of lesion is . Do not migrate across capillary wall unlike granulocytes 182. Type 4 179.c. Type 1 b. Following are the branches of lateral cord of brachial plexus except A) ulnar nerve B) musculocutaneous nerve C) lateral pectoral nerve D) lateral root of median nerve Ans: A exp: ulnar nerve is a branch of medial cord of brachial plexus 6.Ventilation of lungs a. Hypoxemia produces hyperventilation by a direct effect on the A) carotid and aortic body receptors B) medullary chemoreceptors C) J receptors D) lung stretch receptors Ans: A exp: hypoxemia stimulates breathing by a direct effect on the peripheral chemoreceptors in the carotid and aortic bodies medullary chemoreceptors are stimulated by Co2 or H ions J receptors and lung stretch receptors are not chemoreceptors . Regarding atonic bladder.dry mouth and eyes b.Ocular myasthenia a. A. Rt lobe of cerebellum D. Bacteria 165. Cerebral cortex damage D. false statement about Carbimazole a. type 2 c. Which one is not epithelial tumour a. Parasite b. Peptic ulcer pain a. Duodenum b. something about capillaries 174. Hyperglycemia b. Mitral stenosis. Water and salt reabsorption is maximum at a. Premalignant oral lesion a. Cholecystokinin 184. Giardiasis c.llate papillae are present anterior to sulcus terminalis 2.statistical data is of no significance b.Visual body reflex 185. it is caused by A. Following arteries supplying a long bone arises from arteries supplying muscles attached to the bone are A) metaphyseal arteries B) epiphyseal arteries C) periosteal arteries D) nutrient arteries Ans: C exp: at bone surfaces cortical capillaries make connections with periosteal plexus which are formed by arteries from neighbouring muscles contributing vascular arcades to the fibrous peritoneum 5. Following are the tributaries of great saphenous vein except A) superficial external pudendal B) superficial epigastric C) superficial circumflex iliac D) deep circumflex iliac Ans: D exp: deep circumflex iliac is a tributary of external iliac vein and not the saphenous vein 7. Ileum e. damage to parasympathetic efferent pathway B. Cardiac output low in a. Internal carotid and Post Cerebral artery 176. Sjogrens Syndrome a. Wilsons disease A.Something about Plasma proteins 189.Cushing syndrome a.biopsy showin villous atrophy. Alpha plus Beta blocker 181.ACTH 188. Question from biostatistics (Asked about Wrong Statement) a. Reflex involving cerebrum a Light reflex b. Formed from precursor cells in lymph nodes E. Hypotnonic cause swelling c. a waves 169. IM ketorolac given in a. What wil decrease in hypotension a. Whipples disease 155. Submucosal fibrosis 160. Type 3 d. transmitted from eating undercooked meat a. not improved by gluten free diet. lady with exophthalmos. Adenocarcinoma b. not true abt surfuctant a. Colon 157. Arteriolar dilation leads to a.PT b. Right vein B. Pie test C. After few days his behavior for pain is changed & he become angry after touching. Liver C. Where could be the lesion in brain A. 23. As carboxy hemoglobin C. None of above EXPLANATION: That’s why these pts will come with Amenorrhea/Loss of Libido etc. Platelets Explanation: HLA = Human Leukocytes Antigen. Foramen ovale C. The earliest hormonal deffficeincy clinically evident on hypopituatarism is??? A. Edwards C. B.?? A. RBC C. A localized lesion in which of the following produces prolong coma???? A.and belongs to the chemical class of purine analogues. Short Muscle of thumb is Abductor pollicis brevis flexor pollicis and oppones pollicis these are innervates by median nerve and Adductor pollicis oblique and transverse head innervates by deep branch of ulnar nerve main nerve is median 27. D The third part of the axillary artery is related to the cords of the brachial plexus. Internal capsule B. A patient shifts to the A/E Department with a penetrating injury to the 5 intercostal space at mid axillary line which structure is unlikely to be damage? A Sup: vana cava B. Bone Marrow D.Deep branch of ulnar nerve D.9. It is an immunoproliferative drug Explanation: . Patau. The most common type of ankle sprain is lateral which occurs as a result of excessive inversion of the foot and dorsiflexion of the ankle the calcano fibular and anti: talofibular ligament may tear producing marked swelling and pain these two ligament combined to the posterior talofibular ligament constitute the lateral ligament of ankle 28.Deltoid C. one is inappropriate. Explanation: The second part of the axillary artery is related to the cords of the brachial plexus. Parasympathetic Ganglia. WBC B. IVC C. the plexus is closely related. 40 years old male come to the physician with complainant of some think was protruding out from his anus after defecation 2 year history of blood stained stools on proctosopic examination showed that mucous membrane above the level of anal valves the swelling was Mucous membrane contained large enlargement of vein beneath the surface??? A. Phenylketonurea B. B. D. Latissimus Dorsi muscle D.It is located too far posterior to be damage by the exploratory thoracotomy. Chi square test B. Periaquiducatal region at top of mid brain [Reference Ganong’s Nervous System Physiology Chapter] 24. The main adverse effect of azathioprine is bone marrow suppression.. posteriorly. A. Deep branch of radial nerve B. Turners D. Peyer's patches are present in illeum E. E.Internal Hemorrhoid E. Basal ganglia E. E. Striated brush border is composed of microvilli B. T test D. Which part of blood test should be sent for matching histo compatibility of the recipient and donor A. Skin and subcutaneous tissue B.Cerebral cortex D. In the neck. 17. As carbnic Anhydrase in RBC D. Midbrain Explanation: he has Thalamic Pain Syndrome 14. transverse colon D. Atrophy of the short muscle of thumb mainly which nerve is interpolated A. xanthine oxidase is active metabolite form C. Right pulmonary arty Ans . Xanthine oxidase is active metabolite form.TSH. C. As dissolved state in plasma B. portal vien Explanation: All others are posterior relation 12. which is inappropriate ? A.Perianal abscess C. Adrenal medullary Cells. to the fascia of the scalenus medius C. Pectoral muscle and serratus ant: muscle C. Peneth cells contain Eosinophilic granules in apical cytoplasm C. When thoracotomy incision was made to enter the pleural cavity following structure were incised except which one??? A. Angelamans syndrome 20..The axillary lymph nodes receive 75% of the lymphatics of the breast. The main adverse effect of azathioprine is vasculitis E. LH and FSH C. E Anterior fold of the axilla is higher than the posterior fold. Locus Ceruleus C. Anterior tibial &posterior tibial B.R while packaging nd modification occur in golgi bodies 13. In WBC cytoplasm EXPLANATION: CO2 transport in blood Bicarbonate – 70% Carbaminohemoglobin – 23% Dissolved CO2 – 7% 21. Upper lobe of Right lung E. Patient develops sensory loss over left side of body.Calcano fibular and Anti: talofibular Ans. Carcinoid syndrome D. Second part of dudenum is crossed anteriorly by? A. 11.. Right frontal lobe D. its a prodrug. Regareding Azathioprine ? A.C thExplanation. Otic ganglion lies under? A. The longitudinal coat of muscularis externa is arranged in 3 bands in colon Exp: Parietal cells secrete intrinsic factor 10..?????? A. E. Nicotinic receptors sites includes all of the following EXCEPT. Foramen spinosum B. Malignant melanoma E. D. A. Leukocytes = WBCs. External hemorrhoid B.Median nerve C. Sympathetic ganglia Explanation: M3 receptors (Muscarinic NOT Nicotinic) & beta 2 receptors.Calcaneo_tibial E.. Belongs to the chemical class of pyrimidine analogue D.Medial collateral and lateral collateral D. F Test E.. Synthesis of secretory Vesicles takes place in? A Golgi apparatus B Lysosomes C Ribosomes D Endoplasmic reticulum E Mitochondria Explanation: It occur in E. 26. As question asking unlikely damage structure due to right 5 I/C injury Right atrium most unlikely to damage b/c right atrium of heart which extends free 3 costal cartilage to 6th costal cartilage just to the right of the sternum not in mid axillary line.Chronic anal fissure D. Both Frontal lobes. Stylomastoid foramen 15. ACTH B. E. Regarding histology of gastrointestinal tract.D . Prolactin. 16. 25 years old office secretary wearing high heeled shoes inverts and sprains her ankle while running down a flight of stairs which of the following ligament did she most likely injured? A. Maxillary nerve D. The increase level of 5-hydroxyindolacetetic acid in urine is most commonly found in? A.Complete rectal prolapsed Ans. DIC EXPLANATION: Urinary 5HIAA is the best initial test for Carcinoid syndrome. Major form of CO2 in the blood is as. Correlation analysis 19. Polycystic ovarian Diseases E. Its in active form B. Difference between mean of two samples is tested by ? A.B Explanation.Posterior interossus nerve Ans. Regarding the axilla. As Carbonic anhydrase in WBC. "Menopause"" occur before "Menarche". Alkaptonurea C. 25.B Explanation. 18. Right atrium D. Serum E. Skeletal muscles. External intercostals &inner most intercostal muscle E. Internal intercostals Explanation: Latissimus dorsi is a sheet of muscle that cover the back of thoraeic cage and inserts into the floor of bicipital groove of the humrus . B. The axillary artery terminates at the lower border of Teres major. Thalamus C. Psoas muscl E. 22. Bronchial smooth muscles. Nucleus tractus solitarius. B. chief cells of stomach secretes intrinsic factor D. Explanation . Cefixime B. ClB.Musculo_cutaneous nerve Ans . 40 YEARS OLD WOMEN TAKING ANTI-DEPRESSENT DRUG HER HUSBAND RECENDLY PASSED AWAY SHE DIAGNOSED AS DEPRESSED AFTER START MEDICATION HER COMPLAINING OF DROWSINESS.Dorsal surface D. Horizontal group of superficial inguinal nodes B. Pt with primary Aldosteronism E. Type – 4 H. Ventral surface C. 3 years old children H/O avascular necrosis of head of femur what is the chief arterial supply to the head of femur they compromise and cause the avascular necrosis??? A. 31. duchenn myopathy is X-link dominant B. Facioscapulo humeral dystrophy is autosomal dominant D. Internal iliac nodes E.51 AND PCO2 OF 48mmHg ON BLOOD SAMPLE Na+ =136meg/L K+ 3.Lacunar ligament E.Inguinal ligament Ans.R B.V2. Radial nerve lesion commonly result the fracture of the mid shaft of the hummers’ which injures the nerve in the spiral groove the patient unable to extend the wrist and fingers and there is wrist drop.S.Posterior surface Ans. Femoral sheath is formed by??? A. Branch from medial and lateral circumflex femoral artery D. Type -1 H. Marginal artery D. MEDIAN B. ABGs OBTAINED & SHOWS A PH. THE MEASURE OF CENTRAL TENDANCY THAT IS MOST AFFECTED BY EXTRME SCORE IN A SAMPLE DISTRIBUTION IS THE??? A.S. Pt with diabetic ketocidosis C.72 hours 38.D Explanation: The tongue is the most common intra_oral site of carcinomas most tumors are the lateral margin extending into the ventrum. 30. Tip B. REGARDING THE MYOPATHIES ALL ARE THE FOLLOWING ARE TRUE EXCEPT??? A. Ciprofloxacin D. Type-2 H. Becker’s myopathy is X-link recessive C. AN 18 YEARS OLD BOY PRESENTED WITH TWO WEEK HISTORY OF DYSURIA AND PURULENT PENILE DISCHARGE GRAM STAIN OF URETHERAL SWAB SHOW GRAM NEGATIVE INTRACELLULAR DIPLOCOCCI WHAT SPECIFIC TREATMENT SHOULD HE RECIVE ??? A. CONFUSION.Lateral margin E.B Explanation. Median nerve B. VARIAN CORRECT ANSWER D EXPLINATION. Pre-sacral nodes C. AND FATIQUE DEPLETION OF WHICH OF THE FOLLOWING ECTROLYTES MAY CAUSING THESE SYMPTOMS?? A. THE MANTOUX RACTIONS IS AN EXAMPLE OF WHICH TYPE OF HYPERSENSITIVTY REACTIONS??? A. Pt with vomiting B. mean is the measure of central tendency that most affected by extreme score neither the standard deviation and no variance is A measure of central tendency 40.R C. Na+ Correct answeExplanation: hyponateremia is associated more frequently with selective Serotonin reuptake inhibitors Hyponatremia should be considered in all patients who develop drowsiness . pt with hyperventilation syndrome Correct answer A Explanation: the patient has an alkalemia (ph. Mg+ C. Humoral immune response Correct _D EXPLINATION: tuberculin test is exp: of type 4 HSR or delayed type of hypersensitivity reaction (DTH) this reaction develops when primed Th1 cells encounter their specific antigens an inflammatory response evolves over 24. PSO4 D. Obturator artery B. 20 Years old biker has RTA come to A/E department with compliant of unable to extend the wrist and figures which nerve probably damage??? A.Pectineal fascia D.C Explanation.S.Fascia transversalis & iliac fascia C. Cephradine C. Lymph from the nail bed of the big toe drains into the which lymphatic group??? A. convulsion while taking anti-depressant 35. pt with diarrhea D. Vertical group of superficial inguinal nodes Ans. Crystalline penicillin Correct answer A Explanation: N.Radial nerve D. Co-amoxiclav E. Circumflex artery B.Ulnar nerve C.S. More then 7. cefixime or spectinomycin may give (as stat dose) Fluroquinolones are no longer used as 1 st st line due to the high rate of resistance 37. Superficial circumflex artery Ans-A Explanation: the nutrient artery which is a branch of obturator artery reaches the femoral head in children along the ligament of the head and enters the bone at the fovea capitis 34. Internal pudendal artery C.R E. MODE C.gonorrhoea occurs in young age adult gram stain +ve in 25% Culture +ve 50 % current guidelines recommend 1 line treatment with cefotaxime. Posterior descending artery Correct answer B Explanation: anteroseptal M-l is due to an infarct in the territory of the left anterior descending artery 36.V3&V4 WHICH OF THE FOLLOWING CORONARY ARTERY IS MOST LIKELY TO BE OCCLUTED ??? A. Right coronary artery E. Type – 3 H.R D. Fascia transversalis B. Limb – girdle dystrophy is autosomal recessive . MEAN E. Popliteal nodes D. A 56 YEARS OLD LADY IS ADMITTED WITH 30/MINTS HISTORY OF HEAVY CENTRAL CHEST PAIN ASSOCIATE WITH NAUSEA AND SWEATING HER ECG SHOWS ST ELEVATION IN LEADS V1.7. The femoral sheath (crural sheath) is formed by prolongation downward behind the inguinal ligament the tranversalis fascia being continued down in front of the femoral vessels and the iliac fascia behind then. K+ E.confusion. E 33. Deep circumflex iliac artery E. Left anterior descending artery C.2 meq/L Cl_= 100 meg/L HCo3 =37 meq/L WHAT IS THE MOST LIKELY DIAGNOSIS??? A. 32. The highest rate of occurs of carcinoma of tongue is that of A. STANDERD DEVIATION D.Internal Hemorrhoid are dilation of the tributaries of superior rectal vein they are covered by mucous membrane in the upper half of anal canal protruded down due to enlargement of veins and pressure of defecation the dilated vein comes out from anus 29.45) Since PCO2 is elevated (more 40 mmhg) the pt is primary metabolic acidosis with respiratory compensation an elevation of HCO3 could come from loss of acids as well as loss of intravascular volume 39. GLUT2 in kidney and bowl c. Potassium D. Lidocane with central line c. Intervertebral foramen D. This is the mechanism in pregnancy of how breast glands normally grow. The superior cervical ganglion: all are true except a. Ketone and/or acetone levels d.Splenic flexure e. Verapamil e. What is the single most important test in diabetic ketoacidosis (DKA)? a. NEVER start with radiology in Endocrinological diseases. Defibrillate Explanation: Pt in question is Hemodynomically stable (BP : 120/80) so there is NO Point in defibrillating him. they go back and shut off their own stimulatory hormone. The same is true of ketones. Foramen transversarium Ans. Iron C. 57. shorter than refractory period in skeletal muscle d. Cortisol b. ACTH Answer c. has nerve connections with the anterior lobe of the pituitary gland. Increased estrogen directly stimulates the pituitary to release prolactin. GH c. The earliest finding in hyperkalemia on an ECG is? a. 50. Prolactin makes the breast glands grow. e. First do biochemical test to ensure that disease really exist then scan. Procanamide d. VERAPAMIL is wrong. c) motor neuronal processes. b) thoracic spinal nerves. Estrogen makes prolactin. Glucose levels can fluctuate wildly from high to medium. ECG reveals tachycardia rate of 170 bpm and blood pressure 120\80. Peaked T wave e. Pulmonary congestion D. Calcium B. gives off branches that supply the heart . C 42. 47. D. Which of the following endocrinopathies is associated with high prolactin level? a. Great saphaenous vein does NOT: A.Cecum b. most valves are found below the knee level Explanation: it lies anterior to medial malleolus. AV block d. What receptor is affected most by exercise? a. lasts till half of cardiac contraction c. forms part of the roof of the third ventricle. is responsible for hormones and enzymes regulation. Pulmonary congestion and Pulmonary fibrosis). Cortisol does feedback inhibition on the pituitary and hypothalamus to shut off ACTH and corticotropin-releasing hormone (CRH). QRS widening c. Pulmonary fibrosis Correct answer : A. The descending order of medication (which will be used first) Amiodarone > Lidocain > Procanamide. then widening of QRS complex etc. contains up to 20 valves E.5 cm below & lateral to pubic tubercle D. MRI head d. P wave disappear b. Rectosegmoid [Reference Goljan: 50% cases] 43. Myotonic dystrophy is Autosomal dominant Correct answer A Explanation: Duchenne muscular dystrophy is inherited X-link recessive disorder not Xlink Dominant 41. corresponds to the duration of relaxation b. T4 d. b.Transverse colon d. lies 1 hand's breadth behind medial border of patella C. They are all subject to feedback inhibition—after production. The dorsal roots of all spinal nerves contain a) sensory neuronal processes. Serum osmolarity Answer b. pH c. EXPLANATION: Dorsal roots only contain sensory neuronal processes of spinal nerves. 49. 70 years old man admitted with palpitation to ER. The best initial test for Acromegaly is? a. lie behind medial malleolus B. c) lumbar spinal nerves. Amiodarone b. or acetoacetate. Glucose level b. beta-hydroxybutyrate. Absolute refractory period in the heart a. Addison disease c. The level of these ketone bodies is not as important as the level of accumulated acid. b) sensory and autonomic neuronal processes. Glucose suppression test EXPLANATION: They are asking best INITIAL test which IGF & most accurate test is Glucose suppression test. The hypothalamus: a. 53. Hypothyroidism It is reasonable to exclude hypothyroidism in any person with hyperprolactinemia. In the other 3 conditions ( Decreased surfactant production. Vertebral artery traverses all of the following EXCEPT A. d) motor and autonomic neuronal processes. Testosterone c. contains both parasympathetic and sympathetic fibres b. GLUT4 in skeletal muscle Exercise does not affect any of the glucose transporters except the one in skeletal muscle 51. GLUT1 in brain b. GLUT3 in neurons d. Selenium Ans. Which of the following elements is known to influence the body's ability to handle oxidative stress? A. Cushing syndrome or hypercortisolism d. 45. T wave flatting Explanation: Initial change on ECG is Peaked T wave. the pulmonary compliance is decreased. Hypothyroidism e. acetone. but if the patient’s pH on an ABG or serum bicarbonate on chemistry is near normal. COPD B. contains the tuber cinerueum. 52. Estrogen deficiency b. d. GH deficiency Answer d.Ascending colon c. COPD Pulmonary compliance is increased in emphysema (a type of chronic obstructive pulmonary disease). All ventral roots contain motor processes and the thoracic. Foramen magnum B. 58. lumbar and sacral spinal nerves also autonomic nerve processes. c. Decreased surfactant production C. Sympathetic preganglionic axons exit the CNS only with the thoracic and upper lumbar spinal nerves 54. Selenocysteine is part of enzyme glutathione peroxidase which has antioxidant action 44: The most common site of carcinoma of the colon is a. ONLY arterial pressure has NOTHING to do with venous return when pt change from supine to standing position. For Rate control the best INITIAL drug choice is Amiodarone with central line. it does not matter. d) sacral spinal nerve Explanation: Parasympathetic axons exit the central nervous system only with sacral spinal nerves and four of the cranial nerves. 55. lasts till cardiac contraction 56. Pulmonary compliance is not decreased in? A. GLUT4 in skeletal muscle Answer d. drains into femoral vein 3. 46. Which of the following is the only hormone made continuously? a. The placenta makes estrogen. which are made continuously. 48. Parasympathetic preganglionic axons leave the CNS with the a) cervical spinal nerves.E. forms part of the mid-brain. Venous return of lower limb on standing from the supine posture depends on all except A) Deep fascia sleeve B) Arterial pressure C) Valves of perforators D) Calf muscle contraction EXPLANATION: All others options do has effect. Growth hormone (GH) e. which is ideal to give a. Subarachnoid space C. pH The glucose level is not as important as knowing if the patient is acidotic. T4 All hormones are made in pulsatile fashion except for T4 and T3. Insulin-like Growth Factor (IGF) b. extravascular b. generally blood flow is laminar b. Hep B C. Serious adverse effect of morphine? a) Extreme sedation b) Increased intracranial pressure c) Decreased respiration d) Decreased myocardial conductivity e) Decreased blood pressure EXPLANATION: Respiratory depression is one of the most serious adverse effect. myelin figures.Asthma b. nuclear disaggregation of granular and fibrillar elements. HIV. 73. Cell swelling b.acidosis b. age EXPLANATION: ESR has DIRECT relation with Fibrinogen & Globulin (will increase with increase in these two) while INVERSE relationship with Albumin (will Increase with Decrease in Albumin) 66. None of the above Ans: BBBB [Glucose via Facilitated (mainly). Irreversible cell injury: mitochondria swell. lateral c. All of the following are features of apoptosis EXCEPT: a. Malaria.condensation of nucleoli b. hydropic change. d. ascending aorta have turbulent blood flow c.plasma membrane blebbing & blunting EXPLANATION: Reversible cell injury: cell swelling. Simple diffusion D. detachment of ribosomes from granular e. Regarding histology of cervix. villous distortion. 64. transformation zone is a region where over time more fragile squamous cells replaces columnar epithelia cells. c. Amino-acids via Na.lysosomal swelling c. and dissociation of polysomes into monosomes. Sterilization can be achieved with all.cells of ectocervix will nt collected in pap smear d. Except? a. except a. damage to plasma membrane and lysosomal membranes leads to enzyme leakage. Silicosis is most often complicated by a. . Na co-transport E. mitochondria damage is hall mark Explanation: Mitochondria will INTACT in Apoptosis. x-ray skull show larged pituitary. interiovascular e. All of the following are features Irreversible cell injury. Inc numbr of chromophobes D.Tuberculosis e.In parous cervix appear as small slit like opening in center. 71. which of following is true? a. exterio interior EXPLANATION: Normal hemolysis (i.c. Tall boy 14 yr old secretion from mammary gland. blunting. Formation of cytoplasmic blebs d.The SCG is located opposite the second and third cervical vertebræ. Hep E Ans: Hep D (She is NOT pregnant. gamma irradiation c. b. Histological exam of pituitary is likely to show.proteolytic cleavage e. Regarding Blood Flow dynamics All are true. Histologically characterized by pallor.Brucellosis B. 68. rarefaction. The relationship of the heart to lung is? a. squamous epithelium layer forms mucous secreting cells over endocervix b. globulin d.damage to plasma membrane e. So it is medial to lung. so E is unlikely and the HIGHEST mortality is associated with Hepatitis D virus) 75. Glucose Transported from mother to fetus by: A. is the only cervical ganglion that supplies the cranial nerves d. lysosomes swell. albumin e. . Condition is not associated with sinus bradycardia? A. Pasteurization b. medial d. gives off branches that supply the sweat glands of the face e.shirinkage of cytoplasm d. bronchi alveolar carcinoma [Ref: Goljan Respiratory Chapter] 70. medio lateral EXPLANATION: Heart is surrounded by Lungs on either side. acidosis somewhat protective by inhibiting enzymatic reactions. e. c. Hep C D. fibrinogen c.r. Hep D E. Inc number of eosinophils 65. moist heat at 1340C for 3 minutes at 202kPa d. Chromatin condensation c. Bacterial. Transmission of which type of infection is most likely to occur after platelet transfusion? a. Typhoid fever EXPLANATION: Leptospirosis is associated with Tachycardia. Morphologically hallmark of apoptosis is. turbulence decreases the perfusion pressure d. superior b.mitochodreal swelling d.Cotransport] 74. Leptospirosis C. dorsal e. Syphilis. 67. Fatty change encountered in cells invloved in fat metabolism (hepatocyte. HAV B.Carcinoma of lung c. Hepatitis B.Hypothyroidism D. Contrary to our common belief that ONLY Typhoid is associated with Bradycardia there are many other conditions (3 are given here) like Falciparum Malaria etc. INTRAVascular hemolysis is ALWAYS pathological. Primary active transport B. turbulence increases with increased raynolds numbers e. Dec number of pituicytes B. Normally (Physiological) Hemolysis is a. Phagocytosis of apoptotic bodies EXPLANATION: Check Question 71 explanation. mitochondrial swelling.karyorrhexix c. intra vascular c. ethylene chamber e. A. Tndylization 60. 72. EM: plasma membrane blebbing. vacuolar degeneration. myocardium). ESR is increased with decreased in? a. 59. turbulence can generate murmurs EXPLANATION: Turbulence will INCREASE perfusion PRESSURE. 61. infection b. except a. extra + intravascular d. during pre adulscent endocervix is located in vaginal portion of cervix 63. except? a. Lack of inflammation e.Mesothelioma d.Advanced liver disease E. 62. You have to start Nalaxone to such pts. Facilitated diffusion C. e. All of the following are characterized by granulomatious inflammation except a) Sarcoidosis b) Tuberculosis c) Histoplasmosis d) Diphtheria e) Leprosy EXPLANATION: You don’t need it (explanation) ☺ 69. Inc numbr of basophil cells C. A young female developed nausea vomiting and jaundice & suddenly (within hours) she died which is most likely in this patient? A.e destruction of RBCs after 120 days) take place in SPLEEN and Extravascular hemolysis mean Spleen & Liver. Compounds containing histidine Answer: CCCCC (Ganong’s MCQ) 90: Compared to hepatic bile. The Most common cause of death in Adult polycystic kidney disease is? A. This is all a sign of increased total lung capacity. Carbonic acid D. What test should be done to confirm the presence of vitamin B12 deficiency? a. Chlamydia and viruses are intracellular. this establishes the diagnosis of myeloma. Acetylcholine b. What is the greatest stimulant to saliva production? a. Vitamin B12 deficiency EXPLANATION: Any of the microcytic anemias can be associated with target cells. Left optic nerve C. Lisinopril c. The greatest stimulant.3-GPD inside RBCs will decrease which move AWAY Curve toward LEFT and the result will no NO Shift ) 86. it stains the cell wall 88. Reticulocyte count c. Iron deficiency c.3-GDP Answer: D (In Option D – will remain Same – No shift. Right EdingerWestphal nucleus D. Folic acid deficiency d. Superior colliculi B: The absence of both the direct and consensual light reflex response when light is shone in the left eye indicates a problem with the afferent part of the pupillary light reflex pathway on the left. however. Ninety-five percent of patients with SLE should be positive for ANA. Anti-Smith d. MCV b. 84. Right visual cortex E. Peripheral blood smear Answer b. What is the single most specific test for multiple myeloma? a. They are used to identify the etiology of the deficiency. The acetylcholine receptors in the salivary glands are muscarinic receptors not nicotinic receptors 77. Chronic Acidosis E. Left occulomotor nerve (CN III) B. is acetylcholine. All of the following will cause Right shift of Oxygen dissociation curve. 87. Antinuclear antibody (ANA) Although the ANA test has no specificity for SLE. No cell wall b. 79. or the part of the air that cannot be exhaled. When combined with a monoclonal spike on SPEP and lytic lesions. Arteriovenous O 2 . BACTERIAL transmission is common as their growth is promoted. You can have Bence Jones proteinuria alone without myeloma. but was normal in both eyes when light was shone in the right eye. Epinephrine c. Autoimmune hemolysis b. A normal response on both sides when light is shone in the right eye demonstrates that the efferent parts of the reflex pathway. Methylmalonic acid level Methylmalonic acid levels build up in vitamin B12 deficiency. What is the physiologic basis of a barrel chest? a. Which of these can be associated with target cells? a. Gram stain does not stain DNA. Mycoplasma is encased in a cell membrane that does not pick up Gram stain. Hemoglobin B. Bence Jones protein e. A 41-year-old woman was taken to the emergency department after being involved in a motor vehicle accident. The superior colliculus and visual cortex are not involved in the pupillary light reflex. Optic disk D. No stainable DNA Answer a. but not usable lung. Increase 2. 82. Methylmalonic acid level b. RF e. Iris C. The most likely of all the anemias to cause target cells is thalassemia. Which of the following parts of the eye has the greatest concentration of rods? A. Renal Cell Carcinoma C. Autoimmune hemolysi causes spherocytes not target cells. Reticulocyte count Reticulocytes are the measure of new cell formation. by far. are unaffected. and the Schilling test are not used to discover whether there is a vitamin B12 deficiency. Nifedipine e. Protons Answer: BBBBB (Ganong’s MCQ) 91. Berry Aneurysm B. H 2 PO 4 E. Urine immunoelectrophoresis d. 78. Hemoglobin level d. it is the test most commonly found to be positive. Decreased compliance d. both diminish. These increased volumes are all residual volume. gallbladder bile contains a reduced concentration of which of the following? A. Bone marrow biopsy Looking for >10% plasma cells on bone marrow biopsy is the single most specific test for myeloma. Sodium Valporate b. Decrease pH D. this means the ANA has an enormously strong negative predictive value for the disorder. Decreased FEV1 Answer b. Increase Temp C. Which one of the following is most likely to be responsible? a. Bile acids B. as initial acidosis will cause Right shift but with pessage of time the level of 2. Which test is most likely to be positive in SLE? a. Vitamin B12 deficiency can have a low normal level in 10% to 20% of patients routinely. Nicotinic receptors Answer a. 80. Other proteins C. Anti-intrinsic factor/antiparietal cell antibodies c. 65 years old man with Hx of hypertension & epilepsy is noted to have gingival hyperplasia on examination in the cardiology clinic. Anti-Jo Answer a. Homocysteine level increases in both vitamin B12 and folate deficiency. At which location could a lesion account for this pattern of deficit in the pupillary light response? A. RDW Answer b. What will change first in response to iron replacement in Fe deficiency anemia therapy? a. Why does Mycoplasma not appear on Gram stain? a. Too small d. Increased residual volume c. Intracellular c. Sodium ions C. Increased fibrosis of lungs b. Atrovastatin d. Ciliary body B. Acetylcholine EXPLANATION: Saliva production is the only part of the body in which the sympathetic and parasympathetic nervous system both lead to increased production of a substance. 81. Reticulocyte concentration has to increase first for any of the other features of a CBC to change. Anti-intrinsic factor. The only thing that gives >10% plasma cells on bone marrow biopsy is myeloma. the diaphragm flattens and the chest enlarges. Increased residual volume As air becomes trapped in the lungs. Some cancers can give lytic lesions. 76.Explanation: Platelet are stored at room temperature and at room temp. Homocysteine level Answer a. Parafoveal region Answer: EEEE (Ganong’s MCQ) 89: Which of the following is the principal buffer in interstitial fluid? A. Increase pCO2 B. Bone marrow biopsy c. the FEV1 and FVC. SPEP b. Antinuclear antibody (ANA) b. The monoclonal gammopathy of unknown significance (MGUS) accounts for 99% of IgG spikes on SPEP. Fovea E. EXCEPT: A. No cell wall Mycoplasma does not have a true cell wall. Chronic Renal failure D. Both alpha. antiparietal cell antibodies. Neurologic examination showed that the papillary light reflex was absent in both eyes when light was shone in the left eye. Carbamazipine EXPLANATION: Anti-psychotic drug which causes gingival hyperplasia is Phenytoin NOT Valproate or Carbamezipine. Protons D. Schilling test d.and betathalassemia cancause target cells and thalassemia can also be caused by thalassemia trait. The usable part of lung volume. Liver Cirrhosis Answer: CRF (Ref: Goljan Kidney Diseases Chapter) 85. including the Edinger Westphal nucleus and cranial nerve III. Splenic Rupture E. Which of the following parameters is decreased during moderate exercise? A. Calcium channel blockers (especially Nifedipine can also cause gingival hyperplasia) 83. Glucose E. Anti-dsDNA c. Norepinephrine d. Preganglionic autonomic fibers are which type of fibers? A. Vitamin D: steroid hormone mechanism. cardio-pulmonary failure B. 94... ADH D. V2: cAMP mechanism] 98. cleft lip cleft palate. Lower half of left border of sternum 19.both lower limbs swelling and bluish discoloration . 3Rd left ic space D.. Better control of symptoms Explanation: Adding progesterone increases the risk of breast cancer while decrease the risk of endometrial cancer 99. Tricuspid auscultation area? A. Thiazides causes. bowel disturbance. X-ray chest B. vitamin b12 def can be found out by urine test of methylmalonic acid pyruvic acid . Sickle cell disease D. ACTH: cAMP mechanism. C fibers E. A gamma fibers C. Angiotensin II C. 9. Insulin B.. Morphine Tramol butyrophenie fentanyl 21. 95. MRSA sensitive to: aztreonam co amoxiclav clotrimoxazole metronidazole 2. mcv mch mchc all raised . ADH (V1: IP3 mechanism. 2nd right ic space B. microcephaly . 2Nd left ic space C. Mitral area E. PTH-rp with squamous cell carcinoma. Drug given: Erythropoietin Frolic acid Vit b6 Vit b12 18. whats the diagnosis : a: 45X b: 46XXy c: 74 XY+13 d: 74 XY + 18 11. Cefixime B. Increase risk of endometrial cancer E. Ceftaroline C. Neurophysiology chapter] 97. Sarcoidosis E. TCA.normocytic normochromic anemia . a lady delivered 34 weeks baby who was at 30th percentile of weight and height. Pt of chronic renal failure going to dialysis. primary hepatocellular carcinoma : aflatoxin b1 hep E hep C alpha 1 antitrypsin deficiency 3.. antibody with Minimum life. Which of the following is the major immunoglobulin in human serum. (3) IgE = Reagenic antibody.. Which one of the following would result from the use of a combined OCPs compared to an Estrogen only: A. MRSA is resistant to ALL others cephalosporin. cancer cells combatted by : natural killers (BOTH NK cells & CD8 are Anti-Neoplastic) cd8 and t lymphos cell mediated 13. Earliest antibody to be synthesized and Cold antibody. Eisenmenger’s Syndrome C. what is false: A. Increase risk of stroke C. Cushing’s syndrome B. Hyponatremia D.. detected by serum vit b12 levels intrinsic factor antibodies 8. Cephalexin Explanation: The ONLY cephalosporin with MRSA coverage is Ceftaroline.difference B. Pulmonary Arterial Hypertension may be seen in each one. Braodman area 3 1 2 1 recieves all somatic sensation 2 special sensory 3 granular cortex something 4. Which one of the following hormone has 2 different mechanisms action on its receptors? A. sedation B.. Smoking C. Peak expiratory flow E. Angiotensin II: IP3. Decrease risk of thromboembolism B.. Lambert Eaton Syndrome E.. alpha blocker C. 55 years old woman had hysterectomy presents for advice about HTR (Hormone Replacement Therapy). EXCEPT? A. Hypokalemia Hyponatremia Hypocalcemia 16. Breast Milk 93. Ceftriaxone E.. Heart rate C. PTH related protein EXPLANATION: Small cell is associated with Cushing’s (due to ectopic ACTH) & SIADH (due to ADH production). accounting for 80% of the immunoglobulin pool? A) IgA B) IgD C) IgG D) IgM E) IgE Answer: CCCC Below are the some unique characteristics of each antibody: (1) IgG = Smallest (in size) antibody. Pressure B. 1st line phagocyte in blood neutrophil monocyte macrophage old aged Person living at 12000ft had a hx of of MI 6 year back. Na+ influx K+ influx Cl. Hepatitis B B. axial something . Hyperpolarization is caused by. venous thrombosis C... (2) IgM = Largest in size. Histamine releasing opioid analgesic. Cardiac output D. A beta fibers D.. Velocity C.influx 17. Parotid ducts opens into (Asim MCQ) Second upper molar teeth between cheeks and teeth 20.. Heat labile antibody. B fibers [Ref: BRS Neurophysiology Chapter table: Kindly do memorize this table] 96. serology not helpful in a) toxoplasmosis b) ambisis c) giardiasis d) ameobic liver abscess 6 Byssinosis occurres in people working in 1)garment factory 2)lead factory 3)textile industry 4)cement factory 7. Total peripheral resistance (TPR) Answer: EEEEE 92.. HIV EXPLANATION: Hep B is NOT associated with Pulmonary HTN. 2 year old boy comes with fever. Tapping [Ref: BRS Physio. Worst prognosis C. Vitamin D E. Vibration D.. CO Poisoning 10. 6 fingers in both hands . Ceftazidime D. Esophageal stage of swallowing most effected by : myasthenia cerebral cortex stroke just above lateral gyrus Scleroderma 12. antibody that is mainly Intravascular. CBC is helpful in the sense that it will EXCLUDE secondary polycythemia. (4) IgA = Antibody that protect surfaces. A Delta fibers B. Which one of the following is least associated with small-cell lung carcinoma? A. ACTH EXPLANATION: Insulin: Tyrosine kinase. antibody that is present in body fluids like Saliva. DIC is stimulated by?? tissue thromboplastin 15.cause A. which of the following is least relevant? A. EXCEPT? A. Increase risk of breast cancer D. MEDICINE & ALLIED 4 th JUNE 2014 (Paper A & B – Mixed) 1.. One that crosses placenta and antibody that is known as a Warm antibody. Sympthomimetic D. beta agonist activity 14. Virus causes cancer via 1 proto oncoogene 2 oncogene 3 nucleic acid 4 promotr gene Byssinosis occurres in people working in 1)garment factory 2)lead factory 3)textile industry 4)cement factory OPD . A. 100: Which of the following Cephalosporin is effective in MeticillinResistant Staphylococcus Aureus (MRSA) infections? A. Pulse pressure E. Receptors of all of the following are rapidly adopting receptors. Highest concentration in serum. CBC D. Patient with suspected COPD.. Spirometry EXPLANATION: As per recommendations Peak expiratory flow has NO value in COPD management. protein fat carbohydrates deposition by A. flat plate C. Post ileal 39. defective clot formation i guess A. Woman with borderline lab values of MCV. well develped brest .. thromboplastin 5. prot CHO cho Prot fat 40. Astigmatism inappropriate about adrenal gland 61. intercalated disc 42.6 intercostal spaces at posterior side.pain in flexion & Medial Rotation of hip.lipase 28. metaphysic what white blood cells are raised in acute viral infection A. Astigmatism inappropriate about adrenal gland neonate adrenal is same the size of adult adrenal gland Artificial respiration is not necessary if lesion occur at. Skeletal muscles A..below c5 An immunocompromize pt developed s/s of pneumoia BAL fluid revealed boat shaped cyst organism (Nov 2013 Medicine paper MCQ = repeated) Pt unable to invert his foot while the lateral rotation is intact structure involved A. No retrosternal pain also not radiated.. Shock characteristic: tissue hypoxia low BP tachycardia 43. Dr gv some medication due to which Pt conditions improved. fibrogen B. Stabismus C. tibialis ant & tibialis post B. a patient known case of lambago comes with sudden left side chest pain.vision not clear A. Dissection of aorta: medial necrosis (Cystic Medical Necrosis – Rubin’s Pathology) 30. TBM B.c2 B. A. Female with lush push bodyline . are innervated by somatic nervous system B.. Pt with nose lesion . Neutrophils B. (Asim & Shoaib MCQ) A. Neurogenic shock mediated by: loss of vasomotor tone heart pumping decrease 45.. A..amylase 2... uterus not palpable. Subclavian artery arches over: (Asim’s MCQ) clavicle 1st rib 2nd rib 31.above c5 D. liver in its place coz of attachment with IVC ligaments 56.. proteinuria leprpsy wegner's 26. Preileal D.. what sound comes on rapid filling of lft ventricle 48. wound healing ascorbic acid 34. HIGH Protein & Normal Glucose (Asim’s & Chandkian’s MCQ) A. If Ejection fraction increase (Rabia Ali MCQ with wrong key AND BRS Physio MCQ) EDV decrease ESV decrease 50. glucagon C. GH B. Appendix . Takayasu neonate adrenal is same the size of adult adrenal gland Artificial respiration is not necessary if lesion occur at.. fats .vision not clear A..c3 C. Neutrophils B.above c5 D. granuloma. Vitamin K deficiency. A. thrombin 4. 5-10 yr B. Steatorrhea due 2 chronic pancreatitis occur due to deficient: 1. something about secondary active transport (Amino Acid absorption in PCT) 47. male psuedohermaphrodite female pseudoherma testicular feminization 29. 60. Neutrophils B. testes in inguinal region.. spasticaortic opening aneurysm structures compressd.temporal atery biopsy A..22. protein fat. factor 9 def.vision not clear A. aptt high PT low what white blood cells are raised in acute viral infection A. S3 S2 49.azygous vein and thoracic duct 3... oblique fissure in lung extends from : (Asim’s MCQ) t3 . What’s the hallmark of aids: progressive immune-depression opportunistic infections Proliferation of virus inside Tlymphocytes 32. epiphysis D. Lymphocytes 37. tibialis ant & tibialis post B. fibrin 70 yeard old lady. more than 10 yr36.. epiphyseal plate D. Stabismus C. Which hormone not required for growth of sperm: GH bHCG Prolactin FSH 27.c2 B. There is lesion of upper trunk.. Prothrombin 54. Pelvic B.c3 C.above c5 D.t6 t2 t6 t4 t6 59. 2nd order neuron of taste (ant 2/3 tongue) 33. whats wrong: 50% offspring effected complete penetrance 50% females effected Variable expression 35. But tenderness at 5. What’s mode of action of medication: anticholinesterase chlonergic anticholinergic 41. Autosomal dominant. Bacterial meningitis 52. Giant cell artiritis B. Peripheral portion of Metaphysis is supplied by separate artery – Metaphyseal artery) E. sixty year old lady having unilateral severe head ache. Cataract B. tibials ant & fallicus H longus 38. what white blood cells are raised in acute viral infection A. 1st order neuron of taste (ant 2/3 tongue) B. Cataract B. Lymphocytes 53. Cataract B. radiation causes cancer usually in : A. Diaphysis (Supply Diaphysis & Central portion of Metaphysis. short vagina . Scenario about nerve supply to tip of the nose Opthalmic division of trigeminal nerve 44. Decrease motor tone B. Osteogenic imperfecta Defect in synthesis of collagen 55. Stabismus C. Retrocecal C. complained amenorrhoea since puberty . body utilizs nutrients in what order: CHO.. Jugulodiagastric lymph node drains into Palatine tonsils and tongue 57. insulin 70 yeard old lady.below c5 An immunocompromize pt developed s/s of pneumonia BAL fluid revealed boat shaped cyst organism (Nov 2013 Medicine paper MCQ = repeated) Pt unable to invert his foot while the lateral rotation is intact structure involved .. myasthenia gravis Pt. are non straited C. Nutrient artery supplies: A. Right adrenal gland Pyrimidal in shape 58. All wil occur except (Asim’s MCQ) adduction of arm medial rotation of arm Sensory loss over forearm Elbow extension Supination 24. in tractus solitarius A.predominates lymphocytes. Lymphocytes Christmas disease factor 8 def 70 yeard old lady. MI Costochondritis Aortic aneurysm Pleuritis 23.c2 B.. Cardiac enzymes r normal.. tibials ant & fallicus H longus Vomiting center situated Medulla oblongata Lower motor neuron lesion A. Lumber puncture . Typhoid diagnose in 2nd week best by : widal blood cs blood cs plus widal only widal stool cs 25. Astigmatism inappropriate about adrenal gland neonate adrenal is same the size of adult adrenal gland Artificial respiration is not necessary if lesion occur at. MHCH etc: microcytic normo macrocytic macro micocytic hypochromic 51. Scenario on lacrimation while eating like that Facial nerve 46.c3 C.below c5 An immunocompromize pt developed s/s of pneumonia BAL fluid revealed boat shaped cyst organism (Nov 2013 Medicine paper MCQ = repeated) Pt unable to invert his foot while the lateral rotation is intact structure involved A. below c5 An immunocompromize pt developed s/s of pneumonia BAL fluid revealed boat shaped cyst organism (Nov 2013 Medicine paper MCQ = repeated) Pt unable to invert his foot while the lateral rotation is intact structure involved A. A. Secretary vesicles found in organelle A. (Ref: Textbook of Diabetes – Table 24-6) B. Pt for renal transplant . what is Appropriate? Not given in pregnancy Rapidly antagonized by FFP 85. Central Diabetes Insipidus B. SIADH C. Nutritional deficiency. C... 45x0/46xy.. stratified sampling B. Kidney C. Nephrogenic Diabetes Insipidus . acidosis.. Squamous cell Carcinoma marjolin ulcer bowen's disease 68. C. ascending colon C. Drug which enhances the hypoglycemic effect of sulphonylurea? A. Skeletal muscle th Ed. Neutrophils B. Stabismus C. Cataract B. Thyroid hormone D. Starling law of heart (Ganong’s MCQ: Ch#30. C. inc progesterone 91. L2 B. Venous return increased by A.. B. stone in the parotid duct (Asim’s MCQ) 65.HB curve to the extreme left. Q# 6) A. Brain D. tibials ant & fallicus H longus Vomiting center situated Medulla oblongata Lower motor neuron lesion A. stomach B. Elevated metabolism D. Golgi apparatus (Transport vesicle from ER to Golgi Modification of Transport vesicle inside Golgi Secretary Vesicle) B. thiazide.azygous vein and thoracic duct 62.. GH B.SHE LATER DEVELOPED complaint of polydypsia and polyuria. Warfarin given for DVT. explains the increase in cardiac output when the sympathetic nerves supplying the heart are stimulated. does not operate in the failing heart. Heart has definite form (NOT True as Upper as it occurs at the end of 8 week ) B. Astigmatism inappropriate about adrenal gland neonate adrenal is same the size of adult adrenal gland Artificial respiration is not necessary if lesion occur at. 2*2 TABLE Chi-square testing 88. Rapidly adopting fibers. Valve in veins C. 5 87.A.) week & Lower limb in 5 86. Not supplied by vagus nerve (Asim’s MCQ) A. Hormone that stores carbohydrates. Meningitis after pyogenic lung abcess organism? (Asim’s..alopecia Zinc 76.lab findings were given in which urine osmolarity is low and plasma osmolarity high. spastic aortic opening aneurysm structures compressd.. simple sampling 89. Staph B. Autosomal recessive 97. Neostigmine action: inhibit Ach esterase 101.. Decrease motor tone B.. E. Insulin 94. D. 95. Endometrial hyperplasia A. pacinian corpuscles. lipids and proteins in cells is also called hormones of abundance A. A. Estrogen E. Heart B. epithelium changes in endocervix Metaplasia dysplasia 67. ER 79. then Option B is appropriate. Trigeminal ganglia A. High pressure oxygen for 24 hrs (Scenario of Some Smoker).. Cristanee concavity toward nucleus Golgi apparatus 80. meissner 75. Urinatio 73.intelligence normal (case of achondroplasia) A. Furesimide 99. Organ with high atriovenous (AV) 02 difference at rest? A.. Spontaneous Pneumothorax 84. Inhibit gastric acid secretions (Asim’s MCQ) Secretin 72. D. A.azygous vein and thoracic duct 70. Freely bathed in csf C. spastic aortic opening aneurysm structures compressd. tibialis ant & tibialis post B.scaly dermatitis. E. 93. C. bile acids absorbed in ileum 63.. A LADY HAD AN ROAD ACCIDENT AND GOT HEAD INJURY. Apoptosis necrosis degeration 66. BRACHECEPHALY AND MENTAL RETARDATION B. dec estrogen C.vision not clear A. PYOGENIC PEROTINITIS A. Chandkian’s & Rabia Ali MCQ) A. fetal Hb . Which will shift the O2. Phenylbutazone.3 bis 78. Respiration E. After giving vasopressin urine osmolarity increased and plasma osomalilty decreased…what is diagnosis??? A. Explain the increase in cardiac output that occur when venous return is increase B. C7 82. Completely covered by dura B. A. By vaporization of sweat C. Autosomal dominant B..short height.. Sample which is age based grouped then random people taken from each group. Derived from neural crest (Asim’s MCQ) autonomic ganglia 74. explain the inc in heart rate by exercise.c3 C. inc estrogen B.. 3 B. Lies in MCF on the side of cavernous sinus 96. Muscle contraction of leg 102. Lymphocytes 69. what investigation is not necessary in the donor: (Asim’s MCQ) Serum Renin UCE HLA CBC 70 yeard old lady.. Tx of hypercalcemia A...c2 B.. Bacteroids B. Hormone involved in gluconeogenisis Glucagon 71.. Widest part of epidural space A. intrathorcic pressure B.. tibialis ant & tibialis post B. Cortisol C. most abundant protein in the body 83.. Increase CUTANEOUS BLOOD FLOW B contraindicated in inc intra-ocular pressure 81.Coli Reference: GOLDMAN’s Cecil Medicine 24 thed and Harrison’s Principles & Practice of Medicine 18 92. Pneumococci 90. Neural tube is formed in which week A.A short stature n prominent crease line.above c5 D. Limb buds appear (NOT True as Upper limb buds in 4 In These options (If other 3 were wrong. Naked Person with 80% humidity in air & 21 degree temp: heat loss by (Ganong’s MCQ: Ch# 17 MCQ#2) A. Peroxisomes Long chain fatty acid what white blood cells are raised in acute viral infection A. does not operate during exercise. tibials ant & fallicus H longus Vomiting center situated Medulla oblongata Lower motor neuron lesion A. About collagen. Jejunum 77. bile acids helps fat digestion by: Facilitate absorption of Lipid Hydrolysis of LIPID 64. Short limb. A. Bisphosph C. GONADAL DYSGENESIS WITH FOLLICULAR APLASIA 100. CO poisoning B. Decrease motor tone B... baroreceptors. 4 C. conduction and radiation B. OCP 98. inc 2. IT IS? (Rabia Ali MCQ Page 484) A. Inappropriate about atropine. Calcinomimetic B. 5th week what happens? A. B. Some Scenario of UNILATERAL MUSCLE ATROPHY WITH NO SENSORY DEFICIT? like that. A. a lady have difficult in going down stairs examination shows head tilted to left and rite eye was upward ocular musle involved? A. C.?? A. phopholipase 124. serotonin C. 2nd degree Heart Block C.below c5 An immunocompromize pt developed s/s of pneumonia BAL fluid revealed boat shaped cyst organism (Nov 2013 Medicine paper MCQ = repeated) Pt unable to invert his foot while the lateral rotation is intact structure involved 105. Lymph nodes not present in? A. B. what white blood cells are raised in acute viral infection A.above c5 D. B. Antibodies Formed in SPLEEN BY Plasma Cells). Minimal change nephrotic syndrome. Aids not associated with.. Neutrophils B. Resistance of chest wall C. inf rectus. Which hormone decrease gastric motility. what white blood cells are raised in acute viral infection A. Astigmatism C) Pneumocystis carinii inappropriate about adrenal gland neonate adrenal is same the size of adult adrenal gland Artificial respiration is not necessary if lesion occur at..103. Little similar as thyroid.c3 C. What happen in lower motor neuron lesion..A. A.c2 B... Decrease in PR interval A.? A. 1:160 O antigen B. It causes ovulation. metoclopramide B. DIC occurs due to Dec in platelet count/ DIC causes decrease in platelet count 144. A.. alpha 1 glycoprotein. Patient came with complaint of hirsutism ... coagulase . Vasculitis 140.. Decrease motor tone 114.. superior opthalmic vein B. consumptional coagulopathy . Glioma B. 2nd & 3CN B.. Estrogen is released by? A. gastrin 134. C sup rectus 115. A.. 1:80 O ag D.. albumin 143. influenza 70 yeard old lady. A squamous cell carcinoma.. inferior opthalmic vein 126. pupillary light reflex carried by. spastic aortic opening aneurysm structures compressd. Asthma scenario.. B lymphoid series of bone marrow (Leukemia is Unlikely in AIDS. B. upper motor neuron B. C. ACTH C. dystonia 129. Inappropriate about hcg. right sup oblique. Immunological function is significantly reduced in A. Clostridium perfingens exotoxin action by. CCK .. 113. uterus 122.. mesengial deposition B.. B. A. ovary B. Anterior horn neurons damage B... Slow Pain transmitted by unmyelinated c fibers. major part of energy utilized during breathing is to overcome (Asim’s MCQ) A. hyprtension hyprglycemia. underlying Granulation tissue B. 1:160 H ag C. Decrease motor tone B. 1 st degree Heart Block B. it causes LEUITINIZATION OF FOLLICLES 127. Lymphoma can occur in HIV pt.. A. A child with thrombocytopenia after acute viral infection due to. Spleen B. Linear IgG deposition on IMMUNOFLORESCENCE (Confusion is in Qs stem. Area of the brain most activated before performing skilled motor activity A) Precentral gyrus B)spinal motor neuron C) cerebellum... dec basophils B. A sluggish pendular knee jerk due to. Basic drugs bind to. Neutrophils 138. Elastic recoil of lungs B. WPW syndrome 132. Stabismus C. Large airway resistance 128. Blood supply 108. B secretin. Option B is correct NOT bcoz of Lymphoma but because of BONE MARROW. Side effect of chlorpromazine.. Chorea. Which is malignant? A.110. 125. Chrons disease diff from Ulcerative colitis Perianal lesions 104. A. Dec FEV 1/FVC 75 119. Fracture behind humeral medial epicondile loss of sensation in A) medial/3palmer aspects B) Medial 1/3 palmer & dorsal aspect C) Lateral 1/3 136.. OndaNsetron 120. local edema C. hard exudates B. Hormone inhibited by feedback from hypothalamus? Prolactin 106.. D) Cortical association areas (Ganong’s MCQ: Ch: 12 Q: 5) 116. Stabismus Vomiting center situated Medulla oblongata Lower motor neuron lesion A. Lower motor neuton C.don’t rememeber exactly.. Primary inc in ACTH 131. Lymphocytes A) Toxoplasma gondii B) H.muscle wasting ..microscopic finding A. Thrombocytopenia APPROPRIATE A. mole 123. Pneumothorax Ipsilatrral lung collapse and chest wall spring out 107. Microaneurysm feature of AIDs don’t include. 1:160 O and H antigen . sub epithelial deposition C. Pt with chemotherapy induced vomiting drug to b given A.. Cataract B. in-fact CNS Lymphoma is the 2 nd most common cancer in HIV pts ) 141. Thymus 142. Cavernous sinus thrombosis A. A patient burning and recovered completely what factor determines that complete recovery will occur? . Artery in TRUE PELVIS Middle rectal artery 121.. athetosis region involved Vomiting center situated Medulla oblongata Lower motor neuron lesion A.. liver failure 130... If it was Goodpasture’s Syndrome then Option D is Correct) 135.. Lower motor neuron with ant horn of spinal cord 118. Pt with bronchial ca present with hemoptysis due to?? (Asim’s MCQ) A) Erosion of bronchial vessel B) pulm vein C) pulm artery. B hypertonia. Anti-platelet antibody (Explanation: In ITP Body own cells attack on Platelets – The reason why We give Steroids or do Splenectomy is because it is Autoimmune process and SPLEEN is the main culprit site. upper motor neuron lesion 137. 3rd and 4rd Cranial Nerves th 117. A. H. inc clotting time causing dec platelet count B.azygous vein and thoracic duct 70 yeard old lady. dec neutrophis D. thrombocytonpenia impairs APTT pathway C. C.vision not clear A..lesion? A. domeperidon C..?? (Chandkian’s MCQ with wrong key answer) A. sub endothelial deposition D.vision not clear A. Cataract B. muscle wasting.skin pigmentation and truncal obesity.Cushing disease B... substance p B. Meningioma C. A. Typhoid test (Asim’s MCQ) A. immune complex formation. Ach 133.??? A. B. Hyernatremia A. Primary Somatosensory Cortex 168. A. Rh Agglutinins A. Congenital anomaly associated with urothelial CA is A. Galactorrhea (Asim’s MCQ with little modification of question stem) A. carcinoma ovary 159.145. Serum progesterone C. GFR Clinically Measure by: A. gonadal dysgenesis with follicular aplasia 151. Initial management? (Asim’s MCQ) A. Inc muscle tone B. Pregnant lady with normocytic normochromic Anemia. Drug contraindicated in impaired renal failure: Gentamicin 182. Bradykinin .so it causes A.nasopharyngeak CA 3. Vasodilation B. Pulse 115. Atrial rate is lower than ventricular rate B. Pt having Hypertension 210/120 and Creatinine 7. Cause? plasma volume expansion 171. Lipoprotein with highest cholesterol content (Asim’s MCQ) A) HDL B) LDL C) IDL E) Chylomicrons 148. Pt with gum bleed in petechia advised bone marrow likely DX A) Lymphoma B) Leucopenia C) Leukemia 164.leukemia 156. Fainting occur because of prolong periods during which ventricles fail to Contract 175.32 C. Exotoxin produces lethal effects 179.?? A...5 ATP. Essential amino acid is. which one is NOT autosomal recessive A. A bitemporal hemiaopia.rapid ascent a) 2 3 DPG b) PO2 is normal c) PO2 is lower than normal d) Pulmonary edema 170. secondary center of ossification (Asim’s MCQ) EPIPHYSIS 169. TB 165.. cerebral cortex B. Atrial fibrillation A. Broadman’s Area 3 2 1.hemangiopericytoma 158. Stimulation of Hypothalamus Controversial Mcqs Solved By Admin Team With final Answers) Credits : Chief Admins (Umbreen Hashim And Mehtab Alam ) 1)patient on ETT. In complete heart block A. fluid having specific gravity of 1.. release of Stroncium-90 which affect milk products. defect is in? (Rabia Ali MCQ) A. TRANSDUCTION 196. Fainting occur cuz atrial unable to pump blood into ventricles D.adenoCA B. Inappropriate regarding Azygous Vein Pass through Esophageal Opening 189. CCF B. ataxia talengiectasia B.. microcytic hypochromic B.rubella may cause cardiac defect in embyo at 8 to 9 wks s0 select the option nearest to it 3)nipple and areola of multiparouse womans breast are usually at the level fifth intercostals . tractus solitarius 192.006 cause is A. excessive ADH 152. Gastrectomy after 3 months anemia A. Low molecular weight heparin A) can cause thrombocytopenia B) produced by fragmentation n degradation of heparin 178.. PAH 188. heart failure B. Gastric secretions are inhibited by what. A vertical lesion in optic chiasma causes. A.. inc intracellular Ca B. Adrenal insensitivity syndrome which is inappropriate test A. Creatinine B.. pneumonia D. Volume replacement C. Scenario of turner syndrome. Papillary 177.5 ATP & 1 FADH2 = 1. According to Modern P/O (Phosphate/Oxygen) ratio 1 NADH2 = 2. Multiple long bone fracture. a patient came in semi comatose state. Inulin C. Horse shoe kidney 147. inc intracellular Na C. 38 (8 in Glycolysis & 30 from Kreb’s Cylce) | some folks are using FA as reference that it is 32 BUT it is NOT Correct. Ventricular fibrillation is common C. Pulses deficit 195. Facial B. After giving Glucocorticoids. E) Lung. Extrophy – Conn’s Current Therapy 2013) B.. One glucose give ATP.. A. unilateral agenesis of kidney C. right sided homonomous hemianopia 153. US abd 172. Minimal change in blood flow to an organ during exercise A) Heart B) BrainC) Skeletal muscles D) Kidneys.. Tracheostomy 194. Prothrombin 154.P 90/. bladder extrophy (Adenocarcinoma has association with B.42 D. kallikren 186. Serum LH FSH B. Factor 13 C. heat loss is by evaporation . A. B. Water Intoxication causes??? (Asim’s MCQ) A) Slow pulse B) Thirst C) Altered behavior. double ureter D. agranular cortex. Pupillary dilation 187. 167. The process by which DNA is transferred from one bacterium to another by a virus is known as A... Artery involved Anterior wall MI LAD artery 191. NADH2 & FADH2 produced by 1 molecule of glucose is equal to 31 ATPs (NOT 32) So again 32 is NOT correct. Thirst is Reduced By? (Asim’s MCQ) A. Warfarin causes decrease in. present in month of Rh -ve mother when give birth to rh +ve fetus 190. Phenylalanine 155. inc intracellular K 162.. bracheceephale with mental retardation short stature with prominent simian crease B. Burning hot sensation occurs when temp of the surrounding is A.active listening 160.. 174. B. Above 45 (Thermal nocioreceptors activate when Temperature Falls Below 5 C – COLD or shoot Above 45 C) 176. good patient and docrelationship. Alpha adrenergic stimulation causes A. acclimatized. Inappropriate about dopamine. Mecula Densa 180. Atomic bomb. Fracture of Long Bones Fat Embolism 197. Maxillary C. Slow growing Tumor/Carcinoma of ThyroidA. chromophils are 50% of total 150.. megaloblastic 181. Fracture stabilization B. deep to submandibular gland is. A.. present of RBC surface B. A.. Vertical artery along face. Chromophoes are smallest B. Clostridium tetani main site of action is? A. Lingual 157.osteosarcoma B. Dreams unable to recall C.shifting dullness was positive.. A. Ca lung C... JG apparatus B. increase lipid solubility 183. B. A: contraindicated in septic shock. causes increase myocardial infarct area. Solubility of drug is increased by which factor A.28 B.. Fibrinogen B. Delta waves 185. inferior thyroid artery is branch of thyrocervical trunk 163.. there will be? (Asim’s MCQ) Decrease Lymphocytes Decrease Neutrophil 193. 2). The sum of ALL ATPs. Taste fibers through. Ach 184..liver biopsy taken and there were mallory bodies n fibrosis of liver : Alcoholic Hepatitis 166. A lady after abdominal surgery presented with dyspnea susp DVT cause (Asim’s MCQs) A) Endothelial injury B) Endothelial injury & stasis C) Stasis & hypercoagulablity.. In slow wave sleep what is unlikely. Endocervical epithelium of lady reveals sq epi cause? A) Dysplasia B) Anaplasia C) metaplasia 173. Lipolysis C. C. INC Volume of ECF C.. One question about leprosy drugs (Asked about Inappropriate statement): Answer = CLOFAZIMINE IS NOT GIVEN IN PT RESISTENT TO DAPSONE 149. secretin B. Follicular B. Tumor which also invade neural tissue A. 146. 10 microgram per kg dose of dopamine causes vasoconstriction a. ALL Major Biochemistry Texts still follow the Traditional (Old) P/O ratio (NADH2 = 3ATP & FADH2 = 2 ATP) So Correct answer is 38 161... Digoxin don't cause A. Angiotensin II B.. 10yrz 14)LP shud b done at which level.E. spreads to---------.his left eye showed internal squint indicate injury of trochlear 5)pathogenesis of venouse thrombosis in a60 year old female after major abdominal operation endothelial injury and stasis 6)tumor with longest recurrence period is malignant melanoma of uveal tract 7)tumor which s not apudomas Gastrinoma 8). structure palpable is -------. If question asks Site Select Bone Marrow) -Tearing of pelvic diaphragm durng child birth will paralyse-----..E. L3 -4 (BOTH L3/L4 or L4/L5 are correct but L3 is landmark.gastric motuility increased by Ach 12)Muscle forming inf relation wd hip joint .(If group B strep is NOT there in options then) BACTERIOIEDS -POST OP: INF-----------------------. will cause……..endoderm 19)Wht is the energy source after 48 hrs of starvation?? -Large amount of dextrose water will inhibit--------.Ureters -Broncheal cartilages derived frm----.oblique fissure t3 to t6 11).Wt Loss & Polyuria -IUCD user female-----------.Aldosterone 10).Scrotum -On vaginal examination.4)pt when asked to look left. BOTTOM Line is: Select 24-48 hours but 48-72 hours option is also correct) -Just Na excretion-------.GONORRHEA -The diagnostic lab test for the infection by streptococuss beta haemolyticus is ---------blood culture. The thing is Sperm live for 4-5 days but its capability to fertilize ovum is upto 48 hours.Granulocytes (If question specify Cell Select Granulocytes.E. .Th2 -Broncheal epithelium and glands derived from----.peritubular capillaries 20)Most Commonly injured nerve in thyroidectmy--------Extrrnal laryngeal N -Hormonal therapy in metastatic carcinoma.ANP -growth hormone is max rise in----------.obturator externus 13)Radiationz usualy cause cancer aftr B. so select the option with L3) 15)Brain abscess common in Frontal > temporal (Answer: FRONTAL) 16)synapses are absent in dorsal root ganglion 17)Sperms live in females body for 24-48 hrz (Controversy is there.CML -Philedelphia chromosome seen in cell------. Apoptosis -withdrawal reflex mediated by---------.common biological agent for carcinoma of bronchus cmv 9)which drug dont cause gynecomastia androgen (If androgen could cause Gynecomastia then every man will have it) -GENERALLY CAUSE OF PID------------.Staph aureus -Typical presentation of Diabetese-----------.ATINOMYCOSIS ISRAELI -TUBOOVARIAN ABSCESS PID-------.CHLAMYDIA > GONORRHEA -PURPURAL SEPSIS---------------------. Coli -Net tubular reabsorption of sodium Aldosterone and ANP -Mole for mole Na excretion Aldosteron -Just Na reabsorption-----.Levator ani muscle -Ca of rectum metastasize to ? Liver.mesoderm 18)which cells are involved in AIDs --------. spleen .COLI -ABDOMINAL SURGERY-------------.ADH Fat (Triglyceride) -Erythropoietin release frm----. duodenum. but Moore Parsaud says sperm live upto 48 hours.nociceptors 21)After lung abcess cause of pneumonia iz--------. Kaplan physio says 48-72 hours. LIVER -Extravaseted urine from penile rupture.Interphase -Splenic infarction--------.. 5.Frequency ( pitch) -In cell cycle chromosomes replicate in-------. kidney? ans.COLI -MOST COMMON CAUSE OF PERITONITIS (Pyogenic or Spontaneous) ---.sleep -Differnce between first & second heart sound is-------. Capillary endothelium b. Visual cortex e.Bollinger bodies = Fowl pox 6. Vitamin . What could be the probable diagnosis of this patient? a. Blood brain barrier is formed by: a. Which of the following is most strong antioxidant? Ans: a 2.Neutrophils _Enzymes release from Neutrophils ( lysosomal enzymes). Opens intestinal sphincters d. Glutathone b.Guarnieri bodies = -------in Small pox . A patient presents with recurrent epistaxis. None of above Ans: d 7. Fascial nerve e. Vitamin . Which of the following is the site of fusion of binocular vision? a. Vitamin . What is the nerve supply of tip of nose? a. All of above e. Investigation receal decreased factors II.in Rabies Ans: c . Cervical plexus Ans: a 9. Bronchodilation b. Main role is maintenance of this power as performed by: a. Vitreous Humor d.Henderson-Peterson bodies in--------. Parasympathetic stimulation caused: a. A newborn baby with hydrocephalus has a swelling in lumbosacral spinal region.C deficiency e.Negri bodies =-------. Meningoencephalocele e. Meningohydroencephalocele Ans: c 8. Prothrombin Deficiency b. 5. Meningocele c. Constipation e. Protein . Catalase a. Posterior surface of cornea Ans: a Ans: a 3. Most early diagnosis of Vitamin-A deficiency is: (a) Bilot spots in cornea (b) Night blindness (c) Keratomalacia (d) Chielosis (e) Hyperkeratosis Ans: b 10. In patient with increased bleeding time. Spina bifida b.K deficiency d. Continuous basement membrane c. Mandibular nerve d. VII. Lateral geniculate bodies c. Total power of the eye is 59 diopters. which contains neural tissue in it. Retina e. what could be the cause of bleeding a. Tachycardia .Prowazek Bodies = .C d. Anterior surface of cornea b. maxillary nerve c.E c. Retina d. Pericytes d. Von Willebrand's disease Ans: e 4. Lens c. Meningomyelocele d.Cowdry type A = in Herpes simplex virus and Varicella zoster virus and CPSP BCQs with original key DrKhalid Khan 1. Ceruloplasmin e. Optic chiasma b.Molluscum contagiosum . Optic nerve .(Remember: For Strep INFECTION Culture For Strep infection COMPLICATIONS (Post-Strep GN or Rheumatic Fever) ASO titer -Pus contains------. X & protein-C which of the following is most likely diagnosis of this condition? (a) Christmas disease (b) Hemophillia (c) Vitamin-K deficiency (d) Von willebrands disease (e) Liver dysfunction . IX. Decreased gut motiilty c. Ophthalmic nerve b. Hemophilia – A c.Trachoma--Halberstaedter . Oxygen level in the blood will decrease in: 23. Parotid gland supplied by: Option (a) GVA Option (b) GVE Option (c) SVE Option (d) SVA Option (e) GSA Ans: b 20.Ans: c 11. In inferior wall Myocardial Infraction. Primary malignant melanoma of the choroids most commonly metastasizes to: Option (a) Liver Option (b) Lungs Option (c) Brain Option (d) Kidneys Option (e) Breast Ans: a 21. What is most probable diagnosis? Option (a) Pappiloma Option (b) Basal cell carcinoma Option (c) Squamous cell carcinoma Option (d) Chalazion Option (e) Cellulites Option (a) Giant cells Option (b) Epithliod cells Option (c) Multinucleated cells Option (d) Fibroblasts Option (e) Caseation Ans: a 14. Now he is complaining of numbness at upper part of head up to vertex & medial part but medial part is intact. A patient develops sensory loss over left side of body. What it could be: (a)Golgi apparatus (b) Lysosomes (c) Ribosomes (d) Endoplasmic reticulum (e) Mitochondria Option (a) Infection Option (b) Diabetes mellitus Option (c) High mobility Option (d) Foreign body in wound Option (e) Vascular insufficiency Ans: e 18. Bitemproral vision loss is most commonly caused by: (a) Piuitary tumor (b)Crainiopharyngioma (c) Adenoma of hypothalamus (d) Section of optic tract (e) Section of optic nerve Option (a) Decreased rate Option (b) Coronary vasoconstriction Option (c) Hyperpolarization of SA node Option (d) Decreased AV nodal delay Option (e) Decreased stroke volume Ans: d Ans: a 19. A male patient has antibodies against FSH RECEPTORS. What could be the cause of delayed healing in this patient 22. On H & E staining a student sees the hallow structure around the nucleus. A middle aged male has non-healing wound in patient for long time in presence of regular dressing random blood sugar is 130 mg/dl. After few days his behavior for pain is changed & he become angry after touching. Which of the following is diagnostic of granuloma? 13. In a patient with sympathetic stimulation. Medial orbitotomey is done in a patient with tumor in orbit. Which of the following will be lower than normal in this patient? . Which of the following nerves is damaged? Option (a) Supraorbital nerve Option (b) Supratrochlear nerve Option (c) Superior orbital nerve Option (d) Inferior orbital nerve Option (e) Fascial nerve Ans: b 15. which artery should be blocked Option (a) :Left marginal artery Option (b) :Diagonal artery Option (c) :Right coronary artery Option (d) :Right marginal artery Option (e) :Posterior Interventricular artery Ans: a 16. Hostopathological report shows epithelial cells along with fibrous element. A patient has finger like projection on upper lid. what effect will occur on his heat? Ans: b 12. Where could be the lesion in brain Ans: d Option (a) Right thalamus Option (b) Internal capsule Option (c) Cerebral cortex Option (d) Basal ganglia Option (e) Midbrain Option (a) :Hypovolumic shock Option (b) :Anemic Hypoxia Option (c) :Hypoxic Hypoxia Option (d) :Congestive Heart Failure Option (e) :Acidosis Ans: a Ans: c 17. The Diagnostic finding on investigations of Metabolic Alkalosis is: Option (a) :HCO3 more then 24meq/L Option (b) :CO2 more then 24meq/L Option (c) :PH less than 7. Brucellosis is Transmitted by Ans: d Option (a) :Files Option (b) :Ras Fish Option (c) :Unpasteurized Milk Option (d) :Air Borne Option (e) :Blood Transfusion Ans: c 25. his serum Rennin level is higher than normal. which of the following changes will occur in the Nuclear Bag fiber? Option (a) :They increase impulse generation Option (b) :They remain static Option (c) :They decrease impulse generation Option (d) :They increase oscillation Option (e) :They inhibit muscle contraction Ans: a Ans: d 28. A 6 cm Lump in the breast removed.65 to -85 volts Option (b) :It automatically generates impulses.Option (a) :LDL cholesterol Option (b) :Hemoglobin Option (c) :Sperm count Option (d) :Blood Sugar Option (e) :Triglycerides Ans: c 29.4 Option (d) :Decreased O2 in blood Option (e) :Increase serum carbonic anhydrase Ans: a 30. Which of the following is the characteristic of lgM? 34. complains of Dry mouth & Dry eyes. Which investigation will give clue to her diagnosis Option (a) :ANA Option (b) :Anti-ANA Option (c) :RA factor Option (d) :Anti . Which of following drugs will be beneficial for this lady. Regarding SA Node: Option (a) :Its resting membrane potential is .SS A/B Option (e) :ANCA Ans: d 32. Biopsy declared invasive Dectal Carcinoma. A young patient's blood pressure is 150/95. A young women. This tumor is called HIGH-GRADE because: Option (a) :Lymph node involvement Option (b) :Skin involvement Option (c) :Size of tumor Option (d) :Pleomorphism Option (e) :Duct involvement 33. Option (a) :Progesterone Option (b) :Estrogen Option (c) :Prolactin Option (d) :Oxytocin Option (e) :LH Ans: d 26. Four Lymp nodes & skin were involved. Which of following structures in the mouth is derived from Ectoderm? Option (a) :Epithelium of the tongue Option (b) :Submandibular gland Option (c) :Mylohyoid muscle Option (d) :Epithelium of parotid gland Option (e) :Tonsil 24. Which of the following is the STIMULUS for this increased level of rennin? Option (a) :Essential hypertension Option (b) :Increased delivery of sodium to Renal tubules Option (c) :Increased sympathetic stimulation via Renal nerves Option (d) :Vasodilatation of Renal arterioles Option (e) :Increased blood flow to the kidneys Ans: c 31. week pregnant women has Irregular Ulterine Contractions. During the stretch in the Skeletal Muscle. creating rhythmic heartbeat Option (c) :Its membrane is impermeable to Na ions Option (d) :It is supplied by left coronary artery Option (e) :It lies in the septum Ans: b 27. which of the following structures is produced in 3rd week of development? Option (a) :It provides mucosal barrier Option (b) :It is in highest concentration in serum Option (c) :It is smallest molecule Option (d) :It is largest Molecule Option (e) :It crosses the placenta Option (a) :Thyroid gland Option (b) :Parathyroid gland Option (c) :Genital ridge Option (d) :Heart tube Option (e) Tonsil Ans: d Ans: d . The most common site of Fertilization in humans is: Option (a) :Ovary Option (b) :Uterus Option (c) :Fallopian tube Option (d) :Cervix Option (e) :Peritoneal cavity Ans: c Option (a) :PALP Option (b) :CEA Option (c) :bHCG Option (d) :AFP Option (e) :Acid Phospatase Ans: e 46.I Hypersensitivity Option (c) :Type .IV Hypersensitivity Ans: d 40. Pulmonary Artery pressure increases in: Option (a) :Exercise Option (b) :Hypoxia Option (c) :Anemia Option (d) :Hypovolumic Shock Option (e) :Hypertension Option (a) :PICA Option (b) :AICA Option (c) :Anterior Spinal Option (d) :Posterior Cerebral Option (e) :Superior Cerebral Artery Ans: c 42. Decreased sense of touch and vibrations. Least common site for ectopic pregnancy would be at: Option (a) :Ovaries Option (b) :Pouch of douglus Option (c) :Greater omentum Option (d) :Fallopian tubes Option (e) :Cervix Ans: a 44.II Hypersensitivity Option (d) :Type . Defect in the formation of Bulbus Cordis result in all of following EXCEPT? Option (a) :ASD Option (b) :VSD Option (c) :Hypertrophy of right ventricle Option (d) :Congenital cyanosis Option (e) :Transposition of great vessels 43. Most sensitive cells to hypoxia are Ans: b 38. Antibodies are produced by: Option (a) :Lymphocytes Option (b) Plasma cells Option (c) T-Cells Option (d) :Neutrophils Option (e) Endothelium Ans: b 36. One week later he develops Skin rashes. Following is not a tumor marker: Ans: d 39. tyrosine derivative does not include: Option (a) :TSH Option (b) :Adrenaline Option (c) :Nor adrnaline Option (d) :Prolactin Option (e) :Dopamine Ans: d 47. Regarding the vibration Sense all are correct EXCEPT: Option (a) :It is lost in Diabetes Option (b) :Its fibers are traveling in the dorsal columns Option (c) :Its receptor is Pacinian Corpuscle Option (d) :Its receptor is Meissener's corpuscle Option (e) :Is highly correspond to the bony portions Option (a) :RBCs Option (b) :Neurons Option (c) :Nephrons Option (d) :WBCs Option (e) :Platelets Ans: b 45. Most common site of malignancy in patients suffering from nuclear outbreak Option (a) :Haematopoietic Option (b) :Thyroid Option (c) :Lung Option (d) :Breast Option (e) :Bones Ans: a Ans: b 37. Organ having least chances of infarction: Option (a) :Lungs Option (b) :Heart Option (c) :Kidneys Option (d) :Liver .III Hypersensitivity Option (e) :Type . Patients comes with deviation of tongue to right side. A patient with aplastic anemla is given Anti Lymphocytic Globulin (ALG).41. mechanism for presentation? Option (a) :Arthus reaction Option (b) :Type . the artery commonly involved in brain is: 35. The ideal fluid replacement would be: Option (a) :Packed RBCs Option (b) :Crystallines Option (c) :Colloids Option (d) :Whole blood for 3 days Option (e) :Whole blood for 18 days Ans: b Ans: d 53. the lympahtic drainage of testicle is into Option (a) :Para aortic Lymph nodes Option (b) :Pre aortic Option (c) Superficial inguinal Option (d) :Internal iliac Option (e) :External iliac Lymph nodes Ans: a 56.Option (e) :Spleen Ans: d Option (a) :FAP Option (b) :Villous adenoma Option (c) :Tubular adenoma Option (d) :Tubulovillous Option (e) :Metaplastic polyp 48. Presence of pancreatic tissue in gastric mucosa is termed as: Option (a) :Hamartoma Option (b) :Metaplasia Option (c) :Neoplasia Option (d) :Choriostoma Option (e) :Dysplasia Ans: d 54. Man in suffering from testicular Carcinoma. Patient with injury to left 8th cervical segment of spinal cord will not show following sign: Option (a) :Decreased sense of position Option (b) :Vibration below lesion on same side Option (c) :Extensor plantar on left side Option (d) :Dec power of muscles below the lesion on same side Option (e) :Dec sense of pain and temperature below the lesion on same side Ans: e 50. Least chances of renal stones is associated with: Option (a) :Hyperlipidemia Option (b) :Hyper PTH Option (c) :Hyper vit. Most likely suffering from: Option (a) :Pagets disease Option (b) :Hyper PTH Option (c) :Hyper Vit D. Bile salts are reabsorbed from the: Ans: c 55. Soldier comes with heavy bleeding. Patient with old history of adenocarcinoma of colon operated for polypectomy. Mesothelioma is associated with: Option (a) :Vinyl chloride Option (b) :Silica Option (c) :Asbestos Option (d) :Copper dust Option (e) :Carbon Ans: c 57. In Turner syndrome. inc Alkaline phosphatase. Option (d) :Bone mets Option (e) :Osteomalacia Ans: a 51. Patient with bone pains having normal Ca. Vertebrae is derived from : Option (a) :Myotome Option (b) :Sclerotome Option (c) :Dermatome Option (d) :Ectoderm Option (e) :Endoderm Ans: b 60. on histologic evaluation pathologist labelled it as benign growth with no chances into malignant transformation. the genotype would be: Option (a) :XX Option (b) :XO Option (c) :XXY Option (d) :XY Option (e) :XYY 58.D Option (d) :Infections Option (e) :Hyperurecemia Ans: a 49. Foetal period starts after which week: Option (a) :11th Option (b) :8th Option (c) :12th Option (d) :16th Option (e) :21st Ans: b Option (a) :Duodenum Option (b) :Jejunum Option (c) :Ileum Option (d) Colon Option (e) Rectum Ans: c 52. Largest total cross-sectional and surface area is of: . it would be: 59. All true except 1. HTN. Somatostatin dec sec of which hormone Option (a) :ADH Option (b) :Insulin Option (c) :Oxytocin Option (d) :Prolactin Option (e) :Thyroid hormone Ans: c 61. Tachycardia Option (e) :Dec ICP. Example of carrier mediated counter transport: Option (a) Na . Tachycardia 73. Bradycardia Option (c) :Inc ICP. Depolarization of cell is maintained by: Option (a) :Na influx Option (b) :Ca efflux Option (c) :Ca influx Option (d) :K influx Option (e) :Na efflux 65. Digoxin toxicity Option (a) :Inc K Option (b) :Inc Mg Option (c) :Dec Ca Option (d) :Dec Na Option (e) :Alkalosis Ans: b 62. Alpha recptors effects Option (a) :Inc HR Option (b) :Lipogenesis Option (c) :Midriasis Option (d) :Piloerector contraction Option (e) :Bronchodilator Ans: c Ans: a 71. Cushing's triad: Ans: a Option (a) :Inc ICP. Hypoglycemia increases Ans: a Option (a) :SSK Option (b) :Somatostatin Option (c) :Gastrin Option (d) :VIP Option (e) :Secretin 72. Ans: a Option (a) :Retrolental fibroplasia Option (b) :Atelectasis Option (c) Dec surfactant prod Option (d) :CV depression Option (e) :Anemia . Pt complains of chest pain for more than 30 min. HTN. sq in upper 1/3 Option (c) :Has str. Esophagus histology Ans: b Option (a) :Covered by squamous ep Option (b) :Has str. Following is true regarding sterilization: Option (a) :Repeated heating denatures polyvencyl tubings Option (b) :Autoclaving is heating objects at 121 degC at 15 psi for 3min Option (c) :Radiation (UV light) Option (d) :Dry heat Option (e) :Formaldehyde 63. Hyperoxia. ECG shows changes in V1-V4. Tachycardia Option (d) :Inc ICP. hypoTN. Bradycardia Option (b) :Inc ICP. hypoTN. Structures entering thorasic inlet Option (a) :Accessory nerve Option (b) :Aorta Option (c) :Superior vena cava Option (d) :Azygous Vein Option (e) :Left recurrent laryngeal nerve Ans: b Ans: e 69. sq in lower 1/3 Option (d) :Has 3 layers of muscles Option (e) :Esophageal opening acting as anatomical sphincter 68. HTN. It denotes: Option (a) :Anterior wall MI Option (b) :Anterolateral MI Option (c) :Inferior wall MI Option (d) :Lateral wall MI Option (e) :Inferior wall MI 64.Option (a) :Artery Option (b) :Arterioles Option (c) :Capillaries Option (d) :Venules Option (e) :Veins 67. Spinal cord ends at the level of lower border of: Option (a) :L2 vertebra Option (b) :L3 vertebra Option (c) :L4 vertebra Option (d) :L5 vertebra Option (e) :S1 vertebra Ans: e 66.glucose transport Option (b) :Active transport Option (c) :Passive transport Option (d) :H transport Option (e) :Diffusion Ans: a Ans: b 70. Muscle of quiet inspiration Option (a) :Diaphragm Option (b) :Rectus abdominis Option (c) :Internal intercostals Option (d) :innermost intercostals Option (e) :External intercostals Ans: a 78. Which investigation is of first choice? Option (a) :Widal test Option (b) :Blood culture Option (c) :Typhi dot test Option (d) :Bone marrow culture Option (e) :Urine culture Ans: a 80. Thirst is decreased by: Option (a) :ADH Option (b) :Aldosterone Option (c) :Ag II Option (d) :Baroreceptor efferent Option (e) :Inc Hematocrit Ans: d 77. A young child has increased BP in upper limbs while decreased BP in lower limbs. Drug that inc extracellular K (moves K out of the cell): Option (a) :Angiotensin Option (b) H2CO3 Option (c) :Carbonic anhydrase Option (d) :pH Option (e) :Exercise Option (c) :Mucor Option (d) :Aspergillus Option (e) :Blastomycosis Ans: c 81.S. Which of the following fungi produce life threatening infection in patients with diabetics ketoacidosis? Option (a) :Candida Albicans Option (b) :Histoplasmosis Ans: b 86. non reactive HBe Ag & reactive anti HBc Ab. A Patient has enlaged parotid gland with pain in this region. What is the probable diagnosis? Option (a) :Preductal coarctation of aorta Option (b) :V. A Patient has anemia. Dry mouth increases all except Option (a) :Thrist Option (b) :ADH Option (c) :Ag II Option (d) :Plasma osmolarity Option (e) :Plasma vol 82.D Option (c) :Patent ductus arteriosus Option (d) :Postductal coarctation of aorta Option (e) :Transposition of great vessels Ans: e 79. Which nerve is carrying pain fibers from the parotid gland? Option (a) :Auricuolotemporal nerve Option (b) :Glossopharyngeal nerve Option (c) :Fascial Nerve Option (d) :Temporal nerve Option (e) :Greater auricular nurve Ans: a 83. Pacinian corpuscles related to Option (a) :Touch Option (b) :Vibration Option (c) :Pressure Option (d) :Rapidly adapting pain Option (e) :Slowly adapting pain Ans: b 75. Pulse in also week. hyper-segmented Neutrophils on peripheral blood examination & neurological manifestations. A patient has non-reactive HBs Ag. In case of typhoid fever of 06 days. Gastric acid is stimulated by: Option (a) :Gastrin Option (b) :CCK Option (c) :Secterin Option (d) :GIP Option (e) :VIP Ans: d 85. This stages of patient indication what? Option (a) :Acute infection Option (b) :Carrier Option (c) :Transient resolving stage of hepatitis Option (d) :Immunized Option (e) :Chronic active disease Ans: c Ans: a 76. which is the best antihypertensive drug Option (a) :Captopril Option (b) :Diuretic Option (c) :Varapamil Option (d) :Propranolol . Which type of anemia he is suffering from? Option (a) :Folic acid deficiency anemia Option (b) :Iron deficiency anemia Option (c) :Pernicious anemia Option (d) :Thalasemia Option (e) :Autoimmune hemolytic anemia Ans: c 84.Ans: e 74. In a patient with diabetes. Which is most likely type of diarrhea? Option (a) :Infectious Option (b) :Psychogenic Option (c) :Osmotic Option (d) :Metabolic Option (e) :Toxic Ans: c 94. Ketone-bodies. A Patient develps lesion in the Caudate nucleus.OH on left Option (d) :D sugar has no .C. E. A young female is having Goiter. Which type of hypersensitivity reaction is this? Option (a) :Type I Hypersensitivity Option (b) :Type II Hypersensitivity Option (c) :Type III Hypersensitivity Option (d) :Type IV Hypersensitivity Option (e) :A. At what value doctor is confusing & needs further evaluation with glucose tolerance test? Option (a) :04 m mol / 1 Option (b) :05 m mol / 1 Option (c) :07 m mol / 1 Option (d) :10 m mol / 1 Option (e) :15 m mol / 1 Ans: b 89.C. A diabetic Patient is advised fasting blood glucose level. A patient has Aphasia & Facial nerve palsy on same side. Best pulmonary function test for the asthma is: Option (a) :FEV .OH on right Option (b) :L sugar has . Urine Examination of a patient with diabetes demonstrates.D.OH on right Option (c) :D sugar has . A pregnant lady develops pain few hours after delivery. Which artery is most likely blocked? Option (a) :Anterior cerebral artery Option (b) :Posterior cerebral artery Option (c) :Middle cerebral artery Option (d) :Anterior division of middle cerebral artery Option (e) :Posterior division of middle cerebral artery 90. After giving blood transfusion. Which is most common clinical feature Indicating this lesion? Option (a) :Chorea Option (b) :Intentional Tremor Option (c) :Resting Tremor Option (d) :Hemiplagia Option (e) :Nystygmus Ans: a Ans: c 95. She feels difficulty in breathing while lying down. In a patient diarrhea are corrected after the fasting. What is the mechanism of formation of these ketonebodies? Option (a) :Insulin deficiency Option (b) :Hyperglycemia Option (c) :Defective fat metabolism Option (d) :Hypoalbubinemia Option (e) :Defective glucose metabolism Ans: a 88. Heme Binds with Option (a) :Albumin Option (b) :Globulin Option (c) :Hemopexin Option (d) :Heptoglobin Option (e) :Macroglobulin Ans: d 91. Councilman bodies are formed in the process of Option (a) :Infection Option (b) :Apoptosis .OH Option (e) :L sugar has no – OH Ans: a 98.Option (e) :Diltiazim Option (c) :Gallium Scan Option (d) :CPK Option (e) :Ultrasound Ans: a 87. Which type of the goiter she is having? Option (a) :Retrosternal goiter Option (b) :Malignant goiter Option (c) :Toxic goiter Option (d) :Diffuse goiter Option (e) :Simple goiter Ans: a 97. What is another investigation most appropriate for the diagnosis of this case? Option (a) :RF Option (b) :X-Ray Ans: c 96. a patient develops hypersensitivity reaction. shows S1. Q3 and T3.C Ans: c 93.G. Regarding L & D sugars Option (a) :D sugar has .1 Option (b) :Residual volume Option (c) :Tidal volume Option (d) :Vital capacity Option (e) :Inspiratory capacity Ans: a 92. Prenatal chromosomal abnormality can be detected at: Option (a) :Booking Option (b) :10-12 weeks Option (c) :14-18 weeks Option (d) :After 20 weeks Option (e) :In second trimester Ans: c 99.H Option (d) :H. A patient has history of recurrent abortion.C. Which of the following muscle group is involved? Option (a) :Gluteus Maximus + Minimus + Medius Option (b) :Only Gluteus medius Option (c) :Only Gluteus Maximus Option (d) :Gluteus Maximus + Medius Option (e) :Only Gluteus Minimus Ans: a 108.F Option (b) :Synovial fluid for polarized light Option (c) :Synovial fluid for Culture & Sensitivity Option (d) :X-Ray of joint. During general anaesthesia Halothane is given in combination with which of the following drugs? Option (a) :Thiopentone Option (b) :Nitric oxide Option (c) :Enfluran Option (d) :Phenobarbital Option (e) Ketamine Ans: b 102.R Ans: b 100. A middle aged male presents with joint pain since one week. Secondary Hyperaldosteronism can occur due to: Option (a) :Increased rennin Option (b) :Decreased rennin Option (c) :Increased potassium Option (d) :Decreased potassium Option (e) :Hypertension Ans: a 106All of the following mechanisms are involved in development of diabetic gangrene EXCEPT? Option (a) :Trophic changes due to peripheral neuritis Option (b) :Arterioscierosis leading to reduced blood flow to foot Option (c) :Decreased neutrophil motility Option (d) :Excess sugar in tissues leads to reduced resistance to infection especially fingal infection Option (e) :Anaerobic infection is commonly encountered in this disease Ans: b 107. What is the best investigation for confirmation of the disases? Option (a) :R.G Option (e) :Testosterone Ans: b Ans: b 103.H for that she received six pints of blood what is most probable caused of her menstrual problem? Option (a) :Small cell of carcinoma Option (b) :Squamous cell carcinoma Option (c) :Carcinoma of endometrium Option (d) :Clear cell carcinoma of Vagina Option (e) :Uterine fibroid Option (a) :Pituitary Tumor Option (b) :Sheehan's syndrome Option (c) :Addison's disease Option (d) :Adrenal cortex adenoma Option (e) :Psychological problem Ans: b Ans: d 110. Which of the following is most probably diagnosis? .Option (c) :Trauma Option (d) :Necrosis Option (e) :Atrophy Ans: b 104.H & F. Option (e) :E. What diagnosis you are thinking of this patient? Option (a) :Metabolic alkalosis Option (b) :Metabolic acidosis Option (c) :Respiratory acidosis Option (d) :Compensatory acidosis Option (e) :Compensatory alkalosis Ans: b 101.S. A patient comes to you in ill condition. A patient presents with dislocation of hip after an accident. Most common cause of the pelvic inflammatory disease is? Option (a) :Gonococcus Option (b) :Chlamydia Option (c) :Anaerobes Option (d) :Candida albicans Option (e) :E-coli 105.3 PCO2 is 44 mmHg & HCO3 is 17 meq/L.S. One year back she has history of P. His blood pH is 7. A patient presents with secondary amenorrhea. Her baby will be on the risk of: 109. A lady received DES during pregnancy for prevention of the abortion. His serum Uric Acid level is 156 mg/dl. Which of the following enzyme analysis will give help to diagnosis? Option (a) :Estrogen Option (b) :Progesterone Option (c) :L.P. A patient presents with recurrent Rhinitis with Urticaria & Rashes. Normal female pelvis has: Option (a) :Deeper inlet Option (b) :Inturned spine Option (c) :Narrow outlet Option (d) :Round outlet Option (e) :Oval inlet Ans: e 116.Option (d) :inferior vena cava Option (e) :inferior mesenteric artery Option (a) :Type . She is diabetic & hypertensive & one year back she was diagnosed as having carcinoma of breast.H Option (b) :Increased F. Fifth. • BPD is used 16 . • Thyroid is the only endocrine gland that stores its secretions outside the cell. trochlear nerve. • Tensor tympani is supplied by mandibular division of trigeminal nerve. • Right border of heart is made by right atrium. • Inferior orbital fissure contents are maxillary nerve and its zygomatic branch. Ninth. Tenth. He has history of taking some unknown drug. What can be most probable diagnosis for this bleeding p/v? Option (a) :Increased L.II Hypersensivity Option (c) :Viral Infection Option (d) :Arthus Reaction Option (e) :Cell mediated reaction Ans: e 117.H Option (c) :Decreased L.(Age for voting in Election is 18) • Increased alpha feto protein plus increased albumin = Anencephaly. Fifth. Which nerve supplies the posterior aspect of thigh & that can be involved in the malignancy of rectum? 118. A Patient is suffering from carcinoma of rectum. • Sensations from the tip of nose are carried by ophthalmic division of trigeminal nerve. Which of the following is characteristic of polycystic ovary disease: Ans: a 111. • Locus Ceruleus = Nor epinephrine. What is the probable diagnosis of the condition? Option (a) :Appendicitis Option (b) :Pregnancy induced hypertension Option (c) :Oligohydramnios Option (d) :Raised intracranialpressure Option (e) :Pregnancy induced DM Ans: b 119. • Common genital vesicle is herpes simplex virus. Ninth Option (c) :Second.Week pregnant lady has blood pressure 140/95. Tenth Option (e) :Fifth. nasociliary nerve and superior ophthalmic vein. • Superior orbital fissure communicates with middle cranial fossa and it transmits lacrimal nerve.(Age for Puberty in females is 13) • Trisomy 18 = Edward syndrome. Fourth. Sixth. frontal nerve. A semiconscious patient is brought to emergency department. A 60 years old women is brought with bleeding per vagina. A lady with 26 weeks gestation has uterine height of 24 weeks.H Option (e) :No change in hormones Ans: a Option (a) :Carcinoma of colon Option (b) :Carcinoma of endometrium Option (c) :Endometrial Hyperplasia Option (d) :Carcinoma of cervix Option (e) :Cervical erosion Ans: b 112.S. • Vitamin A prevents squamous cell carcinoma. • At term CRL = 36 cm and CHL = 50 cm. NaHCO2 reverses the action of drug. Fourth.14 weeks. Fourth. The ascent of horse shoe shaped kidney is prevented by: Option (a) :External iliac artery Option (b) :Internal iliac artery Option (c) :Superior mesenteric artery Ans: d ****Clear Handy Points****** • Trisomy 21 = Down syndrome. • Most diagnostic test for TB = PCR > AFB > Caseous. • CRL used b/w 7 . abducent nerve. inferior ophthalmic vein and sympathetic nerves and this fissure communicates with pterygo palatine fossa. oculomotor nerve. Tenth Option (b) :Third. • In anesthesia halothane is always given with Nitric oxide. Nucleus raphe synthesize serotonin. On sonography there is no fetus & snowstorm appearance. He develops pain in the posterior aspect of thing due to involvement of nerve.I Hypersensivity Option (b) :Type . • Stapedius is supplied by facial nerve. Eleventh Ans: b 113. (Age for Drinking is 21) • Trisomy 13 = Patau syndrome.H Option (d) :Decreased F. • Trephine biopsy Indicates = aplastic anemia > ALL. Ninth Option (d) :Third. Which of the following probably associate with it? Option (a) :Choriocarcinoma Option (b) :Karyotype-46XX Option (c) :47XX Option (d) :69XXX Option (e) :45XO Ans: b 115. She complains of vomiting & headache. Ninth.30 weeks. Seventh. .S. Which drug he has most likely taken? Option (a) :Phenobarbital Option (b) :Phenothiazine Option (c) :Morphine Option (d) :Diazepam Option (e) :Alcohal Ans: a 114. • Down syndrome occurs 1/100. Which cranial nerves carry parasympathetic fibers? Option (a) :Lumbosacral trunk Option (b) :Sacral nerves Option (c) :Obturator nerve Option (d) :Superior hypo gastric plexus Option (e) :Inferior hypo gastric plexus Option (a) :Third. A 20 . Epidermis of partoid gland is derived from ectoderm. End product of Purine is Uric acid. Bacterial Vaginosis. 2nd most common cause of osteoporosis in old age is Cushing syndrome. Dysplasia is seen in epithelia. Vulvular itching = Chlamydia. Plaque like lesion on posterior superior wall of vagina is squamous cell carcinoma. Wart on the lateral wall of introitus it is caused by HPV.coli. Least positive value for Widal is 1:120. Food poisoning caused by Staph. Trigeminal ganglion is completely covered by dura. Threonine does not contain Sulfa group. hypertrophy of Right ventricle. Referred pain: Cervix S2-S3 <> Ovary -T10-T11 <> Testis T10 <> Umbilicus -T10 <> Kidney T12-L2. MRNA has a codon. German Measles causes Congenital Cataract. Thymus and inferior parathyroid develops from 3rd branchial pouch. Epinephrine . Isoniazid > Hepatotoxicity. Platelet count and PT (except clotting time). Anterior abdominal wall swelling with umbilical cord attached to it in a new born baby is known as Omphalocele. Insulin secretion is inhibited by beta blocker. Sphincteric urethra is known as external urethral sphincter and is supplied by pudendal nerves where as internal os is supplied by inferior hypo gastric plexus. Tamoxifen. Gold & Iron toxicity. Major intracellular buffer is Hb. ESR is decreased when albumin is increased. Anti HBcAg = positive window period. Investigation of DIC D-dimers. Projectile vomiting greenish in color means bilious vomiting so it is due to duodenal atresia but if projectile vomiting non bilious then it is hypertrophic pyloric stenosis. Urea is an important indicator for muscle protein loss.most causative agent is Actinomycosis.10. Thyroid gland is derived from endoderm. Identification of turner syndrome is by barr bodies. b-HCG is increased and estriol is decreased) and if we add up inhibin which is increased then it will be called as Quadruple test. Nerve supply of rectum is hypogastric plexus. Aureus is through entertoxin. In down syndrome: Triple test (alpha feto protein is decreased. Mepacrine & Copper. Cimetidine.• • • • • • • • • • • • • • • • • • • • • • • • • • • • • • • • • • • • • • • • • • • • • • • • • • Sperm life in genital tract is 24 to 48 hours. MAP = diastolic +1/3 pulse pressure. FDPs. Protein S. Ribosome have purple color on Eosin & Methylene blue staining High energy content > Starch. H. Pyrazinamide > Gout.9. -Petechial rash. In papillary carcinoma of thyroid > it occurs in young age and involves cervical lymph nodes. Protein C. Autosomal dominant is hereditary Spherocytosis & Poly cystic kidney disease. To kill spores of surgical instrument > moist heat at 160 C for 1 hr. Spironolactone and Ketoconazole. fish like smell. Erythropoiesis in middle trimester is in the liver. Amidarone. Uterine tube is 10 cm long. Lesser omentum connects with duodenum. Drug which does not cause gynecomastia is Androgen & drugs which cause gynecomastia are Digoxin. L3 & L4. Thirst is least stimulated by blood pressure. Pulmonary trunk relation with the bronchus at the hilum of the lung-mnemonic is RALSR. M Catarrahalis. congenital cyanosis. Homan’s sign is present in DVT in which if you dorsiflex the foot there will be pain in calf muscles. Highest triglyceride = VLDL. Drugs causing corneal opacity > Amiodarone. DVT more common in popliteal vein but pulmonary embolism is through femoral veins. Right coronary artery supplies SA node. Fibrinogen and Prothrombin.Right Anterior & Left Superior. Influenza. Tubo ovarian abscess by IUCD . Trimethoprim (co-trimoxazole ) side effect > megaloblastic anemia plus leukopenia. Coli. Highest cholesterol = LDL. Low serum complement in SLE. Streptomycin causes ototoxity. Pyruvic acid is intermediate from glucose to acetyl coA. Alanine should be taken in diet. Most common organism involved in gynecological & abdominal procedures is E. -Hypoxemia. Diaphragmatic hernia occurs due to absence of pleuro peritoneal membrane. Cisplatin is more notorious to cause renal toxicity. difficile. High energy compound > ATP Antidote of warfarin is vitamin K but if action is more quickly required then FFP. End break down of glucose is pyruvate. RBCs have glycolytic enzyme activity. Classic triad of Pulmonary Embolism:   -Neurological manifestations. Urachal cyst is the remanant of allantois. Apoptosis is inhibited by bcl-2 inhibition. Levator ani muscle is supplied by L2. Rhino sinusitis is caused by Strep Pneumonia. Androgens. Rifampicin > red color of body secretions. C3b & IgG are Opsonins & C5a is chemotactic protein. Bronchial asthma plus hypertensive patient > Best drug Verapamil. Highest lipoprotein = HDL. Aphasia and facial nerve palsy > damage to middle meningeal artery. Nonfunctional nodule or cold nodule has high chances of malignant transformation. Inferior thyroid artery is also related to recurrent laryngeal nerve which runs b/w trachea and esophagus and its most common injury is in thyroidectomy. Beta-lactam acts on the Cell Wall. Superior thyroid artery is related to external laryngeal nerve which supplies posterior cricothyroid muscle of larynx and damage to it will cause hoarseness. Tx of hirstuism is Cyproterone Acetate. Olfactory cells are the only neurons in the body that regenerates.nor epinephrine & dopamine are derived from tyrosine. Chloroquine. Tracheostomy is done at 2nd tracheal ring by pulling the isthmus inferiorly. Girsoefulvin. Superior para thyroid is developed from 4th brachial pouch. Endocardial cushion is important for the formation of four chambers of heart. Sacrospinous ligament does NOT contribute in wall of perineum. Which is not a phospholipid = Plasmalogen. Diabetic plus hypertensive patient > Best drug Captopril. Best way to check bone density is the scan of spine. PLAP (Placental Alkaline Phosphatase) is a tumor marker in seminoma and ovarian carcinoma. End product of glucose gives 2 Pyruvate. transposition of great vessels (but not ASD). Tactile sensation is carried by dorsal white column to medial leminiscus. Prenatal chromosome is detected at 14 -18 weeks. In pre eclamptic patient hydralazine is the drug of choice before surgery. Pseudomembranous colitis is caused by C. 7. Important hormone involved in gluconeogenesis is Cortisol. • • • • • • • • • • • • • • • • • • • • • • • • • • • • • Defect in Bulbus Cordis results in VSD. Inferior wall MI > Right marginal artery block. Lens opacity causing drugs >Chlorpromazine. Vitamin K dependent: Factors 2. HSV is associated with vulvar papules. Intrinsic and Extrinsic pathways of coagulation converge at factor 10. . Opportunistic organism > E. Dyspnoea. Laterally to Carotid arteryd. d. Pacinian corpusclesd. 30 b. Insulin 9.• Nitrogen bubbled precipitator in ascending divers and can be treated with hyperbaric oxygen. Decreases TSH 12. 38 d. a. After trauma Female with sudden acute chest pain. 1200 c. XRay was done which showed Round mass which is calcified a. a. T-Cells d. Her blood sample is reactive blood agglutinates with anti sera B and anti sera D. Inhibits hormone release e. • Iron is stored in bone macrophages. Arch of aortac. Pulmonary embolism 17. Plasma cells c. It does not causes addiction e. a. Thoracic ductb. no other symptoms. Inhibition Iodine pump b. Umbilical arterye. 5000 21. • The most common infectious agent transmitted by blood transfusion is cytomegalovirus (CMV). O + d. • Yersinia enterocolitica. Fat embolism b. a pathogen that thrives on iron. 46 Disclaimer: questions are incomplete and MAY NOT INCLUDE CoRRECT ANSWERS 20. Growth hormonec. Medurula mycetom 24. Which of congenital malformation is most common a. VI (ophthalmic division). 2400 22. ASD b. which blood is suitable for transfusion. which is most appropriate a. Actinomyces b. 15 year old male had history of right iliac fossa pain for 18 hours and was operated for appendicitis. 1000 c. RA 8. • Before blood is transfused into newborns or patients with Tcell deficiencies. mandibular div > foramen ovale) CN VII. Air Embolism c. • Structures passing thru superior orbital fissure.. Splenic artery 4. Hyperreflexiab. SGO?PT c. 7 – 9 23. G1P2 female after delivery develops sudden chest pain and difficulty in breathing and hemoptysis. a. Medially to Carotid Arteryc. Blocks the synthesis of thyroid hormone. Last thing to be returned normal after haemorrhagic shock a. It can be given IV d. CPK 19. Peunomionia 18. Sarcoidosis d. Mecanisim of action of propylthiouracil a. Rapid adaptation receptors a. Hepatic arteryd. Endothelium 14. Antibodies are produced by a.. smear shows red granule and filaments 1-3 microns. Aspirgilosis b. PANc. Hypermeagnesemia causes a. B RH Positive c. Hydatid cyst . 14 – 18 c. LDH b. 1000 b. Free nerve endingsb. Dec acetyl choloine uptake 7. is the most common contaminant of stored blood. RBC in peripheral smear e. Internal Jugular vein is related a. VI > superior orbital fissure (Maxillary div > foramen rotundum. After forced expiration residual volume will be a. c. Lymphocytes b.. VIII > internal acoustic meatus CN IX. 2500 d. Blood pressure c. Cortisold. histopathology will show which cell a. which is present in donor lymphocytes. Gynea june 2014 1. Blocks the synthesis of thyroglobulin. Cardiac contractility d. TS 16. A female with regular cycle of 21 to 23 days what will be the ovulation days a. X. Androgene. Tuberculosis c. Ventricular Fibrilation c. Cardiac Temponade d. A+ b. 32 c. AB + 2. Meckels diskc. Hormone required for brain development a. XI(cranial root) > Jugular foramen (accessory nerve spinal root >foramen magnum) CN XII > hypoglossal canal. Azygus vein 3. X-Ray d. Pulse pressure 13. 1200 d. Anterior to Carotid arteryb. A 46 year old farmer with history of discharging sinus on foot. Heart rate b. hemoptysis and shock and then death. Posteriorly to Carotid arterye. Hanging causes death by a. a. Causes profound analgesia c. NOT-FAL NASOCILLIARY<>OPTHALIMIC VEIN<>TROCHLEAR<>FRONTAL ABDUCENT<>LACRIMAL • Cranial Nerve Foramina CN I passes through > cribriform plate CN II > optic canal CN III. Ketamine used in repeated burn dressings because a. A Female during develivery looses blood and requires blood. Neutrophils e. where blood will divert. VSD c. Following arches over left lung root. IV. Right gastric arteryb. Cornibacterium c. Its not narcotic 10. This prevents the patient from developing a graft-versus-host reaction or a CMV infection. Fracture of atlas 11. 12 – 14 b. Hashimoto thyroiditisb. Male with transmural infarction died suddenly in washroom after 8 days. Fracture of odontoid process Disclaimer: questions are incomplete and MAY NOT INCLUDE CoRRECT ANSWERS b. Lymphocytes b. Internal Carotid Artery posteriorly 5. In complete Glucose metabolism number of ATP a. Patient IVC is blocked just above the emergence of azygus vein. a. Thyroid hormoneb. 4800 e. Ruffinis corpuscles 6. ECG shows S1 Q3 T3 pattern which is most appropriate test. Atrial fibrillation b. It does not cause hypotention b. a. Hepatic veinc. 15 year gardner with history of chest pain. • CT pulmonary angiography is the best test to detect Pulmonary Embolism. Macrophages c. 500 b. Neutrophils 15. Male breathing with Tidal volume 500ml and RR of 10/m what will be the volume. it must be irradiated to kill donor lymphocytes. Best example against single organ? a. SLEd. Fentanyl 32. Thyroid b. How much pressure is required for the opening of aortic valve. Radial nerve c. Increase thickness of glomerular membrane c. H-Band d.25 PCO2 65 and HCO3 24. Choricarcinoma 30. 48. Which amino acid is essential a. Male brought to the ER with critical condition. ARF b. Type 2 heart Block 40. Biceps femoris medially b. Voluntary b. Median nerve b. Foramen cecum represents embryological structure. Ejaculatory duct opens at a. Mickels diverticulum b. Young male comes to you with the histry of problem in errection and is emberessed by his enlarged breasts. Decreased Oncotic pressure. Urea d. Na b. A delta d. Axillary nerve 41. Partially Compensated Respiratory Acidosis d. Pons b. Partially Compensated Respiratory Alcalosis e. Transverse tubules c. Folds of vocal cords 47. Ketamine b.1 what is the cause? a. 50. Submendibular gland surgery will cause damage to a. with drooling on mouth a. Female with cholelithiasis which enzyme a. Inulin b. Cystic artery is branch of a. Urine Osmolarity c. A alpha b. Cerebello-Pontine Angle 58. Basal cell carcinoma b. Phenylalinine. Disclaimer: questions are incomplete and MAY NOT INCLUDE CoRRECT ANSWERS a. Renal flow is measured by. H. Decreased Amino acid absorption b.25. Vocal cords b. Alkaline Phosphatase d. Drug that is contraindicated in Intracranial HTN a. a. A patient had resection of terminal ileum. AST b. Brocas b. Trauma to Pterion will cause bleeding of a. Respiratory and metabolic Acidosis 29. a. Fastest conduction in a. Gustatory sweating is caused by. Wirnickie c. a. Disclaimer: questions are incomplete and MAY NOT INCLUDE CoRRECT ANSWERS 55. A-Band 38. Hypoglossal nerve 57. Raspiratory alkalosis c. Diuretics d. Cl c. Nerve supply of dorsum of hand just near thumb a. Umblical Sinus/fistula? c. a. 39. thymus 49. 51. Squamous cell carcinoma c. X-Ray b. Urachal Cyst e. GGT 54. Two Z-Lines b. Lingual nerve b. HCC b. Increased GFR is caused by a. Gastroduodenal b. HIV+ patient surgery done then which antiseptic to use after surgery to clean floor a. Ca 37. Incresed Gonadotrophic hormones. Which of tumours does not metastasize Disclaimer: questions are incomplete and MAY NOT INCLUDE CoRRECT ANSWERS a. it prevents . Middle meningeal artery 53. Has branches from anterior and posterior parts 44. Melanocytic tumour??? c. 1% hypochlorite b. Patient presented with inappropriate Speech a. Plasma osmlarity b. Cornibacterium 26. 60 d. K+ is 6. Multisynaptic reflex 35. a. Acute Respiratory Acidosis b. Phonation is produced by a. Is along side cavernous sinus c. Isthamus of thyroid gland lies at a. Increased water content in stool c. Decreased arterial pressure b. 4% hypochlorite c. Regarding facial nerve 62. ADH Levek 33. 59 female Non Insulin dependent diabetes and HTN on antihypertensive brought to the emergency with semi collapsed state. How to defrentiate water derivation from SIADH??? a. Trigeminal ganglia a. ABGs showed PH 7. 2-3-4 tracheal ring 46. Regarding renal tubular absorption which of the following is passively absorbed. Patient with defect in lumen of allantois a. CRF c. Prostatic urethera 36. Right hepatic 42. CT c. Young boy with 3 week history of facial weakness and naopharyngeal sinusitis. 100 c. a. Supranuclear d. A beta c. Water 56. what will result a. Patient having progressive facial weakness for 3 months and hemiplegia lesion is in. 59. Biceps femoris laterally 31. Amoebic liver abscess test done to diagnose a. B Fibres e. Thiopental c. Heart Failure? 28. Prolactinoma b. Regarding sarcoplasm of muscle fiber is between a. AFP is raised in a. Thyroid cartilage b. Urachal Sinus 27. ALT c. Common pereonial nerve is related to a. a. Anterior portion lies in trimiginal canal b.Influenza b. Aldosterone? d. Increased CRH d. IHA 52. Left gastroepiploic c. A sharp object touches the foot patient withdraws. Teratoma d. Increased enterohepatic circulation 34. 50 b. on examination bilateral facial weakness. C Fibres 45. Para aminohippurate 60. Urachal Fistula d. Pharyngeal part of Deglutination a. Gluteraldehyde d. Heparin action on DVT. Soap and water e. 80 43. 61. Cereberum?? c. In inguinal canal 82. Nitrosamines e. Stomoch 76. Lentiform + globus pallidus b. Regarding pempaniform plexus a. a. X-Linked Dominant d. asbestosis 89. Stage IIb d. Cryoprecepitate b. Factor IX is not available what is next best choice.. decreased pepsin secretion 71. IV dose 100mg b. Flexor policis longus e.??? 70. flexor polisis longus and abductor policis longus? c.50 % undefentiated c. a. Disclaimer: questions are incomplete and MAY NOT INCLUDE CoRRECT ANSWERS 79. Thrombin time 73. Vien of heart which itself converts to coronary sinus. appropriate diagnosis a. surgery is done which artery is most likely to be damaged. Sickle cell anemia c. Estrogin containing pills are associated with a. Developed from venteral messentary 69. Emboliztion b. Immunehistochemistry of leukocytes 65. Direct c. Idiopatihc thrombocytopenic purpura lab test a. Mucur Disclaimer: questions are incomplete and MAY NOT INCLUDE CoRRECT ANSWERS 93. infection a. Which of the following disease in pregnancy will cause cataract a. synovial membrane and valla something…. 50 – 75 % undefrentiated d. Related to right colic flexure in lower pole c. a. Flexor policis brevis d. Which of the following is most malignant a. flexer sheath of thumb contains a. Boy with lymphadenopathy painless. Filum terminale 85. CT e. 1 year boy with anaemia showing microcytic cells. Symphyses pubis c. Flexor policis longus. examination showed hepatosplenomegaly. anorexia and fly bite. Sutures of skull 64. Fibrin degradating products b. Carpus Stritum a. biopsy large cells with multiple mytoses. Female with high grade fever. Measles b. anemia and hematuria a. 25% undefrentiated b. Regarding spironolactone a. mailase enlarged cervical lymphnodes biopsy reveals granulamotous lesions. Malaria 67. 90. Regarding spleen a. Is steroid 86. Internal iliac c. lymphadenopathy. Duodenum b. 88. a. Great cardiac vein b. Stage IV 68. 25 . truncal vagotomy causes a. a. a. Para aortic nodes 81. Female with fever. a. Diabetic Ketoacidosis patient managed but dies. Caudate + globus pallidus . Thelesemia d. Autosomal Recessive c. P. Femoral 83. Female with right ovarian fibrosis. Jeujenum d. More than 75 % undefrentiated 72. Spinal cord anteriorly attached to a. Tumour in right upper lobe will cause collape of a. Stage III e. Which of the following is primary cartilaginous joint. Stage IIa c. Indirect b. Histopathology grade of poorly differentiated tumour a. Schistosoma b. Giliasons capsule 84. Epiploc Appendiges are on a. Regarding kidney a. Middle cardiac vein 95. Breast cancer. Related to Rib 5 and 6 Disclaimer: questions are incomplete and MAY NOT INCLUDE CoRRECT ANSWERS b. Tuberculosis c. flexor pollicis longus b. Whole Blood d. Kala-azar b. Hemophelia b. X-Linked Recessive 91. Right upper apical b. Lymphoma with cervical enlargement and splenic involvement stage. Organization c.a. PCV 75. Kidneay and surrounding fat coverd by gerrota fascia b. PT c. Person from sibbi Baluchistan with history of fever. decreased acid secretion c. Bronchogenic carcinoma a. Leukoplakia b. Uterine artery 80. a. Smoking b. Patient developed SCC of bladder 8 years back had granulomatous appearance of bladder biopsy. Ileum c. Rubella during pregnancy c. Bladder stone d. Female after delivery has developed sepsis and DIC which test is NOT appropriate a. Factor VIII is not available what is next best choice. 78. 87. Lymphocytosis on peripheral semear b. Platelets c. Costochondral b. AFB demonstration 66. BT d. Alar ligament b.falciparum 96. Propagation 63. Stage I b. Right lower c. Testicular tumour will first drain to a. Whole Blood 74. Left circumflex artery supplies 92. Bleeding time 77. Two brothers having hemophelia and but sister does not have what is inheritance a. Hernia medial to inferior epigastric artery. Internal iliac nodes c. Erythroplakia c. decreased motility b. External iliac b. Platelets d. Cryoprecipitate b. b. a. Sigmoid colon e. FFP c. Chest X-Ray b. 94. Autosomal Dominant b. aPTT b. Exernal iliac nodes b. appropriate diagnosis by a. failure therive. elder brother regular blood transfusion a. Anaphase d. A+ b. Palatine tonsils b. type of polys.2 MCV 68 a. Male presents with dull right sided chest what is the cause a. Taking any Oral prednisolone 116. Thymus 111. Laterally to Carotid arteryd. a. a.Botulinum food poisoning. Globulin. X-Ray shows pneumoperitoneum. Biopsy of tissue shows Lymphoid tissue with stratified squamous epithelium a. Rapid adaptation receptors a. A Female during develivery looses blood and requires blood.c. Myoglobin 99. before surgery antibiotic should be given a. b. IUCD c. and weakness labs show hb 6. Hyperreflexiab. Endoplasmic reticulum 112. Internal Carotid Artery posteriorly 5. Post tibial artery c. Patient IVC is blocked just above the emergence of azygus vein. Ceftrizime? b. Most common intracellar buffer a. Regarding spinal nerves a. What causes osmotic diuresis a. Umbilical arterye. 97. Following arches over left lung root. her cortisol level is <1 what is most important question to ask a. Patient with Tobacco chewing showing ulcer on iner lip margin which is indurated and pale is a. Hepatic veinc. Insulin 9. HPV 115. Androgene. Causes profound analgesia c. Medial longitudinal arch piller a. a. to parrelel??? a. Meckels diskc. Basic drug binds to a. HCO3 b. Sartorius b. Metaphase b. 15 cm 118. Manitol 100. Cl 98. Morphine and naloxone 101. O + d. It can be given IV d. Right gastric arteryb. Cervical cancer increased risk in a. Anterior to Carotid arteryb. Hyperplastic c. Along intercostal vessels to posterior thoracic node. Hypermeagnesemia causes a. Patient with C. Cytoplasm d. Flexor of knee a. characteristic feature will be a. A1glycoprotein d. Chronic irritation b. Teres minor d. Hemoglobin c. PANc. Navicular 124. Telophase e. Subscapularis e. Regarding trachea a. Pectoralis major b. Painful defecation nerve supply a. Ruffinis corpuscles 6. Best example against single organ? a. Flexor halluces longus d. Which of the following is blood supply of head of femur a. Question regarding apoptosis 105. Tibialis anterior gynea august 2014 1. It does not cause hypotention b. Erythropoietin 117. Tibial nerve b. Nucleus b. Pacinian corpusclesd. Medially to Carotid Arteryc. Pudenal nerve 119. Ribosomes c. Supraspinatus 122. Her blood sample is reactive blood agglutinates with anti sera B and anti sera D. Posteriorly to Carotid arterye. Tallus b. c. Adductor magnus d. Male died after adenocarcinoma. a. Major Breast lymphnodes. Adenomatous b. Pharmocodynamica Drug-Drug interaction a. Free nerve endingsb. Nutrient 108. Following muscle does not attach to humerous a. Pectoralis minor c. Amagdaloid + globus e. Thyroid hormoneb. Ketamine used in repeated burn dressings because a. Hepatic arteryd. Calcaneum c. b. Dead Neutrophils 102. Azygus vein 3. Lateral cirumflex humural c. Lymphnodes c. Vomitingc. Albumin b. Patient with crf on dialysis with anemia what should be given a. Prophase c. C-shaped rings with cavity anteriorly b. Hashimoto thyroiditisb. Hormone required for brain development a. Thiazide and lithium b. Prometaphase 104. which blood is suitable for transfusion. Melanoma 123. Protein 103. Iron deficiency anemia 114. Having Headache b. where blood will divert. a. Hemartomous d. Respiratory muscle depression 121. Flexor halluses bravis e. Boy with high grade fever and tender abdomen. Splenic artery 4. Pneumonia Disclaimer: questions are incomplete and MAY NOT INCLUDE CoRRECT ANSWERS 106. Thoracic ductb. Structures posterior to medial malleolus except. Basal cell carcinoma c. Na e. RA 8. Pleural effusion. Internal Jugular vein is related a. autopsy showed thousands of polyps in colon. Oxygen storage in muscle a. Along internal thoracic to internal thoracic nodes??? 110. Medial circumflex humural b. AB + 2. Chromosome lines in center in a. Diarrhoeab. Gastrocnemius c. Obturator d. c. Arch of aortac. B RH Positive c. Pyers patches d. Metaplastic 107. Derived from neural crest only? 113. Growth hormonec. Caudate + lentiform d. Biceps femoris 109. Female having complains of dysphagia. Female with cushing disease came for investigations. Gentamycin 120. SLEd. Basophilia is caused by a. It does not causes addiction . Pus of follicle contains a. Dec acetyl choloine uptake 7. Cortisold. PO4 d. Exit from intervertebral foramen b. Dead bacteria b. Squamous cell carcinoma b. Along Axillary vessels apical nodes. 1200 d. T-Cells d. Sarcoidosis d. Antibodies are produced by a. a. Two Z-Lines b. XRay was done which showed Round mass which is calcified a. 2400 22. 46 Disclaimer: questions are incomplete and MAY NOT INCLUDE CoRRECT ANSWERS 20. Ejaculatory duct opens at a. 5000 21. a. Heart rate b. A patient had resection of terminal ileum. A 46 year old farmer with history of discharging sinus on foot. Squamous cell carcinoma c. a. ECG shows S1 Q3 T3 pattern which is most appropriate test. Fracture of odontoid process Disclaimer: questions are incomplete and MAY NOT INCLUDE CoRRECT ANSWERS b. d. Prostatic urethera 36. X-Ray d. Increased enterohepatic circulation 34.25 PCO2 65 and HCO3 24. Regarding renal tubular absorption which of the following is passively absorbed. a. A-Band 38. 1000 c. After trauma Female with sudden acute chest pain. Air Embolism c. histopathology will show which cell a. 7 – 9 23. 500 b. Urea d. Hanging causes death by a. what will result a. Increased water content in stool c. Fracture of atlas 11. 12 – 14 b. Heart Failure? 28. Peunomionia 18. ADH Levek 33. Medurula mycetom 24. Its not narcotic 10. ARF b. Na b. Lymphocytes b. Decreased Amino acid absorption b. Partially Compensated Respiratory Acidosis d. H-Band d. a. Regarding sarcoplasm of muscle fiber is between a. 30 b. A female with regular cycle of 21 to 23 days what will be the ovulation days a. K+ is 6.1 what is the cause? a. Teratoma d. no other symptoms. Umblical Sinus/fistula? c. Young boy with 3 week history of facial weakness and naopharyngeal sinusitis. Decreases TSH 12. Plasma osmlarity b. Endothelium 14. 4800 e. Dyspnoea.e. Blocks the synthesis of thyroglobulin. c. Urachal Fistula d. a. Thiopental c. with drooling on mouth a. Last thing to be returned normal after haemorrhagic shock a. Ca 37. Aspirgilosis b. Acute Respiratory Acidosis b. CPK 19. Ventricular Fibrilation c. Tuberculosis c. SGO?PT c. TS 16. After forced expiration residual volume will be a. VSD c. Choricarcinoma 30. Raspiratory alkalosis c. Biceps femoris laterally 31. Fentanyl 32. Fat embolism b. Which of tumours does not metastasize Disclaimer: questions are incomplete and MAY NOT INCLUDE CoRRECT ANSWERS a. Transverse tubules c. Neutrophils 15. Cornibacterium c. Male with transmural infarction died suddenly in washroom after 8 days. Neutrophils e. Inhibition Iodine pump b. Mickels diverticulum b. H. Male brought to the ER with critical condition. Urachal Cyst e. Young male comes to you with the histry of problem in errection and is emberessed by his enlarged breasts. Male breathing with Tidal volume 500ml and RR of 10/m what will be the volume. Multisynaptic reflex 35. Basal cell carcinoma b. Cl c. 59 female Non Insulin dependent diabetes and HTN on antihypertensive brought to the emergency with semi collapsed state. Drug that is contraindicated in Intracranial HTN a. Cardiac contractility d. hemoptysis and shock and then death. Respiratory and metabolic Acidosis 29. Atrial fibrillation b. . on examination bilateral facial weakness. RBC in peripheral smear e. ASD b. Common pereonial nerve is related to a. How to defrentiate water derivation from SIADH??? a. Diuretics d. 2500 d. Pulmonary embolism 17. Pulse pressure 13. 15 year gardner with history of chest pain. LDH b. 1000 b. 32 c. CRF c.Influenza b. Patient with defect in lumen of allantois a. Blood pressure c. A sharp object touches the foot patient withdraws. a. Inhibits hormone release e. ABGs showed PH 7. Hydatid cyst 25. Macrophages c. Mecanisim of action of propylthiouracil a. 15 year old male had history of right iliac fossa pain for 18 hours and was operated for appendicitis. Actinomyces b. 1200 c. Urine Osmolarity c. Urachal Sinus 27. 14 – 18 c. G1P2 female after delivery develops sudden chest pain and difficulty in breathing and hemoptysis. Which of congenital malformation is most common a. which is most appropriate a. Biceps femoris medially b. Partially Compensated Respiratory Alcalosis e. Cardiac Temponade d. 38 d. Ketamine b. Lymphocytes b. smear shows red granule and filaments 1-3 microns. Blocks the synthesis of thyroid hormone. In complete Glucose metabolism number of ATP a. Cornibacterium 26. Plasma cells c. A alpha b. 100 c. Decreased Oncotic pressure. Immunehistochemistry of leukocytes 65. Renal flow is measured by. 50 – 75 % undefrentiated d. Radial nerve c.. ALT c. a. Increased CRH d. IHA 52. Thyroid b. AFB demonstration 66. A beta c. flexor polisis longus and abductor policis longus? c. examination showed hepatosplenomegaly. Stage IV 68. 25% undefrentiated b. HCC b. Thrombin time 73. a.??? 70. Left gastroepiploic c. Kala-azar b. appropriate diagnosis a. Which amino acid is essential a. Folds of vocal cords 47. HIV+ patient surgery done then which antiseptic to use after surgery to clean floor a. Developed from venteral messentary 69. Decreased arterial pressure b. decreased motility b. Stage IIb d. Thyroid cartilage b. a. Type 2 heart Block 40. Organization c. Boy with lymphadenopathy painless. Voluntary b. flexor pollicis longus b. a. Phonation is produced by a. Nerve supply of dorsum of hand just near thumb a. Heparin action on DVT. Flexor policis brevis d. Prolactinoma b. aPTT b. Patient having progressive facial weakness for 3 months and hemiplegia lesion is in. Is along side cavernous sinus c. Sutures of skull 64. Wirnickie c. Lingual nerve b. C Fibres 45. Submendibular gland surgery will cause damage to a. Cerebello-Pontine Angle 58. GGT 54. Disclaimer: questions are incomplete and MAY NOT INCLUDE CoRRECT ANSWERS 55. synovial membrane and valla something…. PT c. Disclaimer: questions are incomplete and MAY NOT INCLUDE CoRRECT ANSWERS a. a. Soap and water e. a. 48. Symphyses pubis c. Trauma to Pterion will cause bleeding of a. A delta d. Lymphoma with cervical enlargement and splenic involvement stage. Patient presented with inappropriate Speech a. FFP c. Inulin b. Female with fever. Cryoprecipitate . biopsy large cells with multiple mytoses. Platelets d. Anterior portion lies in trimiginal canal b. Pons b. Phenylalinine. decreased acid secretion c. flexer sheath of thumb contains a. Lymphocytosis on peripheral semear b. Related to right colic flexure in lower pole c. mailase enlarged cervical lymphnodes biopsy reveals granulamotous lesions. Isthamus of thyroid gland lies at a. Gluteraldehyde d.a. How much pressure is required for the opening of aortic valve. Water 56. Flexor policis longus e. Regarding spleen a. Amoebic liver abscess test done to diagnose a. AST b. Alkaline Phosphatase d. 2-3-4 tracheal ring 46. Factor IX is not available what is next best choice. Brocas b. Female with cholelithiasis which enzyme a. X-Ray b. B Fibres e. Person from sibbi Baluchistan with history of fever. Which of the following is primary cartilaginous joint. truncal vagotomy causes a. Middle meningeal artery 53. Increase thickness of glomerular membrane c. 1% hypochlorite b. Right hepatic 42. Median nerve b. Factor VIII is not available what is next best choice. CT c. a. Emboliztion b. CT e. Axillary nerve 41. Vocal cords b. decreased pepsin secretion 71. Whole Blood 74. Stage I b. Cystic artery is branch of a. Increased GFR is caused by a. Aldosterone? d. Flexor policis longus. Hypoglossal nerve 57. Trigeminal ganglia a. Malaria 67. Melanocytic tumour??? c. BT d. 39. 50. 80 43. Para aminohippurate 60. 4% hypochlorite c. Regarding facial nerve 62. 59. Related to Rib 5 and 6 Disclaimer: questions are incomplete and MAY NOT INCLUDE CoRRECT ANSWERS b. Pharyngeal part of Deglutination a. Cereberum?? c. More than 75 % undefrentiated 72. 61. AFP is raised in a. 51. Fastest conduction in a. Idiopatihc thrombocytopenic purpura lab test a. Chest X-Ray b. a. Cryoprecepitate b. 60 d. it prevents a. Costochondral b.50 % undefentiated c. Gastroduodenal b. thymus 49. Histopathology grade of poorly differentiated tumour a. lymphadenopathy. Foramen cecum represents embryological structure. Has branches from anterior and posterior parts 44. 25 . Incresed Gonadotrophic hormones. anorexia and fly bite. Stage IIa c. Propagation 63. 50 b. Gustatory sweating is caused by. appropriate diagnosis by a. Supranuclear d. Stage III e. Prophase c. Carpus Stritum a. a. Na e. Chromosome lines in center in a. Adductor magnus d. Para aortic nodes 81. Hemoglobin c. Lymphnodes c. Alar ligament b. Tumour in right upper lobe will cause collape of a. Femoral 83. Gastrocnemius c. Regarding kidney a. Prometaphase 104. Patient developed SCC of bladder 8 years back had granulomatous appearance of bladder biopsy. a. Telophase e. Caudate + lentiform d. Right lower c. Pharmocodynamica Drug-Drug interaction a. Middle cardiac vein 95. In inguinal canal 82. Duodenum b. Vien of heart which itself converts to coronary sinus. Biopsy of tissue shows Lymphoid tissue with stratified squamous epithelium a. PCV 75. 97. Basophilia is caused by . Leukoplakia b. Erythroplakia c. Lentiform + globus pallidus b. infection a. Question regarding apoptosis 105. a. Dead bacteria b. Bronchogenic carcinoma a. 90. Oxygen storage in muscle a. Breast cancer. Which of the following is blood supply of head of femur a. A1glycoprotein d. Two brothers having hemophelia and but sister does not have what is inheritance a. Great cardiac vein b. Flexor of knee a. Sickle cell anemia c. Autosomal Dominant b. Stomoch 76. Kidneay and surrounding fat coverd by gerrota fascia b. Hernia medial to inferior epigastric artery. Bleeding time 77. anemia and hematuria a. Fibrin degradating products b. Ileum c. P. Rubella during pregnancy c. Myoglobin 99. Male presents with dull right sided chest what is the cause a. 87. Male died after adenocarcinoma. IV dose 100mg b. PO4 d. Thelesemia d. c. Female with high grade fever. Pleural effusion. Pneumonia Disclaimer: questions are incomplete and MAY NOT INCLUDE CoRRECT ANSWERS 106. Epiploc Appendiges are on a. Basic drug binds to a. Jeujenum d. Smoking b. Left circumflex artery supplies 92. Obturator d. Mucur Disclaimer: questions are incomplete and MAY NOT INCLUDE CoRRECT ANSWERS 93. Right upper apical b. Metaphase b. Along Axillary vessels apical nodes. type of polys. Sigmoid colon e. Along internal thoracic to internal thoracic nodes??? 110. Morphine and naloxone 101. Sartorius b. Globulin. Pus of follicle contains a. Exernal iliac nodes b. Adenomatous b. X-Linked Recessive 91. HCO3 b. Manitol 100. c. X-Linked Dominant d.b. Which of the following disease in pregnancy will cause cataract a. surgery is done which artery is most likely to be damaged. Tuberculosis c. Major Breast lymphnodes. Thymus 111. Medial circumflex humural b. Bladder stone d. Along intercostal vessels to posterior thoracic node. Schistosoma b. Regarding spironolactone a. Female after delivery has developed sepsis and DIC which test is NOT appropriate a. Measles b. autopsy showed thousands of polyps in colon. Hyperplastic c. Internal iliac nodes c. Anaphase d. Estrogin containing pills are associated with a. asbestosis 89. failure therive. 94. b. Internal iliac c. Direct c. Metaplastic 107. Hemophelia b. Hemartomous d. Nutrient 108. 78.falciparum 96. Caudate + globus pallidus c. Most common intracellar buffer a. Pyers patches d. Testicular tumour will first drain to a. b. Biceps femoris 109. Uterine artery 80. Platelets c. a. Regarding pempaniform plexus a. Which of the following is most malignant a. Cl 98. Female with right ovarian fibrosis. b. Amagdaloid + globus e. 88. Filum terminale 85. Palatine tonsils b. Giliasons capsule 84. Lateral cirumflex humural c. Thiazide and lithium b. 1 year boy with anaemia showing microcytic cells. Protein 103. Is steroid 86. External iliac b. Albumin b. What causes osmotic diuresis a. Dead Neutrophils 102. Autosomal Recessive c. Diabetic Ketoacidosis patient managed but dies. Nitrosamines e. a. Indirect b. to parrelel??? a. Whole Blood d. Disclaimer: questions are incomplete and MAY NOT INCLUDE CoRRECT ANSWERS 79. Spinal cord anteriorly attached to a. elder brother regular blood transfusion a. Pectoralis major b. Depressor anguli oris2.C6B. PancreasD. Hep B 13. Melanoma 123. External Carotid ArteryC. Internal Carotid Artery 17. Structures posterior to medial malleolus except. 21. Subscapularis e. Formed only 30percent by chorioid plexus4. Gentamycin 120. Ribosomes c. Axillary Nerve 5. Having Headache b. Peroneus BrevisC. Formed 500ml/day 3. CystE. CB. Loflar bodiesb.C8 All other options were wrong but didnt know what to choose among above two. Pain from Gallbladder felt in Anterior Triangle of neck. Basal cell carcinoma c. Bone marrow depression 12. Granulomas D. Age estimation at fifth to 12th week is done through: A. Vomiting c. 9. Rhinophycomycosis4. Vagus Nerve has wide distribution in thorax and most of abdominal organsB. Flexor halluces longus d. Complex hyperplasia without atypia3. Cryptococcosis2. Crown rump length C. Cholesterol StonesB. Chronic irritation b. Peronues LongusB. CholangioCAD. Spotted calcificationsB. Following muscle does not attach to humerous a. S1B. Diarrhoea b. Ceftrizime? b. Patient with Tobacco chewing showing ulcer on iner lip margin which is indurated and pale is a. Patient with crf on dialysis with anemia what should be given a. Erythropoietin 117.. MELAS syndrome histological finding: A. before surgery antibiotic should be given a. Inclusion bodiesC.A sprinter during running had injury to his ankle. Medial longitudinal arch piller a. Flexor halluses bravis e. Crown heel length D. Pectoral NodesD. Biparietal diameter B. Part of Endothorac FasciaC. Tallus b. Infectious mononucleosis 5. Calcaneum c. Simple hyperplasia2. Soap Bubble appearance 11. Erythroblastosis fetalisC. HEP B. Formed by ependymal cells2. S3c. Covers Pectoralis MajorD. C-shaped rings with cavity anteriorly b. X-Ray shows pneumoperitoneum. Endoplasmic reticulum 112. ApicalB. Tibialis anterior 1ST JAN 2015 SURGERY AND ALLIED: 1. Tibial nerve b. Teres Minor 3. Exit from intervertebral foramen b. Most common cause of Hepatocellular CA in developing countries: A. most fluid loss will be through: A. her cortisol level is <1 what is most important question to ask a.2 MCV 68 a. Patient with fluid loss now show metabolic acidosis e hypokalemia. Female with cushing disease came for investigations. Parasympathetic something 7. Orbicularis oris ans5. Female having complains of dysphagia. IUCD c. a. Tibialis Posterior 6. Patient with C. Folate deficiencyD. Mumps 18. Aspergillus 19. Jujenum 4. Depressor labii inferioris3. Lateral pterygoid 16. Chicken pox4. Giant Cell Tumors histo finding: A. Tricep. Young adult with raised ALP. Mucormycosis3. Pigment StonesC. Cervical cancer increased risk in a. Histoplasmosis5. Most appropriate about Clavipectroral Fascia: A. L4 8. Teres minor d. Which of following has highest chances of developing endometrial cancer 1. Abdominal circumference 22.Pectoralis MinorC. Biceps. 15 cm 118. Chronic endometritis 20. Regarding CSF 1. Red ragged fibers (some says inclusion and some say LOFLAR) 10. Nucleus b. HPV 115. Hepatitis 14. and weakness labs show hb 6. Pectoralis minor c. Inversion of foot by which muscle: A. Supraspinatus 122. Depressor labi superioris4. Cytoplasm d. Incubation period of which disease is longest: 1. Inferior Phrenic Nodes 15. Squamous cell carcinoma b. Boy with high grade fever and tender abdomen. Respiratory muscle depression 121. Pudenal nerve 119. StomachB. Squamous metaplasia 5. characteristic feature will be a. Derived from neural crest only? 113.Covers Pectoralis Minor 2. Teres MajorD. jaundice and decreased Hb Scenario A. Navicular 124. Regarding Lymph drainage of breast except: A. Closure of lips involve: or unable to close lips which muscle involved: 1. There is a visible ecchymoses around ankle joint: . Post tibial artery c. Regarding trachea a. He can stand on his toes but with severe pain. Complex hyperplasia with atypia4. Superior Thyroid artery arise from: A. Pectoralis MajorB. Above the clavicle connects wid Pretracheal FaciaB. Iron deficiency anemiasB. Peroneus tertiusD. this mechanism of Reffered Pain appropraite statement is: A. First branch of Subclavian ArteryB. Hep Cc. DuodenumE.Botulinum food poisoning. If Lateral Cord of Brachial Plexus will damage which muscle will get paralysed most likely: A. Regarding spinal nerves a. Measles2. Painful defecation nerve supply Disclaimer: questions are incomplete and MAY NOT INCLUDE CoRRECT ANSWERS a. Fracture of surgical neck of Humerus damage to: A. Chronic Fungal infection with rhinosinusitis causing medial erosion and granulomas: 1. Taking any Oral prednisolone 116.a. Accersory nerve gives fibers to Gallbladder…C. Blast Cells in peripheral blood: A. Supraclavicular NodesC. Iron deficiency anemia 114. Rubella3. Arachnoid villi are visible There were two questions about CSF. Regarding jerks corresponding with spinal segments: A. Lateral foot dermatome: A. Colon C. Optic tract fibersC. with bleeding from i guess Femoral Artery something. Simple squamous3. Albumin and something (Option and stem were imp in this question which no one could recall properly so cant really confirm the answer now and offcourse we most of us didnt know if whatever they marked was right) 57. MineralsB. ulna 29. Profofol increases cerebral blood flow3. Sigmoid and rectum 25. RetinaD. Newborn with a Erythroblastosis Fetalis having Bloodgp Bpos what is best for management: Any blood with Rhpostive? cant rem options and correct ones 54. and due to hypovolemia which will be LEAST affected: A. Brainc. Supination occurs here5. Hinge joint4. Terminal ileumB. A 30 year old woman in azad kashmir was injured mildly in an earthquake that killed many people apart from her distant uncle. VIIIC. a fibers B. 20%C. A 31 year old young man has heaviness in his scrotum for six months. A patient came in ER. Stratified squamous keratinized2. Adh secretion 55. CraniopharyngiomaD. Developmental crisis 41. Craniopharyngioma 58. Ilioinguinal nerveC. Project to Visual Cortex 48. Myxomas2. Regarding ADH secretion: A. Buccopharyngeal Membrane: A. Middle constrictor 31. Skin integrity 43. On labs his alpha feto protein level was 81ng/ml while bHCG level was 15 IU/l. Simple columnar4. Longest spineB. Carcinoma expleomorphic adenoma 39. Function exclusively linked with vit E: 1. Transverse colonE. 30%B. Anterior hypothalamus B. For assessment of nutritional status of a person alongwith weight loss and something u will assess: A. Solely to thalamUSE. Glioglioma C. Covered all around by muscles3. Pineal tumorC. Intestines 47. While a surgery on submandibular gland which has more chances of injury: 1. Uterosacral ligament5. Femoral nerve 30. C7 Vertebrae has: A. A man during fasting his urine become concentrated because of: A. Inferior constrictorC. Thiopental reduces heart rate2. Choriocarcinoma4 Teratoma 45. On examination he is having genitals hypoplasia: A. Morphine reduces apnoeic threshold4. Parts from all constrictorsD. Medial to epigastric Artery 53. Which ligament supports odontoid process of axis on atlas( stem not sure) A. Thiamine 59. Regarding elbow joint: 1. Ascending colonD. Plantaris tendon 23. Percentage of Plasma out of ECF: A. Obturator nerveB. Cerebral Malaria caused by: A. Adenomas3. Glioblastoma multiforme B. Hyperglycemia with HepatomegalyB. Same side orchiectomy was done and Which of following could be the cause 1. 50% 52. Pretracehal fascia infections spreads to: A. Regarding Pterygopalatine Raphe: A. Regarding Thalesemia A. Most radiosenstive tumour: A. If a person is taking Protein in his diet which will be necessary: A. Hyperglycemia with splemomegalyC. inhibits synthesis of thyroid hormonesB. Posterior hypothalamus 38. Plasmodium FalciparumB. Receives oculomotor fibersB. Sciatic nerveD.Otic ganglion is between mandibular nerve and tensor tympani. RiboflavinB. Most important characteristic of Glycogen Storage diseases are: A. a delta fibersD. Carbohydrate metabolism2. c fibers 37. Shivering center is located in: A. Sensory supply of uterus passes through which ligament: 1. Hypoglycemia with HepatosplenomegalyD. Highest number of alpha resceptors are present in: 1. Anterior mediastinumB. Facial nerve 42. Sensations from tip of fingers by: A. Foramen rotundumD. Endothelial protection3. inhibits release of thyroid hormones 34. Penile urethra4. Stomodium from ForegutB. PTU: A. Styloid process 24. Cushing disease 26. Lingual nerve3 Mandibular nerve4. Foramen lacerumB. Neck of bladder and proximal urethra3. Joint between humerus. Which tumor involves extension to nerve sheath: 1. Hemangiopericytoma4. Tranverse ligamentD. Uterocervical ligament 27. radius. Which one combination is correct: 1. BuccinatorB. PT Is indicated by: A. Broad body 50. She would most probably be in 1. Situational crisis4. Direct Inguinal Hernia: A. Cardinal ligament4. Posterior longitudinal ligamentB. Hypoglycemia with Hepatomegaly 32. VitaminC. Foramen ovale C. Defect in globin production 56. External urethral opening 28. Leydig cell tumor2. a beta fibers C. Prothrombin (or Thrombin) was an option (and 7 wasnt in da options) 51. Posterior mediastinum 36. Released in increased Plasma osmolarity . Capsule deficient posteriorly2. HypothyroidismB. Superior thyroid artery2. Amoeba causes lesions in which part of gut: A. Defect in ratio of chains B. Fibers ascend to thalamocortical areas D.A. Medial geniculate body A. Nose from Pharynx 33. Skeletal MusclesE. Membranous urethr5. On examination a solid 5cm mass is palpated. Pelvic 35. RetrocecalB. Trigone2. A young boy with inc appetite weight gain and sleep problems. Anterior longitudinal ligamentC. Embryonal carcinoma3. MyocardiumB. A needle when inserted 2cm below and lateral to pubic tubercle will be closely related to on: A. Most common position of appendix: A. Social crisis2. NarcolepsyE. Stratified Squamous non keratinized 40. Broad ligament2. Anterior epithelium of cornea is: 1. It's superior border is formed by: A. Decrease intake of water B. XIIIB. Ligamentum nuchae 46. IXD. Ketamine causes bronchodilation 44. CecumC. Personal crisis3. Plasmodium Malariae 49. Round ligament3. On labs WBC 17000cells/mm3. synthesis of t lymphocytes within lobule 96. Collecting DuctB. SA Node generates spontaneous impulses. Isovoulmetric Contraction 84. Nucleus ambiguous D. Cranial nerve 9 80. Lingual artery C. is an increase in heart rate due to an increase in central venous pressure. SretptococcusD. Recurrent branch of median2. Fires dynamic respiratory drive 89. Surgeons injects dye during surgery through: A. Following is true: A. Appendicular Artery is a branch of: A. Appendicectomy was preformed. Dysphagia due to liquids only: A. Internally to occipital bone 61. Increases inspiratory potentialB. Maxillary artery D.Function exclusively linked with vit E: 62. Myxomas most common location: A. Isthmus attached to cricoidD. Ligamentum Teres A. On labs his PT and aPTT were normal. Hemangiopericytoma 4. Facial artery B.Loss of abduction and adduction of fingers and adduction of thumb with intact skin sensations involves: 1. Germinal epithelial protection (seniors confirmed) 82.Inion is: 97. Surrounded by prevertebral fascia C. Maintains posture and balance (The Bainbridge reflex. Naproxen 70. A- . Schistosoma hematobium 95.Heterophile antibody is seen in: A. O-D. Gonorrhea 83. Carcinoma expleomorphic adenoma 86. Endothelial protection C. Mast cell destruction B. AnemiaB. Locus ceruleusB. Polymorphonuclear neutrophills 3. Protonopia 92. Bladder carcinoma: A. Drug induced thrombocytopenia 88. Supplied by 3 paired arteries B. Which of the following would be seen on histology 1. Myxomas2. Plummer Vinson syndromeB. Heparin induced thrombocytopenia B. Blood group A+ transfusions reaction due to: A. also called the atrial reflex. Regarding HEART what is appropriate: A. Clonorchs sinuses 79. Iliocolic Artery (all of the options were very close and with slight shuffling) (Confirmed by all seniors. PeroxicamC. Sarcoma 71. Group of young men bathing n beech next day develop blisters on back shoulder limbs region cause is: A. Minor calyces 72. Dental carries due to: (or infection due to) A. Endothelium venules destruction 81. Skin integrity D.[1] Increased blood volume is detected by stretch receptors (baroreceptors) located in both atria at the venoatrial junctions) 75. Right Marginal Artery A.60. Deep branch of ulnar 87. Post 1/3 of the tongue supplied by: A. Hassals corpuscles B.Regarding pneumotoxic center following is true: A. LactobacillusC. Color differentiation due to red and green: A. Inferior Wall MI artery involved: A. A+B. LCX 66. PregnancyC. Secretes TSH 91. Highest Cardiac Output in: A. Nucleus raphe 1. Renal Column: A. Adenomas3. Venous drainage in bracheocephalic vein 90. 64. IbuprofenB. Sup thyroid 67. Interlobar arteryC. Cholangiocarcinoma spread by: A. Posterior to frontal bone D. Glissening mass at apex of appendix 73. Herpes simplex C. What is correct about thyroid: A. Left Atrium 74. Serosa subserosa muscularis mucosa submucosa mucosa 76. Beri Beri 85. Rapid filling B. Fibrillin B. Posteroinferior prominent part of occipital bone C. Infectious mononucleosis  B. T lymphocytes in wall of appendix 2. Left Atrium supplied by: A. Regarding Thymus: A. Regarding Bain Bridge Reflex: A.Deep to post digastric and near palatoglossus a structure runs obliquely upwards A. Regarding Pituitary gland: A. Increases HR B. Staph 93. 18 year old girl has fever tenderness in right ileac fossa. Tractus solitariusC. Carbohydrate metabolism B. BacillusB. Malignant tumour mesenchymal in origin: A.Nucleus of general visceral efferents of tenth nerve to palate muscles is present in: 68. Obstruction to RCA after giving Marginal branch will affect: A. Scenario bif on Marfans wid mitral valve prolapse died and on autopsy is prolapse this was due to defect in A. Aortic valve closure outsets: A. Closure of aortic and pulmonary valve 65. Second Heart sound produced by: A. Interlobular artery D. Layer of the viscera from outermost to innermost: A. Posterior Cecal ArteryB. Part of pterion B.Which tumor involves extension to nerve sheath: 69. donot confuse urself and follow seniors advice) 94. Spectrin A.A 52 year old man underwent coronary artery bypass graft two weeks ago and now presents with purpuric patches on skin. Decreases HRC. AV Node 63. Most suitable NSAID during Lactation: A. Isovlumic Relaxation (or Isovolumetric relaxation) C. Neuromuscular blockade 78. B+C. Phenol with clorhexidine Cant recall other statements but i guess i have seen this question somewhere. The gene encoding the cholera toxin was introduced into V. Inhibits both gastric acid secretion and parasympathetic 110. cholerae carry a variant of temperate bacteriophage called CTXf or CTXφ. Renal Vein 130. 10%C. HbAB. IP3 something Cant really recall correct statements about this questions so its better to read the topic and save it in ur mind if question comes again you can choose da correct one :) (The cholera toxin (CTX or CT) is an oligomeric complex made up of six protein subunits: a single copy of the A subunit (part A). LysosomeD. Na+. PeroxisomeB. Scenario was given about massive splenomegaly: A. Transudative 123. Regarding BI Agonists: A. Essential amino acid: A. ICAM and VCAM are for: A. 5%B. Nucleous Pulposus herniation will cause compression of: 104. Antiseptic for skin: A. Cause of global blindness: A. connected by a disulfide bond. Spread of renal tumor will be through: A. Estrogen and Progesterone C.2 g/dl: A. Chlamydia Trochamatis 127. The A1 portion of the A subunit is an enzyme that ADP-ribosylates G proteins. sural and tibial nerves. Cl−. Two point discrimination: (repeat mcq) A. In respiratory Alkalosis renal compensation is: A. Deltoid is on malleolar side About 10 or 15 repeated. have no attachment to liver 103. and missing key words. RubellaC. Phenylalanine D. 15% 111. Mechanism of Vibro cholera: A. Retention of C02: 117. Surgery for removal of ovary done which is damaged: A. Contains Hepatic DUCT and artery 101. Fecal incontinence after trauma damge to: A. Long Saphenous ends at mid of thigh (so hard to recall this question options.. Herpes SimplexB. and five copies of the B subunit (part B). External anal sphincter 115. Regarding Lower Limb: A. This results in constitutive cAMP production. allowing it to permanently ribosylate the Gs alpha subunit of the heterotrimeric G protein. hB B. Poliomyelitis: A. Cl channelB. and HCO3− into the lumen of the small intestine and rapid dehydration. Short Saphenous Vein form from superficial arch something. NaHC03 excretion 113. Regarding cancer mutations in: A. Protooncogen B. HbC 128. Posterior communicating Artery other options seemed incorrect 105. Scenario about CHF than asked about reason for edema: A.98. About ankle joint: A. Progesterone we had so many questions who had such vague statements. lateral mallaeolar ligament on medial side E. K+. Paraaortic LN 118. which in turn leads to secretion of H2O.) C. Respiratory failure 106. L5-S1 (OR L4-L5) was in a option 116. Lymph drainage of Testes: 120. (It supplied by deep peroneal. Regarding H2 Blockers: A. B. Increase HR A. Which is normal HB: A. Virulent strains of V. ScapulaB. LipsC.[13] Binding exposes its active site. Checks probability B. Leucocyte adhesion 100. Cardiac disease scenario wad there and they asked about the fluid accumulation. dec HCO3C. the disulfide bond is reduced. is a bilayer structure A. The five B subunits form a five-membered ring that binds to GM1 gangliosides on the surface of the intestinal epithelium cells. Protoplasmic Astrocytes 125. Mitochondirac. Upon binding. Exudative B. Repeat mcq from asim regarding gray matter foot processes: A. Regarding P Value: A. they were weird n confusing. Supplied by superficial paroneal nerve C. Palms 129. Chornic Malaria (Both does so choose da option on ur own risk :) 124. HbFC. 109. Protein D. H2O2 found in: A. so cant really decide the mostt appropriate one for this question also. Anterior horn cell defect A. Regarding Lesser Omentum: A. Supplied by superficial artery B. Phosphate 126. hC03 C. Thalesemia carrier percentage in Pakistan: A. Internal Iliac ArteryB. RER 99. Most important renal tubular buffer is: A. Dec Colloid osmotic pressure 108. the complex is taken into the cell via receptor-mediated endocytosis. Once inside the cell. External Iliac ArteryC. Estrogen B. Uterine Artery 112. Fibrocartilage on hialine surfaces D. 107. proteins more than 3. Percentile something. while the A2 chain fits into the central pore of the B subunit ring. better to read the topic) 119. the superficial peroneal nerve also supplies the ankle joint) 114. saphenous. Attaches to second part of duodenum 102. and the A1 subunit is freed to bind with a human partner protein called ADP-ribosylation factor 6 (Arf6). Leishmanias B. so do Asim n Shoaib well! There were questions about: • Cruciate Anastamosis . Occasionally. Breast atrophy in young female is due to: A. cholerae by horizontal gene transfer. Joining of ICA and Basilar artery: A. arch of aorta is in front of it c. membranous 22.  47. hepatitis c.  45.. silicosis d. IL6 b. hemolytic anemia d. hemophilia b. diagastric has two belly between these two belly central tendon is attached to a. 36. 30. CRL at 17. about calcitonin a. use extensive handwashing c. morphine b. which is secondary cartilagenous joint a. 20 to 25cm 44. 7.  24. membraneoproliferative type 1 38. epiphysis b. fibrin (repeat of few days back paper ) 18. cimetidine 10.. 18. lidocaine b. 56. women having non productive cough continoously went to vacations and remained ok there for one month and then came back again to take care of her pets and the disease came back x ray normal . histamine degranulation 43. 10 59. generalized lymphadenopathy biopsy shows diffuse polyclonal hyperplasia a. 35.  58. total bilirubin 10 unconjugated 8 conjugated 1 a. h/k blocker a. most abundant immunoglobulin a.. single lobed 13. class three anti arrythmic a. lateral side of elbow C7 c.ray showing lower lobe plaques a. bilirubin will appear in urine c.  37.  23. pt having HIV and having infection a. 43. completely retroperitoneum b. 40. metaplasia a. decrease cd 8 cells 28. none of his brother has such problem a. glucose 1. factor 5 b. when will left ventricular pressure will be highest with normal pressure a. cannot be given in pain of padget b. extension of dura d. adrenaline b. asbestosis c. glycine b. 46. na absorption 49. call the police c. 26. will not effect the kidney 19. which is appropriate a. coronary blood flow is maintained by a. cefo something bholgae  7. lie in front of thyroid d. dorsal colume and trigeminal nucleus b. pubic bones together by A. wear glooves b. 48. Growth hormone c. erythropoietin b. neonate having inclusion infection of eyes same infection mother has in her genital canal a.  50. group with disease risk factors compared with control a. vit k deficiency c.  52. 31.. chlamydia 33. hepatic growth factor aurbhe options theendont remember 3. 33. pubic symphasis 41. RCC b.. ididnt see other options and marked Y chromosome with kLgi gene coz dr Salahuddin Kamal sir said that its from old ganong 5. 9.. 12. hhv 8 ( kaposi ) dontremmber other options 4. you working in ICU what thing you need to do to prevent iatrogenic infections a. case control b. type 1 hypersensitivity 2. hyoid b. urea b. 32. smoker. max absorption of which substance a. sertoli cell d.  39. mesentry starts at duodenojeujinal junction 56. a. anthracosis 37. say him to use contraception e. which involves in histamine release a. 41. 42. T cell lymphoma d.  17. if main somatosensory area along with all terminal destroyted what will be effected a. lambda chains in kidney elm a. costal margin in midclavicular line T8 34. styloid 12. pt weight gain cold intolerance case of hashimotothyroditis best investigations a. in epidydimus non differentiated cells at the BASE a. conjugated bilirubin raised a. 28. it will lead to hepatitis ( very wierd q due to and lead to strange english   ) 35. growth factors 51. join dura with vertebra b.20 wks a. hodgkin lymphoma 29. typical kallmann syndrome aya. hashimotothyroditis is a.  19. aldosterone secretions 50. 34. which factor def will lead to bleeding a. tissue injury is due to which amino acid a. u will keeo treating b. adenosine b. pt came with cardiogenic shock how would u treat him a.anti microsomal antibodies and antithyroglobins antibodies 26. endothelial growth factor c. 3. spermatogonia A b. action of angiotensin 2 a. NO 14. 15.. hepatitis 32. after hepatectomy what will help in regeneration in initial stages a. membranoproliferative type 2 b.  27.. will be helpful in vitaminosis d c. type 2 3. lateral geniculate 58. penicillin 31. dopamine 47. 16. generalized sympathetic activity 53. IgG 39. functional change 11. quinidine c. extension of filumterminale 2. HiV c.  49. worker in construction company reles in chest x. epi plus epiphysial plate 9. 25. herpes zoster b. left homo hemi 57. 13 to 17cm b. he went to physician .• Medical Geniculate Nucleus • Resp Alkalosis n compensation • Internal Thoracic Artery • Cervical Vertebra osteology • Pituitary Tumour • Foreign body in Supine Position which lobe involved • Meningitis type values were given • Development of brain which give rise to which 5th January 2015 Medicine and allie what iss appropriate about denticultae ligament a. hemolytic anemia is caused by it b. tissue thromboplastin 45. pt with HIV says you are not to tell anyone a. destruction of right optic track a. which are parasympathetic nerves a. strangestatment pulse 120 bp 120/50 what respone will help in overall survival of body a. will u go with his will 24. join adjacent vertebra c. nipple at T2 b. 54.  55. optic tract goes to a. omperazole b. true about thymus a. DIC started with a. secondary center of oss. stone obstruction by it c. post streptococcal 21. factor 12 46. a.  21. poor anti inflammatory b. 38. cimetidine 8. cerebral ischemic respone 2. 57. boy having hemarthrosis and driarrhea for last few weeks .  14. hypoxix hypoxia wala typical question 52. paracetamol a. patient having pain in chin area he took tegrel 200 he didnt get ok. glomerulonephritis. cohart 25. circadian rythum is controlled by a. best drug for gonorrhea a. drug for gas gangrene a..  53. symphasis 40. what is true about forth part of duodenum a. crigler e. gilbert 16. paraneoplastic syndrome a. T3 T4 TSH b. vasoconstriction b. type 4 27... taking warfarin what shouldnt be taken together a. isovolumetric contraction 17. pt with sore throat for six weeks . spermatids 23. b. 20. crescent in kidney elm specimen formed by a. due to decrease in helper t cell decrease b. red purple nodules on arm oraalthrushhdiarrheaa cause of it a. small cell ca 36. physician saw eruption along trigeminal nerve a. amiodaron 20.  22. dorsal colume and spinal area and trigeminal ( issitarha kay mixed options theen ) 55..a. .. cipro b. lie on sternohoidsternothyroid muscles b. what angiotensin 3 will do a. wilson b. tramadol 6. infectious mononucleasis b. choline 42. RBC production will be coz of a. suprachiasmatic nucleus 48. berylliosis b. spermatogonia B c. wear mask 30. 29. 51. TSH c. extend from lower border of thyroid to forth costal cartilage e. absorbs ca from intestine 15. ElM granular appearance . trigeminal neuralgia 54. will persuade him to tell her wife d. Thiopentone Q19 Which of the following statements is not true about etomidate? A. two in number b. what is present in umbilical cord at birth a.MCPS) Q1 Which of the following anaesthetic agents can induce epilepsy? A. 69. In a child undergoing an elective/routine eye surgery D. Dexamethasone Correct answer : A. what is true about pseudocolumnar epithelium a. Mephentermine D. Isoflurane C. Irritant nature of ether increases salivary and bronchial secretions C. Combined Spinal Epidural anaesthesia Correct answer : C. Diazepam D. Esmolol Correct answer : C. Controlled Mechanical ventilation (CMV) Q9 The most appropriate circuit for ventilating spontaneously breathing infant during anaesthesia A. Digital nerve block Q5 Which of the following is correct regarding colour coding of anaesthetic gases? A. In which congenital anomaly pul trunk and aorta will remain attached a. except: A. Ether C. two umbilical arteries b. 3.  59. Induction is slow B. Horner syndrome a. Ketamine Correct answer : D. ln a patient with a large tumour in the oral cavity Correct answer : D. Sevoflurane B.  61. Atracurium Q7 A Lower Segment Caesarean section (LSCS) can be carried out under all the following techniques of anaesthesia except: A. Mapleson A or MgiIl’s circuit C. It precipitates coronary insufficiency B. Eversion is done by a. Subtalar joint 74.  68. retinoblastoma b. which virus causes ca a. colonic ca 62.  76. Atracurium B. Hydralazine C. all cells are at base b. Cricoid is half ring b.1 mg/dl ? A. physiological dead space a.Asad Senior Doctor(FCPS. nerve free endings b. what will be the epithelium of pre.  63. Mydriasis b. Methoxyflurane Correct answer: A. Trunchus arteriosus 73. Sevoflurane Q15 Which of the following agents is not used to provide induced hypotension during surgery? A. inguinal canal a. General anaesthesia B. two umbilical veins c. 64. pretrachealfasica 69. Diazepam Correct answer : C.  72. Ketamine Q4 ontraindications to vasoconstrictors in local anaesthesia? A. Q20 Which one of the following agents sensitizes the myocardium to catecholamines? A. digoxin is recommanded for a. Epidural anaesthesia C. Peronouslongus 75. Vecuronium C. Nitric oxide – Blue C. small cell ca c. Orotracheal tube C. How many bronchopulmary segments are in upper middle and lower lobe of right lung respectively a. trachea surrounded by a. Sevoflurane Q2 Which of the following is not a definitive airway? A. radial 67. Laryngeal mask airway Correct answer : D. Affects blood pressure and is liable to produce arrhythmias Q14 Which one of the following anaesthetic agents causes a rise in the Intracranial pressure: A. Nitric oxide – Blue Q6 Which anaesthetic agent is preferred in a patient taken for surgery with bilirubin of 8. with cold . Halothane D. Appropriate about cerebellum a. 5 72. Mapleson C or Waters’ to and fro canister D.  62. Ptosis 76. allantions d. atrial fibrillation b.  66. It precipitates coronary insufficiency C. heart blocks 63. Purkinji present only in cortex b. Ondansetron C. Pethidine Q17 Which of the following intravenous induction agents is the most suitable for day care surgery? A.terminal bronchioles a. simple cuboidal 65. Nasotracheal tube D. 70. Laryngeal mask airway Q3 naesthetic agent contraindicated in hypertensives is? A. 74.  71. Spinal anaesthesia B. Atropine D. Spinal anaesthesia C. increase in lung injury 70. supply to anterior aspect of right atria d. reduce in excercise b. prostatic ca d. Tracheostomy B. Thiopentone sodium C. formed by conjoint tendon c. In cardiopulmonary resuscitation C. Propofol Q18 Which of the following anesthetic agents does not trigger malignant hyperthermia? A. Affects blood pressure and is liable to produce arrhythmias Correct answer : D. Ketamine C. Pressure support ventilation (PSV) D. Propofol Correct answer : A.6 mg/dl and serum creatinine of 2. Propofol Correct answer : C.  65. Cricoid attached to phyrangeal?\laryngeal? Muscles 77. Assist-control ventilation (ACV) Correct answer : A. Suxamethonium D. all cells reaches apex c all have nuclei at same level 66. Sevoflurane B. Pethidine Correct answer : D. Morphine B. Rocuronium Correct answer : A. Controlled Mechanical ventilation (CMV) B. krause bulbs 64. Regional anaesthesia Correct answer : C. ductus venosus 61. right coronory artery arises from aortic sinus and it will give branch Posterior ventricular waht is true abt it a. Jackson Rees modification of Ayre T piece B. Halothane . Halothane B. Cautery cannot be used D. Lignocaine D. Ketamine Q13 All of the following are the disadvantages of anesthetic ether. Pancuronium D. Thiopentone B. Bains circuit Correct answer : A. Digital nerve block D.  75. Oxygen – Black with grey shoulders B. absent at birth b. supply to pulmonary conus 68.  60. Cricoid attached to vocal cords c. Desflurane D. Jackson Rees modification of Ayre T piece three in number c. from ant sup iliac spine to pubic tubercle 71. What is true a. Sodium nitroprusside B.60. 67. Suxamethonium C. It inhibits cortisol synthesis D. Isoflurane B. Present throughout cerebellum Q10 The laryngeal mask airway used for securing the airway of a patient in all of the following conditions except: A. Isoflurane C. 2. Helium – Orange Correct answer : B. Inversion and eversion takes place at a. In a difficult intubation B. Caudal anaesthesia D. Thiopentone Correct answer : D. Anaesthesia MCQs Dr. Mephentermine Q16 Which of the following agents is used for the treatment of postoperative shivering? A. It is an intravenous anesthetic B. Propofol B. Propofol D. Synchronized intermittent mandatory ventilation (SIMV) C. cold recptors are stimulated when body will be cold which one will be stimulated a. ln a patient with a large tumour in the oral cavity Q12 The following are used for treatment of postoperative nausea and vomiting following squint surgery in children except: A. pseudostrtifiedcolumner b. Caudal anaesthesia Q8 The following modes of ventilation may be used for weaning off patients from mechanical ventilation except: A. Cyclopropane – Red D. Thiopentone sodium C. It causes pain at site of injection Correct answer : B. supracondylar fracture pt unable to extend her lateral fingers and sensorey loss of lateral two third of palm nerve damage a. median b. Ketamine B. Propofol D. 20minThyroxinTestosteronegnrh…2minOxytoxin. Vecuronium C. Procaine B. Rocuronium D. Procaine C. Doxacurium Q26 Which of the following is not an indication for endotracheal intubation? A. Desflurane D.. Succinyl choline D. Doxycycline D Cefotaxime Correct Answer B. Procaine Q23 Which one of the following antibacterials should not be used with dtubocurarine? A Norfloxacin B Streptomycin C. Pulmonary toilet D. What will be the anesthetic induction agent of choice? a) Thiopentone b) Ketamine c) Propofol d) Diazepam Correct answer : c) Propofol Q35 Which of the following produces dissociative anaesthesia a) Ketamine b) Propofol c) Thiopentone d) Enflurane Correct answer : a) Ketamine Q36 Which of the following inhalational agents has the minimum blood gas solubility coefficient? a) Isoflurane b) Sevoflurane c) Desflurane d) Nitrous oxide Correct answer : c) Desflurane (factual question) Q37 Which of the following in anaesthesia will produce decreased EEG activities a) Hypothermia b) Early hypoxia c) Ketamine d) N2O Correct answer : a) Hypothermia Q38 Which of the following is not true about Xenon anaesthesia a) Non explosive b) Minimal cardiovascular side effects c) Slow induction and slow recovery d) Low blood gas solubility Correct answer : c) Slow induction and slow recovery Q39 Merits of nasotracheal intubation is a) Good oral hygiene b) Less infection c) Less mucosal damage and bleeding d) More movement or displacement of endotracheal tube Correct answer : a) Good oral hygiene Q40 Most common nerve used for monitoring during anaesthesia a) Ulnar nerve b) Facial nerve c) Radial nerve d) Median nerve Correct answer : a) Ulnar nerve Q41 All are seen in Malignant hyperthermia except : a) Bradycardia b) Hyperkalemia c) Metabolic acidosis d) Hypertension Correct answer : a) Bradycardia Q42 Side effects of oxygen therapy are all except : a) Absorption atelactasis b) Increased pulmonary compliance c) Decreased vital capacity d) Endothelial damage Correct answer : b) Increased pulmonary compliance Q43 Muscle relaxant of choice in hepatic and renal failure a) Cisatracurium b) Rocuronium c) Vecuronium d) Rapacuronium Correct answer : a) Cisatracurium Q44 A young boy undergoes eye surgery under day care anesthesia with succinyl choline and propofol and sfter 8 hours he starts walking and develops muscle pain...uppr part of tracheac. Mepivacaine Correct answer : A. Correct answer : d) Perception of an ordinarily non noxious sitmulus as severe pain Q28 Which side effect is commonly seen with fentanyl? a.. Atracurium B.Q21 Which of the following inhalational agents is the induction agent of choice in children: A.Hypertension Correct answer : a) Chest wall rigidly Q29 Which of the following drugs is contraindicated in a patient with raised intracranial pressure? a) Thiopentone b) Propofol c) Midazolarn d) Ketamine Correct answer : d) Ketamine Q30 The following anaesthetic drug causes pain on intravenous adminstration: a) Midazolam b) Propofol c) Ketamine d) Thiopentone sodium Correct answer : b) Propofol Q31 Which of the following fluorinated anesthetics corrodes metal in vaporizers and breathing systems? a) Sevoflurane b) Enflurane c) lsoflurane d) Halothane Correct answer : d) Halothane (Factual question) Q32 Which of the following is the neuromuscular blocking agent with the shortest onset of action? a) Mivacurium b) Vecuronium c) Rapacuronium d) Succinylcholine Correct answer : d) Succinylcholine Q33 In general.. Propofol Correct answer : Succinyl choline. What is the likely cause? A.Shortest half life Aldosteron. Pneumothorax Correct answer : D. the last muscle to be rendered akinetic with a retrobulbar anesthetic block is: a) Superior rectus b) Superior oblique c) Inferior oblique d) Levator palpebral superioris Correct answer : b) Superior oblique Q34 A 20-year old patient presented with early pregnancy was admitted for Medical Termination of Pregnancy (MTP) in day care facility. Due to the effects of eye surgery C. Rapacurium B. Lignocaine B. Methoxyflurane B. Early mobilisation B.cricoid GYNAE N OBS PAPER 4TH FEBUARY 2015 2. Isoflurane Correct answer : B.. Streptomycin Q24 Which of the following is the muscle relaxant of choice in renal failure? A. Pneumothorax Q27 Which one of the following is the description used for the term allodynia during pain management? a) Absence of pain perception b) Complete lack of pain sensation c) Unpleasant sensation with or without a stimulus d) Perception of an ordinarily non noxious sitmulus as severe pain.b. Muscle pain is a common adverse effect Q45 Which of the following is not an amide linked local anaesthetic? A. Bupivacaine D. Maintenance of a patent airway B.1 t0 5min . To provide positive pressure ventilation C. Rocuronium Correct answer C Atracurium Q25 Which one of the following muscle relaxant has the maximum duration of action? A.thyroid…. Sevoflurane Q22 Which one of the following local anesthetics belongs to the ester group? A.. Lignocaine D.Chest wall rigidity b. Bupivacaine C.Pain in abdomen d. Sevoflurane C.Development from 4th n 6 th arch a. Doxacurium Correct answer : D. Pancuronium C Atracurium D.Tachycardia c. Dibucaine Correct answer : Procaine. It is an ester linked anesthetic 1. ) 7.relaxation will occur d/2: muscle spindle.... His BP is 90/60.drains by left hepatic duct 33..component of reproductiv health b... 1st line of defence in malignant melanoma excision a macrophagesb cytotoxic T cellsc. abt quadrate lobe.. foramen ovaleB.. )Not Included In Blood Coagulation Profile  .not frm any pharyngeal pouch.adeno diphosphatec.chlamydiac...pouch4 21. epidural anestheisa failed in second stage..DysgerminomaPhyloids tumourSq...ivc. group of lymphnodes 24.Gonadal ridge 30.. Premature delivery related to anomaly A .capitulum..natural killer cells 5. lowest oxy concentration in which part of fetal circulation??svc.10ml/l 3.pouch 2D.functionally part of rt lobe C.trichom 11. muscle is stretched...Bt..failure of ovulation...renal a genesis (As da kidney develops from metanephric mesoderm whch forms glumeruli nd collecting tubuls and ureteric bud from mesonephric mesoderm. pleomorphism increase nuclear to cytoplasmic ratio increase mitosis anaplasi or keratin pearls 27.lies bw ivc and lig venosum B...hypothalamic dysfunction 6.lmbu stamtement k sath 28. the cause of failure is due to which segment not . serum cortical level is 11 microg/dl .umbilical vein.Pt....3. Safe mother hood a..supply by internal thoraxic artryb. blood sugar level is 110 mg/dl...??B.trochleaC....candidab. a... CEA 22.i did thisC hyperchromasiD cell swrlling 16.cell ca..19.MMR maternal mortality rate A 5/1000 live birthsb 5/100000 populationc50/10000 live births 25.medial epiB..adenosine monophosphateb. What is the diagnosis?  A... Plt…tlc.c.. Pt comes in to the emergency department with seizures..asim gynae A epidydymis B sertoli cellC scrotumD seminiferous tubules8. 13.related to child n mother health only d.ageb.infection .hw to assess age by ossification centre of which part of humerus A.. parathyroid develops frm A. Basal cel ca..umbilical artey 23.pouch 3E. Tetany 12.dec meternal mortality 34..traumac..... Cushings diseaesC... Hasselbach triangle related to???Direct inguinal hernia 4.breast which is inappropriatea... cytokeratin pe b tha.related posteriorly by pectoralis major d.decreas water Rice's ridge…mercier?? Ridge... Baby born edematous aftr 2 days shows signs of renal failure on examination rounded mass in pelvic area on autopsy cystic mass is seen a.adenyl cyclase 14.frm pouch 1C.heredityd.pre eclamsia 37. The specific feature of Apoptosis A cell shrinking B karyolysis…..golgi tendon organ 10..patent ductus arteriosis 18...Another name o interureteric ridge is? a.21ml/l 2.lack of gonadotrophinsc.15l/lt 20. 12yrs old girl... Serum Sodium level ?. which one is secreted after sweating 9.... Medullary rays containA. His serum calcium level is 6. primigravida 28 weeks gestation came with puffy face and hypertensive n hypoalbuminuria a acute glomerulonephritis b lipoid nephritis c.... Decreased erythopoitin production 29. Tumour marker of CA colonA.failure of mesonephros development b.b.n dose of anesthetic was repeated.. The carcinoma wch doesnot cause distant mets.... A man with 70 cardiac output 60mmhg po2 8mg/dl hb o2 bind1.related to mother health only C. Addisonian CrisisB.failure of metanephros development c... axillary tail drains into post.ptt…anti thrombogin 3 15.. Collecting ducts 17.increas na....perineal membrane 26.. cancer pe tha k etiology does not depend on?? a. Serum potassium level ?...incraes adh…b. inferior fascial layer of urogenital diaphragm is. Jugler vein coresponding on ecg A a wave for atrial depolarization B a c wave coresponds to pr interval on ecg 36. A tall man with normal external genitalia married but had no kids he had normal ejaculation petiod but semen analysis showed azospermia cause may b …...retromemory space devoid of bloodc. Which one act as second messenger in many body reactions a. ONE QUESTION OF AFTER TRAUMA pt speak only few words WHICH AREA OF BRAIN INVOLVED ? BROCAs WERNIKEsPYRAMIDIAL GYRUSTEMPORAL REGION 31.so da right ans wd b failure of union of metanephric nd ureteric bud.true is A. Common cause of infertility a... Hypocalcemia in CRF A..lateral epi 32.. Truncus arteriosisc. 4ml of 2% lignocaine vail contain?? 80mg 35.failure of union of mesonephros n ureteric bud d.. Preterm labour due to.adeno triphosphated.. While doing LP what structure is damaged A dura matter B pia matter C vertebral ligament  d ant longitudnal ligament 53. X ray of pregnant women chest a.... stress hormone not in surgery procedure a acth.d.Cloacal memb failure to proliferate cranialy whaat happen a...endometrial ca’’’c.. Fetal cortisol 45.b.. Microtubule presents in a. There was one question the end product o f purine ? Urea  Ammonia Uric acid (ans) ..false positive…e...levator ani..true positive.mortality 47..post fornix of vagina 52.sensitivity . Conduction n radiation Sweating By prespiration  66.. All cause of uti except…asim gynae a. Heavy smoker small noduls on vocal cord may include all except A. Pain on deffecation. Diaphragm Ex... 1decrease heart rate.s2….. Parturition caused by A...para sympathetic stimulation cause ..which type of hypersenstivity? A.. slow growing tumor of thyroid papillary ca (ans) follicular ca 67.centriole…b. A newly married female came with pain in rt illiac fossa her TlC 12000 she has fever n vomiting her lmp was 3 weeks back what is diag A ac appendix. ..exostrophy of bladder b.prevelance of disease b.inferior rectal nerve 41.mitrocondria 61. How good a test at corectly diagnosing people with the condition it is…. Pudendal nerve supply all except…asim gynae a...type2Ctype3Dtype4 48.thick and viscus saliva sectetion 56.hsv 2 a…cervical carcionoma b..2. Cervical cancer common causea.lysosomes b.anorectal fistula c.c. newborn meningitis a nesseria meningitis…...intercostals Innermost intrcostls Intrnal intrcostal 65.....golgi apratus c..B paha…C glucose…D Na 55...C uterine cause…D fallopian tubes 58.chlamydia 50.sacral part coverd with hyaline cartilage c...recto vesical fistula d. Female exposed to bomb blast radiations...gollgi apratus….hsvc. Uterus n breast aftr pregnancy regress by a.whch effected most A. In quiet inspiration the muscle wch increases the transverse and ap diameter of thorax is .specifityc. Haematopoitic tissue B..l2…s1..b... Occulomotor nerve supplies A all extta occiler muscles B trigeminal ganglion C sup n inf fibers pass thru sup n inf orbital fissures…???? D compltely parasympathetic to cilliary muscles 40.receive diphtheria vaccine 2 weeks ago.hyperplasiaB..... Possitiv predictive value affected by a......tuberculous 49....plane variety of sinovial joint b...covered b amnion on its fetal surface b.asim gynae a... These was question regarding a man in room temp 21 degree humidity 80%  How ll heat b lost from body..is termed as chorioalontoic placenta d. Kidney 46.type1B.atrophy 42.. a....c.ovarian ca 51..endoplasmic 57. Superficial inguinal nodes 44...imperforate anus 64..B salpingitis.neoplasiaB. Regarding spermatogenesis A.....b Adh…c Glucagon.s4 38.d Insulin 39..B.....joint pain..seperate from uterine wall along plane of stratu spongiosum e..incidence of diseas c.dudercin bassilusc.periphral vaso dilation  3..take directional movement in seminiferous tubules 43...undergone anesthesia t10…l1.is hemochorial c. Patient wid fever. Capicitation occur in epidydmus c...b ecolic cryptococus neoformans 54…Following has high clearance value A inulin .confidens interval 59. Human pkacenta a.pudendal nerveb. Starts at birth. . Sacroiliac joint a...sacral part coverd with elastic cartilage 60.ischiocavernosis.hpvb..can not b dangerous for women if abdomen coverd with lead sheet 63.seperate from uterine wall due to rupture os many uterine vessels 62.....irradiate with radiation of 45mil rads b.ecolib....s3. Lymphatic drainage of labia majora B.. 40 b... pelvic ureter NOT a.. One ques was  Pregnant lady anaemic iron level seen by increase in : Cbc…......Sr ferritin…. Diabetic pt wd neuropathy nd foot ulcer A.murcur 77.increas c... femoral nerve its root value  L2 L3 L4 99.cause of hepatocellular ca a.cardiac toxicity ocours is reversible b. Least peneterance a.b..55 85…left ovarian artry arise from Abdominal aorta 86.effect of drug on body Pharmacodynamic 88.follicular phase 98.leser sac 87. Tubular organ nd lymphoid tissue in mucosa nd submucosa A....premalignant lesion of vulva Pagets diseas 95..baby at station 2 during delivery needle pirces Para aortic nodes Ant fontanele a.ip3 B.b....supra pubic incision….50 c.b.pierced parietal peritoneum before entering bladder 79. CO 5...left colic flexure.shows incresin cardiac out put with incr venous return b..Ivc+ra 84..starling s law…ganogn mcq a....endometrial biopsy on 18th day of lmp shows 81.Tibc 74..pancreas…b.short duration 75.. Angiopathy nd neuropathy 78..axillary lymph nodes 69.???..anteroposterior view Right border of heart formed by: svc…Ra…Lv….caudate with corpus striatum....by activating genes c..basal ganglia…a.....68...cardiac output mainly depends a... Feco oral rout Hep E…Hep b…Hep c…Hep d 71....o with excersice 92. Cecum 97. behind mesen colon b terminal part is uterine artery c.oxytoxic n adh arises from\.lead…..hep c 91..median n its range 70.6 HR 70 EDV 160 ml EF???? a 0.ovulatory ohase…c.but mostly people mark this 90..secretory phase... Steriods act by a... esmolol not a b seletive beta blocker short acting??? 96. Appendix B. most common cause of dysplasia hpV Multiparity 76.b..Inf epigastric artry damage 82..aflatoxin diet b.less toxic  c.....copper….b. 0...c.common cause of hepatocellular ca a.anterior relation of kidney a.. Fibrinoid necrosis 73.tegment tempeni 94.. Rupivacaine preffered to bupuvacaine due to a.ovaries lymphatic drainage 100..on stroke volume....hep b.pain in medial of thigh on medial rotation of thigh Pelvic appendix 80...mode n its standard deviation c.venous return 93. 0.hep c…I mark this c.upper outer quadrent of breast drain in to a.aim of drug therapy…asim gynae Non pharmacological methods prefer 1st 89.mean n its standard error of mean b.HLA DR4 83...Hypothalamus…nurohypophysis Rheumatoid arthritis ... Confidence interval a...c...camp 72. Polyarteritis Nodosa A..both hep b n c…. .glucagon increase by 117....not transported by plasma proteinsa..Palate 119…true about spermatogenesis…asim gynae 102.local anesthetic effect…asim ques Uterine artry a.adh causes Excerscise…Glucose Contraction of vesels 137.after gastrectomy which should add in diet 130....AB132…1 carbon transfer byFolic ascid…Biotin…I mark this 133..bleeding aftr c.Hemolytic anemia 110....Pltlt transfuse a.1 spermatid give four sperms 103..HELLP SYNDROME…..completed in 20 days…b.bitemporal hemianopia b..section ruptr of 121.leukemia…I mark this….hb 6 dysphagia platlet normal Iron def anemia 115.o2…b.c.heart muscles…c.true about supra renal gland Supplied by inf pherenic artry a..which anti emetic hav maximum bio availability Vesico vaginal fistula Metoclopramide…but maximum mark this 104..breast feeding started With in half hour…With in hour…2 hrs Palatine tonsil…Tongue.i….not depend on route of administration Uppr fibers of chiasm c....parasympathatic cut down mostly affected GGT…ALT a.pregnancy antithyriod drug…Propylthiouracil 108.bilirubin.A+…b....after c.Hemolysis.preg lady on dental chair aftr some time unconscious due to pressing which structure.git muscles…b.co2..elivated liver enzyme...Black water fever 111..hb 8 retic 10% red cells decreas ….Ketogenesis in liver 116....Evacuation of uterus…Blood transfusion….thrombocytopenia a..skeletel muscles 125.effect change by change in ph 106.reaction of blood transfusion of A POSITIV GROUP a...essential fatty acids…Linolenic ascid Urine culture 135.autosomal dominant Odansetron… I mark this 105.......osteogenesis Decreas plasma osmolarity .....gas gangrene….spleenomegaly splinter hemorrhages fever Blood culture Blood cp 131.sle 129.hypotension 112..Adh.female with iud best management….b..Toxic shock 126…plasmodium falciparum….lyphoid.started at birth…c..Thrombocytopenia 113...not a feature of malnutrition…...mcq of free redicals not remember… 136....i mark this pta ni brain ghom gia tha but smj ni a rae thi coz sb hi transport krte Wet gangrene 123.polycystic kidney …..c...section there is dribbling of urine thru vagina after 15 days 120..independent of adding epinephrine Middle fibers of chiasm…Lower fibers of chiasm 107.insulin deficiency…Increase activity of lipoprotein lipase…....apthous ulcers….carboxylation of fatty acids by…Biotin 134.Posterior fontanale dec urine osmolarity 101..A-....vit b12….b.superior mesenteric artry thrombus intestine black 122.iron 114.vitamin b12 deficiency with mouth ulcers n perineal ulcers 128.alcohopl inhibits ..structure in mouth coverd with stratified squamous epithelium 118.1st indicator of hypovolumic shock is 127.Oxytoxin 124…in preg gallbladder stones investigation 109...Inferior vena cava Olygouria…m.c. ..drain in dural venous sinuses 153..highest protein content HDL 146. Fio2 max dose which does not cause fetolental adrenoplasia:)  0.c.medullary rays By collecting ducts 142.extensors of thigh….L4 …..anteriorly related to pituitary…..5… 1 2.physian should discourge the anger of pt n atendents againt department…I mark this b.group 149.nor epinephrine b. resting membrane potiential. Oxygen-enriched air has a higherFiO2 than 0...b...anion gap….. to avoidoxygen toxicity 140.9% oxygen.. In emergency department a..a. -cardiac reserve.rectus abdominus…b..adrenalline 148..00.terminal bronchioles 164.Acetylecholine c...Below FORAMEN MAGNUM a.verteberal coloum attachment L2…. up to 1.b..phycological therapy should b imp for pt and atendents n family 147.....26  Fraction of inspired oxygen (FiO2) is the fraction or percentage of oxygen in the space being measured.a.. which means 100% oxygen. GFR increase by .10ml/l 3.seen in some cases of turner syndrome…b.not supplying scalp Occipital arteries Maxillary artery Posterior cerebral 143..acute inflammation exudates Increase lymphocytes Protein more than 3gm 150.. Medical patients experiencing difficulty breathing are provided with oxygen-enriched air.comon cause of uti in old age urinary obstruction indeweling catheters hosp acquired 158.pregnancy with antituberculous drugs instead of taking ocps Ethambutol Pyrazinamide Rifampicin 144. mucosal glands absent in. produced by intestinal bacteria...internal oblique 141. Natural air includes 20. A man with 70 cardiac output 60mmhg po2 8mg/dl hb o2 bind 1.primary cause of lymphadema Dec oncotic pressure due to protein dec Problem with lymphatics 139.. wound healing...all cases.c. due to K efflux NA K pump 159.new cases…..only pt should allow to come in…aftr discussion c.. Post synaptic sympathatic mainly secretes a. Blood remain in ventricles after max contraction?? 163...21.epileptic women on antiepileptics become pregnant what should u advise Add valproic asid Change to phenol barbitone 145. which is equivalent toFiO2 of 0.a. FiO2 is typically maintained below 0. ECM+cytoskseton….15l/lt Osteosytes have many nuclei…I mark this by tukka Density increase aftr deposition 138..cadherins…Integrins 160.optic chiasm…....5 ....21.21ml/l 152.barr bodies inappropriate…. which means a higher-than-atmospheric FiO2....myofibroblast contraction…fibroblast contraction 161.Increase growth by increase interstitial growth 2...difference b/w measured anions n measured cations 162....vit K .external oblique …d. Prevelence is.. antioxidant vit E 156.completely inactive 154…a boy with rash sneezing n rhinorhea Type 1 hypersencitivity 155.5 even with mechanical ventilation...most dilated part of spinal cord 151.decrese resistance of afferent arteriole Dec resistance of efferent arteriole 157. degree of differentiation 167.. pericardium 182. -major buffer in blood.. -intrinsic pathway activation. S1 heart sound.diaphragm 189-opsonization.decrease helper T cells in AIDS 179.. 3rd month  191..pseudomonas 169.. golgi tendon organs 174. -inflammation.. metaplasia definition .8 cm from midline 176. ischemia.isovolumetric contraction 170.. Bicarb 184..pain on lying ..27.. -arterial n venous occlusion.. CRL becomes half of bitemporal diameter ..coagulation necrosis 181..macrophages r basic scavengers 177-neutrophills r basic phagocytes in blood 178..post operativ wound greenish pus ..5 172..cimetidine.collagen exposure 186.increased maternal age 173... loss of ankle jerk. -shift of O2 dissociation curve to right 192…thyroid gland. broder's classification.teratoma 171.5th to 10th week 166.C3b 190.165.relaxation of muscles. -local metabolites of myocardium 187-not a basic tissue. down synd..S1 185.decrease hepatic clearance 180.... all germ layers.. ant relation of kidney.velocity of blood 168. apex of heart.golgi tendons 175.median of diff values.. tension in muscles .blood 188.. rubella infection.. definition of FRC Volume of air remain after tidal volume 183.drainage in deep cervical lymph nodes 193. Which one combination is correct 1. brufen ans. lateral malaeolar ligament on medial side 5 deltoid is in maleolar side ans #14.Sensory supply of uterus passes through which ligament .A 52 year old man underwent coronary artery bypass 1. Fibrocartilage on hialine surfaces 1..Regarding elbow joint 3.Most common position of appendix Increases inspiratory potential 2. Supraclavicular nodes 1. Ilioinguinal nerve following could be the cause 11. Supplied by superficial artery 3. Capsule deficient posteriorly 2.About ankl joint 2. Progect to frontal lobe 6.Lymph drainage of lower medial quadrant of breast will NOT drain into 1. Embryonal carcinoma ans…3. Carcinoma expleomorphic adenoma part of duodenum…4.3.most suitable nsaid in lactation #16.Appendicular artery is branch of Post cecal Iliocolic ans 7. Profofol increases cerebral blood flow 3.. Fibers ascend to thalamocortical areas ans 4.. Pectoral nodes ans 2. Deep branch of ulnar ans .January 1st 2015 by mitral valve…. Fires dynamic respiratory drive ans 1.Medial geniculate body 3. ketorolac. Herpes simplex. Heparin induced thrombocytopenia 2.Lesser omentum 1. Pelvic appendix 2. Supination occurs here 5. Hemangiomas #18. Which of 2.Loss of abduction and adduction of fingers and adduction of thumb with intact skin sensations involves 1. Joimt between humerus.A needle when inserted 2cm below and lateral to pubic palpated. Supplied by superficial paroneal nerve ##4. Longitudinal  Transverse  Laigament flavum Following is true 1. Covered all arpund by muscles 3. Does not attach to liver. Retrocecal ans 1. Myxomas 2. Gonorrhea 1..sulindac #17. Hinge joint ans 4.Heterophile antibody is seen in 8. #####2.Regarding pneumotoxic center following is true 1 3.Ligament between dens of axis and process of atlas Ant. Adenomas 3. On examination a solid 5cm mass is #10. Longitudinal  Post. Recurrent branch of median…2. Same side orchiectomy was 1. Drug induced thrombocytopenia ans 12.A 31 year old young man has heaviness in his scrotum for six months. Morphine reduces apnoeic threshold ans 4. Is a bilayer ans #9. Receives oculomotor fibers 4. radius. Paracecal 13. Medial sternal nodes 4. Apical nodes 3. ulna 5. Solely to thalamis 5.3. Infectious mononucleosis ans…2. Ketamine relaxes bronchial smooth muscles #15.Which tumor involves extension to nerve sheath 1. Obturator nerve ans done and smooth glistening mass was seen. Leydig cell tumor…2. ….what is correct about thyroid gland 1)supplied by 3 paired arteries 2) surrounded by prevertebral fascia 3) #19.. Thiopental reduces heart rate 2. Optic tract fibers 5. Encloses hepatic artery and hepatic duct…2. On labs his alpha feto protein level was 81ng/ml tubercle will be on while bHCG level was 15 IU/l. isthmus attached to cricoid 4) veous drainage in bracheocephalic vein ans 5) forgot the last option 1. graft two weeks ago and now presents with purpuric Choriocarcinoma…4 Teratoma patches on skin. Inf phrenic modes 2. On labs his PT and aPTT were normal. Attaches to second 4. Biparietal diameter  #22. Situational crisis ans occipital bone ans.Thrombosis of right coronary atery after right marginal 4. Germinal epithelial protection ans #24. Nucleus ambiguous ans 4.Nucleus of general visceral efferents of tenth nerve to 2. Facial nerve ans @25. Leydig cell tumor2. Tibialis posterior ans atlas ####. visible ecchymoses around ankle joint 1.Function exclusively linked with vit E 1. Trigone 2. Internally to occipital bone ####38. Endothelial protection 3.Amoeba causes lesions in which part of gut 1. Flexor digitorum 4.. On labs his alpha feto protein level was nuchae 81ng/ml while bHCG level was 15 IU/l.2. Facial artery…2. Choriocarcinoma4 Teratoma ####37. Left atrium ans…2. Skin integrity 4. Round ligament..A 31 year old young man has heaviness in his 1. Maxillary artery4. Infundibulum of pulmonary artery #27.. Foramen ovale ans’’3.3. Lateral 3.Granuloma…Cyst…raggede reddd fibers #34.A sprinter during running had injury to his ankle. Cardinal #30.Age estimation at fifth to 12th week is done through 1.Inion is 2. Anterior longitudinal scrotum for six months.. Torn tendoculcaneus2. Embryonal carcinoma ans3. Carbohydrate metabolism 2..3.2. Right ventricle…3.Closure of lips involve 1. Ligamentum is palpated.. Styloid process orchiectomy was done and smooth glistening mass was seen. AV bundle. Superior thyroid artery 2. Sigmoid and rectum 1.. Terminal ileum and cecum ans.While a surgery on submandibular gland which has more chances of injury 1.. Descending colon and sigmoid artery has branched off will result in infarction of 5. Depressor anguli oris. Posteroinferior prominent part of 3. Broad ligament ans. She would most probably be in 1.Which ligament supports odontoid process of axis on 4..4. Penile urethra 4. Crown rump length ans palate muscles is present in 3. Posterior to frontal bone 4. Orbicularis oris ans’’5. SA node. Neck of bladder and proximal urethra ans 3.. Social crisis #28. Membranous urethra 5.3. Uterocervical ligament can stand on his toes but with severe pain. Simple squamous Depressor labi superioris’’4. #21. Lingual nerve 3 Mandibular nerve 4.1. Sup thyroid #32.3. It's superior border is formed by 1. Personal crisis 1. Tranverse ligament ans…4.Cardiax myxomas are most common in 1. Torn plantaris tendon ans Transverse colon #31. Paroneus longus 2.. Part of pterion. He ligament.A 30 year old woman in azad kashmir was injured mildly in an earthquake that killed many people apart from her distant uncle. Stratified squamous keratinized 1.Anterior epithelium of cornea is #29. Ascending colon…3. Developmental crisis 4.2. Abdominal circumference #33.5. On examination a solid 5cm mass ligament…3. AV node structure runs obliquely upwards ans 1.Loss of inversion 1. There is a #20. Foramen lacerum. Apex of heart…4. Uterosacral ligament.Highest number of alpha resceptors are present in 1.MELAS syndrome Loflar…Inclusion . Posterior longitudinal ligament…2. Foramen rotundum…4. External urethral opening ###35. Depressor labii inferioris..2.2. Stratified squamous non keratinized ans 5. Same side #26. Which of following could be the cause 1. Nucleus raphe #23..5. Simple cuboidal . Paroneus brevis 3...Deep to post digastric and near palatoglossus a Infundibulum of pulmonary artery. Locus ceruleus 2. Simple columnar pterygoid 4.Otic ganglion is between mandibular nerve and tensor tympani. Tractus solitarius 3.2. Lingual artery ans3. 2. Crown heel length  1. .mcv.. -heart blood supply -page 600 azimshaikh .. Narcolepsy5. ABGs(arterial blood gases):Po2.. Appendicectomy 6.a)arcuate artey b)minor calcyes c)collecting tubule d) interlobar n interlobular art 13.pretracheal fascia infection spread to.O2 saturation. Mch.Pco2. -Respiratory capacities.hco3.a)stomach b) colon c)panrease d) duodenum 14.phosphorus.. Histoplasmosis5... On labs WBC 17000cells/mm3. 500 mg/dl 9.. Important topics: -Opening in diaphragm. Formed only 30percent by chorioid plexus 4.apt. Infectious mononucleosis ans5.na. Chronic endometritis ####43. Blastomycosis6.platelet count.like: Serum ca.for nutritional status measurement .serumosmolarity.cl..pt.. Complex hyperplasia with atypia ans 4.mast cell destruction 10.magnesium. Option i forgot #####41.ant mediastinum 2.amylase. Formed 500ml/day ans 3.cholestrol. Hypothyroidism2. ascorbic acid 5.clefts -epithelium derivatives -skull foramina -cytochrome inducers and inhibitors.id irect)..pituatry tumour cause.k.pt e fluid loss now show metabolic acidosis e hypokalemia...hct and erythrocyte count..carrier mediated  7.Uricacid. Extends superiorly and encloses scalenous anterior E. Cushing disease ##########45.Clavipectoral fascia A. Chicken pox4.vitamin  12. Glissening mass at apex of appendix ####42.nahco3 excretion         U need to memorize some of the important values.urineosmolarity. Mucormycosis3. -celiac truk and branches -type of hyaline cartilage -cnsrecepters -epithelium -pharyngeal pouches.1st cervical process.group of young men bathing n beech next day develppe blebs on back shoulder region cause is.abt meningitis. Complex hyperplasia without atypia  3. Encloses pectoralis major C.50 yr old e lt hoarsense.Ast.A young boy with inc appetite weight gain and sleep problems. Arachnoid villi are visible ####44. CSF R/E ……………………………………. Cryptococcosis2. T lymphocytes in wall of appendix 2. Esr.2 g/ DL.ph...retention of CO2...fibrinogen. Pineal tumor ans3.mchc. Craniopharyngioma4..resp failur was preformed.creatinine..protein required. Rubella3.albumin. 11. Measles2.bilirubin(direct. Which of the following would be seen on histology 1.CSF 1..Chronic Fungal infection with rhinosinusitis causing medial erosion and granulomas1.anionga p.reticcount.odontoid process 15. Formed by ependymal cells 2. Rhinophycomycosis4.aspergillus (ans #######Simpler bcqs about bladder cancer schistosoma  Cholangioca clinorchis  Pubic symohysis secondary cartilaginous joint  Malignancy metastasis  Exudate  Itp labs  Brachial plexus multiple twisted questions and same for heart blood supply VWF scenario 1...daily production of csf.exudative pattern 4.glucose transport.. Alt.. Part of endothoracic fascia reflecting anteriorly B.cardiac infection e protein > 3. hemianopia of bilateral nasal half of visual field defect 3. On examination he is having small genitals 1.glucose... Hb.. BUN. Gh. Polymorphonuclear neutrophills ans IMPORTANT POINTS:: 3. Squamous metaplasia 5. Simple hyperplasia 2..####39.renal column.18 year old girl has fever tenderness in right ileac fossa. Mumps ###40.. Protein in urine..Incubation period of which disease is longest 1..n resp alkalosis renal compansation. Encloses pectoralis minor ans D.mediastnal lymph node e Lt recurrent laryngeal involvmnt 8.Which of following has highest chances of developing endometrial cancer   1. Anteriorwall(LAD):v1-v4 -Ulnar nerve: 1.aVL >enters palm anterior to the flexor retinaculum. -cardiac enzymes -chlorpromazine can cause parkinsonism -carotid sheath content -parkinsonism is aggrevated by Haloperidol -chemorecepter and baroreceptor -Digoxin toxicity can increase by chlorothiazide -hormones second messengers -centraly acting antihypertensive is methyldopa -T and B cells’ CD numbers -Amino acids derivatives -Serotonin is secreted from carcinoid tumor which is responsible for sign and symptoms.anteriolateral__LAD or LCX(v4-v6) >passes behind Medial epicondyle of humerous. -tumor markers Axillary nerve: -type of errors—biostat -passes through Quadrangular space.Lateralwall(LCX)__I.brevis and tertius 1. increased ECF osmolarity.III.-dna and rna viruses Radial nerve: -bones of hand -Descends Infront of lateral epicondyle.but mainly RCA) -Median nerve: -AV node__RCA >Descends on Lateral side of axillary and brachial arteries.fibrocartilage. Angiotensin III -SA node__RCA(sometimes LCA. -ulcerative colitis and crohn’s -lesion causes wrisr drop. -Glycogen storage diseases FOOT everters and inverters: -joint classifications. C.Inferiorwall(RCA)__II. -Granulation tissue in MI appears within 9-10 days and becomes prominent within 2-3 weeks. Conducting system blood supply: 3. -RBB__LCA >supplies most of the flexors of the forearm(arm flexors supplied by musculocutaneous nerve) >supracondylar fracture cause median nerve injury--------apelike hand deformity -LBB__RCA+LCA ECG diagnosis of MI: A. 2. Inverters: -cartilage types(hyaline. B. 2.anteroseptal__LAD (v1-v2) >Descends on Medial side of axillary artery and brachial artery. -reflexes values -lies against surgical neck of humerus. -purines and pyrimidines -supplies extensors of arm and forearm. decreaded volume of ECF.aVF >injury leads to claw deformity. -AV bundle__RCA >Enters palm Behind the flexor retinaculum. -age related type of meningitis -lesion wil cause paralyzed deltoid and loss of cutaneous sensation over lower 1/3 of deltoid region. .elastic -tibialis anterior ---examples) -tibialis posterior -bones classifications Everters: -thirst is stimulated by: -peroneus longus. -2ndweek___antibody/widal -PGI2__by endothelial cells -3rdweek___stool -TXA2__by platelets -4thweek___urine culture Blood transfusion induced electrolyeembalance is as follows: -vit b12 absorbed from terminal ileum -hypocalcemia -middle rectal artery remains in true pelvis -hyperkalemia -renal medullary rays—collecting ducts -hypothermia -Anxiolytic effect by GABA -metabolic acidosis -lipid which does not contain glycerol is sphingomyelin -left shift of oxy-hem curve -taste fibers from anterior 2/3 tongue passes thru internal capsule *common peroneal nerve injury__loss of everion and dorsiflexion -for controlling tachycardia.N *cervix—internal and external iliac L. -GH adenoma—acidophil _heparin____monitoraptt -ACTH adenoma---basophil _warfarin___monitorpt -TSH adenoma—basophil(chromophobe) *in axila—cords of brachial plexus -on contraction of diaphragm the intrathoracic pressure decreases.N *Urinary bladder__internal and external iliac L. -glucose transported mainly by facilitated diffusion Lymph drainage: *labia majora and glans penis---superior inguinal L. *Tibial nerve injury__loss of inversion and planteflexion. Amiodarone>lidocaine>procainamide -pelvic parietal peritoneum is supplied mainly by Obturater nerve. *base of neck----roots *posterior triangle of neck---trunk and division *detoxification of drugs---ESR _Lateral horn is present in T1-L2 -Boat shaped cyst—pneumocystis carnii _Dorsal colums present only above T5 -Epidural __middle meningeal artery _Large ventral horn is present in -Subdural__superior cerebral vein (C5-T1+L2-S2) TYPHOID fever investigations: -Eosinophilia present in: -1stweek____blood culture hodgkin lymphoma and polyarteritisnodosa.N *prostate__internal iliac L.-posterior interventricular artery accompanies middle cardiac vein -corticotropic (basophilic) adenoma__cushing disease -prolactinoma---acidophil(30%) -Anterior interventricular artery is accompanied by great cardiac vein..N -allantois__Urachus__median umblical ligament -Somatotropic (acidophilic) adenoma__gigantisim -amino acids—Na cotransport -sarcoma—hematogenous -carcinoma—thru lympatic -night terrors happens in NON-REM -nightmares happens in REM -glutamate for fast pain fibers -substance p for slow fibres -glutamate released by rods and cons . the alkaline phosphatase is not increases.alphamotorneurons -leaky junction in small intestine and gall bladder Inverse muscle stretch reflex: -cryptsecretion -Disynaptic -villiabsorption -Ibfibres -ABO incompatability in infants: -gamma motorneurons *O motherAorB baby -Golgi tendon organs *A motherB or AB baby -Horner’s syndrome happens if lesion above T1 *B motherA or AB baby -in the cerebellum.etc -nuclei which lies beneath the fourth ventricular floor: -fat absorption occur primarily in jejunum -oblique fissure extends from T3—6th costal cartilage on both lungs Vestibular. -circumvillate—bitter taste -vommiting and swallowing centres is in the medulla Stretch reflex: -most colonic water absorption occur in proximal colon -monosynaptic -in multiple myeloma. -Ia fibers affected -tight junction in colon .granule cells is the only excitatory neurons while all others are inhibitory -progesteron is reason behind increased temperature during ovulation -trouble going downstairs==Trochlearnerve damage -Fibroadenomanot precursor of CAbelow 25 yr -trigeminal neuralgia—v2+v3 -Fibrocystic changesprecursor fro CA20-50 yr -Glossopharyngeal nerve supplies carotid body and parotid gland -Granulosa cell tumorcall exner bodies -loss of accommodation causd by damage to midbrain -Dysgerminomashrilerduval bodies -lesion to pretectum leads to loss of reaction to liht but intact accommodation --in renal failure: *early stage—dilute urine -liquid dysphagia is due to neuromuscular incordination *late stage____concentrate urine -study brain stem lesions very well.-blood brain barriers made by astrocytes -increased plasma osmolarity stimulates osmorecepters in anterior hypothalamaus.abducent.for example medial medullary syndrome. -fungiform—salty and sweet -foliate---sour taste -intestinal cells of kajal are the pacemakers of GI smooth muscles..vagal... -groove for subclavian artery is at first rib.hypoglossal. -spinal cord ends: -azygos vein enters thorax inlet *in infants:L3 -right middle lobe has 2 bronchopulmonarysegment *in adults:lower border of L1 -diaghragm is not attached to T 10 -Subarachinoid space ends at (S2-S3) -Tendon of popletial muscle is intracapsular -internal capsule---projection fibres -DIFFICULTY in: -at the level of superior colliculus—occulomotornerve nucleus-midbrain—red nucleus *flexing knee and thighsartorius -genu+anterior 2/3 of posterior limb-----------------motor *flexing knee and extending hipsemitendinous *flexing hip and extending kneerectus femoris . 7.BLASTOCYST IMPLANTATION -strongest layer in small bowel in circular layer 2NDWEEK…. The patterning of the limb development is regulated by Homeobox-containing (Hox) genes.  The upper limb buds appear low on the embryo due to the dominant development of the head and neck.narroeing.10 but not 4(calcium)  RESPIRATORY DIVERTICULUM FORMED -connections of cerebulum  PHARNGEAL APPARATUS FORMED *cerebellum to midbrain through superior cerebellar peduncle<SCP>  OTIC PLACODE FORMED ---medial handepitrochlear -lateral ventricle roof formed by body of corpus callosum -dermatome of perineal region is S3S4 -CVAheart sounds in JVP C wave first sound V wave 2nd A wave 4th -in cell cycle: Replicationinterphase Divideprohase A lign 2 chromosomesmetaphase -noradrenaline and serotonin metabolized by MAO -adrenaline by COMT *cerebellum to pons through MCP<middle> 5TH WEEK *cerebellum to medulla through ICP<inferior> METANEPHROS BEGIN TO WORK .9.  The upper limb buds form opposite the caudal cervical segments and lower limb buds form opposite the lumbar and upper sacral segments..  Upper limb buds become recognizable during week 4 and the lower limb buds become present by the end of week 4 (day 28 ). the somites (4) are well formed and the neural tube is also formed but it is opened at the rostral and caudal neuropores .-Conusmedullaris is pial extension to the coccyx -narcolepsydefect in hypothalamus -revise muscles of laryns.. -somites are present in paraxial mesoderm 1STWEEK………..Weeks 9-38: Fetal period . -lateral handinfraclavicular  At the beginning of the 4th week. implantation and formation of bilaminar Weeks 3-8: Embryological period .2.  STOMACH AND FORGUT ORGAN FORMED  PRONEPHROS DEVELOP EARLIER AND MESONEPHROS DEVELOP LATE.widening.EPIBLAST AND HYPOBLAST -embryonic primary vesicle is mesencephalon 3RDWEEK……ECTODERM MESODERM ENDODERM FORMATION HEART DEVELOPMENT START Lymph drainage of 4TH WEEK….  PRIMODIAL GERM CELL IN 4TH WEEK -liver makesfactors 1.tensing vocal cord…etc -U wave in papillary repplarization Pt had ankle sprain: -absorption of short chain FFA happens in colon -if pt can stand on toerupture of plantaris tendon -osteosarcoma in metaphysis -if cannot stand on toerupture of achiles tenton -occulocardiac reflex mediated by trigeminal Embryoooooo -thyroid derived from endoderm -albumin has low molecular weight and high concentration -amoebic abcess occur in Cecum>ascending colon>sigmoid >rectum Week 1-2: formation of zygote. . a. 1Turner syndrome karyotype.  Notches develop between the digital rays of the feet.CANALICULAR PERIOD Mesenchymal models of the bones in the limbs undergo chondrification to form hyaline cartilage. What’s your diagnosis.BEGINS TO DIFFERENTIATE 6TH WEEK 12WEEK. Colon c. Abormal glucose tolerance c.. a. a. . Weeks 9-12  The fetus has short legs and small thighs at the beginning of week 9. Squamous Cell Carcinoma c. Parasympathetic supply cut down b. Frontal 9. Infection 6. Sarcoidosis c. Iron deficiency anemia c. Foreign body b.  Primary ossification centers are present in all long bones  Order of ossification: Clavicle. At the end of week 8.after 1hr 198 after 2hr 194. Somatic efferent 8. Liposarcoma 7. a.Langerhan giant cells are found in a.Fibrous dysplasia more common in. at the knee joint. Abnormal glucose tolerance 3.EMBRYO REMAINS UNDIFFERENTIATED Upper limbs show regional differentiation with developing hand plates 7WEEK. 24 WEEK TO BIRTH….ALVEOLAR PERIOD OPTIC VESICLE APPEAR. 7-16 WEEK. At the beginning of week 8. a.  By the end of week 12. Temporal b. 20WEEK.GONADS PRONEPHROS REGRESS Weeks 34-38 DEVELOPS Secondary ossification centers appear in the epiphyses . Diabetes mellitus b. RBS 122. 2014) LIMB ROTATIONS BEGINS 8TH WEEK. Crossed by lingual artery and divi des into facial and temporal terminal b.Local anesthesia crosses the placenta by.Gram -ve infection causes septic shock in which of the following tracts. there are distinct regions in the limbs. When it is replaced around age 25. The epiphyseal cartilage plate intervenes between the diaphysis and epiphysis. Hydralazine 5. Facilitaed diffusion c. The clavicle develops by intramembranous ossification and later develops articular cartilages.Bp 180/110. Active transport d. The first ones to appear are in the distal end of the femur and the proximal end of the tibia. a. growth of the bones ends. the upper limbs have reached their final relative length but the lower limbs have not. Megaloblstc anemia 13. Wegners granulomatosis d.MALE AND FEMALE EXTERNAL GENITALIA RECOGNIZED MIDGUT HERNIATE THROUGH UMBLICAL CORD ………. 1-6 WEEK. Aplastic anemia b. Diffusion b. Biliray tract b. Bulk flow e. Smal bowel d. TRACHEOESOPHEGEAL FOLD FORMED BRONCHIAL BUD ENLARGE Bones appear during week 5 as mesenchymal condensations in the limb buds . Syphlis 4.TERMINAL SAC PERIOD 32WEEK-8YEAR.  The digits of the hand are short and webbed. etc. a.Glucose tolerance (OGTT) test was done on a lady who is fasting. with headache. Lateral to retromandibular vein 10. FCPS Part 1 Gynae/Obs Questions (Feb 12th.PHENOTYPIC DIFFERENTIATION COMPLETE. Pinocytosis 12.Atonic bladder caused by a. 44 XXYc. Cell membrane c.PSUDOGLANDULAR PERIOD 16-24WEEK.a.Which one is not an epithelial tumor. PRIMODIAL GERM CELL MIGRATES INTO GONADS TRACHEA AND ESOPHAGUS SEPERATES FROM EACH OTHER FOR LUNGSSS.Factor that delays wound healing locally is. 47 XO b.Ionisng radiation in bomb blast causes.True about External Carotida. Week 8 (Last week of embryonic life. Tuberculosis b. RER 11. Centriole b. Microtubules found in a. Methyl dopa b. Maxillary c.22DAY 7TH WEEK. a. femora. Sphenoid d. with long fingers and distinct toes. Adenocarcinoma b. 44 XYY 2. favourable treatment. L3 d. Ovarian Artery b.Uterus remains in position by. diagnosed as transtional cell CA. Parasympathetic cut down supply b. Transverse cervical ligament c. culture only d. A. Nitrofurantoin e.Hormonal therapy against malignant condition acts by.c.During 2nd week which is appropriate test for typhoid fever .After the birth of a baby the perinatal mortality rate and Still birth can be calculated in. Carbohydrate b. Pleuritis 41. Glucagon b.Anal canal extent.Regarding Breast the following is not true. Sensory loss on sole and medial aspect of foot 22. No change 34. Type4 24. a. Type 1 c. Exercise b.Collagen. To stain cells d.In neonate the spinal cord terminates at the level of.Insulin is inhibited by. Tetracyclin b.Atonic bladder is caused by. a.Uterus prolapsed but anus in position which intactligament is intact. a.Turner Syndrome a. Klebsiella c. Postive Babinski sign c. a. T wave b.Edema is caused by. a. Death within 1 mon b. E.Hormonal therapy causes regression of tumour by.Urinary tract 14. Urogenital diaphragm 23. a. Dopamine 45. Smoking c. Degeneration c.Local anesthesia crosses placenta through. Posterior fontenelle c. Facilitated Diffusion c. a. a.Characteristic of Grave’s disease. Ester vs amide c. a. extends from mid sternum to midclavicular line 27. the source of energy for the body is. Apoptosis b.a. Vaginal cyst b. a. Anaphylaxi b.Compression of S1 nerve root results in . a. Bone marrow 47. T4 decrease c. Clear cell carcinoma of vagina 48. Blood c. Uterine Artery d. Anterior fontenelle b. To denature proteins e. Round ligament e. Billirubin level increases in 7-14days after birthd. 45XO 53. 45 yr female patient presented with hirsutism on the body and is shy to show her hands to the physician. Myocarditis b. Coamoxiclave d. Pseudomonas b. Nitrosomines 26.A woman with chest pain and history of fever. Loss of ankle jerk b. Systemic response to injury 39. L4 e. It can be seen under light microscope 50.Stilbestrol in first trimester of pregnancy. Coli 17. AB. Which organism is involved.Klienfelter Syndrome a. a. Pelvic diaphragm d. Type of transport is a. Broad Ligament c. RBC increase in number b. a.Amniocentesis vs trans-cervical chorionic villous sampaling before 14 weeks a. The needle will pierce. visceral afferent 19. a. TSH decrease b. Cardinal ligament b. Alcohol 16.Which antib0tc safe in pregnancy in 8th month a. Simple diffusion b. Increase in HR at constant Cardiac Output b. Between levator anni and Anus 32. 1 yr c. liquefactive necrosis 37.10% Formaldehyde is used for sending Biopsy specimen. a.Glucose transport across a membrane is directly proportional to concentration graddient . Acetylcholine b. that becomes worse on lying down a. O36. Norepinephrine c. a.The following is true about acute Inflammation. . a. coopers ligament attaches it firmly to the underlying chest wall b.Reaction after diphtheria vaccine. Increase in the above 20. Passive diffusion 43.Percentage can be calculated as a.Growth hormone secretion is increased .Regarding septic shock does not include. L1 b.Abortion performed by dai. Have Xchromosome d. Active transport b. a. 1 day 30. To prevent autolysis c. Active Diffusion d. high dose steroids get benefit 42. Formed by chondroblast b. Isovolume contraction 40. Inulin 46. Somatic efferent c. Why? a. a. 47XXY 54. 10% increase risk of fetal talipes 56. a. 1 week d. Triglycerides 38. a. Creatinine b. Na co-transport 49.Non carcinogen a. By dividing the number by the total number of items and multiplying by 100 29. Broad ligament 18.Clinically GFR is measured by. a. Isovolumic relax c. a. Bulk fl0w c. Lymphatic obstruction 44. Reason a. a. T3 decrease d. Ovarian venous plexus c. Fat 58. Atropy b.After 48 hr of starvation. 1% Decrease chance of miscarriage b.Severe transfusion reaction occurs when A+ blood is given to . Apoptosis 51. Stool culture e. Type2 d.To prevent autolysis of cell 31.Epidural block 3% chlorpromazine is more rapid than 2% lidocaine. a. Beta adrenergic blockers c. a. a. Weight increases in 1st week 25. Local response to injury b.Post op wound causes greenish pus discharge . a. hematoma formed by a. Heterochromatin c. Facilitated diffusion d. Cotrimaxazole 55.Stroke Volume increase by. Muscle protein c. Radiation c.Preganglionic fibers release . a.Amniocentesis done in a pregnant lady at term pregnancy and the baby's head at -2 station. Muscle glycogen b. ↓ hydrostatic pressure b. Type3 e. Methylsulfate b. Aromatic amines b. Degeneration c. wht will be broken down to provide energy. Acetylcholine 21. Liver glycogen d. Broad ligamen b. Sagital suture 57.b. Hperglycemia 35.A 50 yr old smokers works in a tyre factory. what is the cause of this disease. Cyclophosphamide d.a.Barr bodies. Outer anal sphyncter 28. PKa d. Lipid solubility b.After 48 hours starvation. lung compliance increases c. a. Present in all types of connective tissue 33. Always inactive b. ↑ osmotic pressure c. Concentrati 15. Widal+Blood culture b.Baby after delivery. Augmentin c.Pressure in aorta least during. Sterilisation b. Pericarditis c. T12 52. L2 c. a. 0. The reason for his black color urine is. a. Vertical group of lymph nodes b. ICF/ECF osmolarity inreasd b. Superior group of lymph nodes 95.The medial quadrants of breast are supplied by. Most likely cause:a. high grade fever . It should be acceptable to the general population 71.Glycolysis is. a.1st heart sond is related to which phase of cardiac cycle. Hepatic disease d. Middle cardiac c. (paper 1) 64. ↑ colloid osmotic pressure 83. MCH and MCHC were given. Vitamin C c. C3B 106. a. Vitamin B-12 101. Spleen b. Isovolumatric contraction 90. a.A scenario about Odd's ratio was given. a. a.S1 lesion a. a. The most potent opsonin. a. Cytomegalo virus b. Adrenal tumor b.The labia majora lymphatic supply. Man runs loses 2 litre sweat . Labia minora c. Totally benign 66. Conduction and radiation 92. a. Pleura b. Dry gangrene b. Midazola d. a. AST c. Propofo e.Which tumor is derived from all the 3 germ layers. Polycystic Ovaries d. Dysarthria is the result of damage to. loss of heat by .Fat necrosis occurs in .A man died 5 days after MI and large blood clots was found in pericardial sac .9% Nacl 96. The mode of loss of heat from his body is by. Chest pain increases due to. Each has its own blood supply b. Vitamin D d. Puberty c. a. Centrioles 75. a. Temporal b b. Early diagnosis cures early c.Type of necrosis in brain infarction . 40% did not. Reversible b. Trauma c. a. Increase in coloid osmotic pressure d. Coagulationb.Hamartoma is.Jaundice in a pregnant lady.Fast pain fibers. Drug induced 59.Most effective gluconeogenesis is from. Labia majora 99. Right renal artery c.Bronchopulmonary segment.A patient taking oral contraceptive pills and antitubercular drugs and still got pregnant the drug responsible? a. was given antimalarial 1 day back.A 20yr old male has dyspnea on lying down.Severely dehydrated fluid to be given a. The structure most likely to cause this is. Tocoplasmosis is. a. a.The overdose decreased of which drug can bby alkalanization of urine to increase its excretion. Most common complication of dibetes mellitis ? (help me correct the stem and the other options fellows? a. Parasitic infection 109. Great cardiac vien b. ALT b. G6PD b.Person naked in room. Decrease in capillary permeability b. Human papiloma virus d. a. Ovarian tumor b. a. Pericardium 89. 16 yr child with cola color urine since one day . Interthorascic nodes b.Which of folowing causes least respiratory depression. a. Inc arterial po2 . Phenobarbitone 86. a. Which of following acts at central and peripheral chemorecrptors for respiration. Acute pancreatitis b. Teratoma c. LDH c. Thiopentene 72. Melanocytes b. Axillary group of nodes 94. GGT 84. diagnosis of premature ovarian failure made. Kawasaki disease 67. Raynaud's phenomenon 104. IUCD 108. reticulocyte 110.During an open surgery of heart the doctor saw a vein bleed. Nasophyrngeal Ca 107. Congenital adrenal hyperplasia c. Ketamin c. Different no on both sides 76. a. Increase in capillary permeabilty 78. Hb 6 . Sweating c.The enzyme most sensitive to the damange of heart muscles after MI is. Enlarged isthmus of thyroid b. Parasite 102.Person lying supine.A 20% patients received HRT. Delta 87.Regarding aminocentesis. Clioris b. The best way to present such a data is. Cerebellum b.The blood supply of right adrenal gland is. Some drugs cause RBC hemolysis b. Posterior group c.motor supply of SCM a.30 yr female presented with secondary amenorhea.Trop T b. Fats c. a. Chancroid 60. It should be 100 % sensitive and specific b. Liver 100. Values of PCV.Criteria for screening test in diagn0sis of cancer a. a.Lysosomesb. Insensible perspiration b. A scenerio of normocytic hypochromic anemia was there. ICF volume increased 103. Decreased hydrostatic pressure c.Fibrous dysplasia site a.Microtubules are present in.Drug action won’t be affected if bound to Alpha glycoprotein ina.Genital tubercle from. Rifampicin b. a. Pie chart 61. Rhabdomyosarcoma d. Occulomotor Nerve c. temperature 21 degree centigrade. ↑ capillary permeability b. Protien b. Myocardium c. Autoimun 79. a. Hexose phosphate is converted to lactate anerobicaly and pyruvate aerobicaly 81. PNH 80. Diagnosis made by a. Perforation of placenta 65. He died due to .The cause of RBC fragility. Heart c. Ultrasonography being normal. Neoplasm b.Increase in interstitial fluid presure in: a. a. Superior Vena cava 68. what will happen after the marathon. Herpes simplex type 1 c.What’s the cause. Pericardial tamponade b. humidity present. Infection 98. a Morphin b. a. Radiation and conduction 82. Wet gangrene c. IVC b. a. Smal cardic vein 97. Plasma cells 105. Broca's area 44 111. Adenoma b. Isoniazid 69.Which of the following causes increased interstitial pressure. he vein bleeding most probably may be. Ascending pharyngeal nerve b. Toxoplasma. Retrosternal goiter 73. Myoglobolin 63. and replaces it with isotonic plain water of 2 litre. Antibodies are formed from a. a. Ankle jerk loss 91. Sphenoid bone 88. a. By enzyme deficiency 74. Female of middle class has cervcal lesion a. Liver 62. Falciparum (blackwater fever) c. a. a. Faty acid oxidation is helped by a Biotin b. Helminth b. We had to calculate the Odd’s ratio 70. Triglycerides 77. Motor branch of facial Nerve 85. lateral group of lymph nodes c.A person naked with 80% humidity in room. Viral infection c. Maxilla c. Tumor caused by EBV a. Fantany b. a.Least site of tumor metastasis a. Liquefaction 93. Risk inc of talipes if performed before 14 weeks b. What is not present in dermis a . a. a. Leukocytes culture e. Simpl diffusion d. Medial 2/3 of thalamus b. Heratoma 128. TSH decrease b. Most common cause of premature ovarian failure. Tropomyosin 153. III. Langerhan Cells are seen in. The interna liliac artery was ligated to stop bleeding. a. Parasite 161. a. a. Heavy smoker has a small nodule on vocal cord. Granulosa cells 115. Corona radiata thickness. Faccilited diffusion 162. a. Labia minora c. About parasympathetic supply. ESR d. Vitamin D d. a. Levator ani peineal body 119.35 Nacl solution b. Insulin is inhibitted by a. Cyclophosphamide 120. Dry gangrene b. A-alpha b. Cells responsible. Alkaline phosphate b. c2. c2. Increased salivary secretion 121. Urinary bladder parasympathetic supply.where does the needle insert. A pregnant lady with the baby's head at -2 . ALT 144. 114. c5 159. ACH b. Mikel's diverticulum 148. Sphincter urethre ani d. Dopamine 125. a. Graves' disease. Positive Babinski sign c. Euctus deference b. . Local anesthetic pass placenta by. Theca externa c. a. The anterior wall is strenghthened in its lateral third by inguinal falx 126. Tubular organ with characteristic aggregates of lymphoid tissue a. Fatal disease of patient. Old age 155. But it needs to be learnt. a. ECF volume increase 134. Don't remember the mcq completely. Bacteremia b. a. Hepatitis c. C fiber d. VIII. X b. Blood culture c. a. Most comon carcinogen. Anterior fontanelle b. Pneumonia 118. Medial 1/3 of thalamus d. Uterine cervix b. a. Preganglionic sympathetic nerve fibers release . Toxic shock c. Vitamin C c. a. Drug induced d. a. Stratum spongiosum plus startum compactum 136.Urachus b. a. ratio > 4 at 32 week 137. c3. Appendix b. Actin b. Ester vs amide c. a. Painless deliver 129. Which tumor is malignant. Hepatocellular carcinoma d. VII. The cause may be. Adrenal carcinoma c. Shouldn't be told to the patient ad not the family b. a. Simple squamous epithelium wid clefts and smooth muscles. Most common complication of dibetes mellitis. Anemia d. Pleural pain reffered to the shoulder by. Posterior fontanelle c. a. V. Lipid solubility b. Why? a. Uterus prolapse but anus is intact. a. Lateral 1/3 of thalamus c. a. a. Which cranial nerves are parasympathetic. A delta c. Hypernatremia c. a. a.isovolumic relaxation 132. Ovarian venous plexus 117. Shrink in hypertonic solution c. a. XII 157. Virus b. a. Chemicals 139. Glucose croses placenta by. High myelinated 146. Lateral wall of third ventricle is formed by . Difussion gradient Aortic pressure lowest during which phase of cardiac cycle?? a. RBC osmotic fragility Test. Hyperkalemia b. Meningioma 140. The cause of death in gas gangrene is due to . a. Urinary bladder 135. Oligohydramnios b. Epidural block 3% 2 chlorpromazine is more rapid than 2% lidocaine?? a. Co-transprt c. Atrophy b. 113. c3 b. Theca interna b. Which structure not damaged. Remanant of vitelline duct. Autoimmune b. sharp pain from mechanical origin is mediated by which fibers. a. Biotin b. II. c1. Internal thorascic group of lymph nodes 159. Anencephaly b. Polycythemia b. Mesonephric duct functionally persists as. She has extravasation of urine. Beta blockers 122. Loss of ankle jerk b. In the resting stage the binding sites of actin mysin fibers are covered by. Increase in interstitial fluid only b. c4 c. To cause lysis of cell 133.atrial systole b. Hyperplasia d. Increase in intracellular fluid c. a. Sensory loss on sole d. Fronal suture 156. a. Pregnancy c. Renal agenesis d. Hypercalcemia 124. Active transport b. c3. a. a. Lesion of s2 s3 s4 nerve causes. Regarding the boundaries of inguinal canal. To help he specimen absorb stain d. c4. Fast. T3 dec d. Female presented with shock with history of abortion by dai . a. Which organ is ruptured? a. Concentration 160. Uterine artery b. Myosin c. a. Fallopian Tube 149. Active Transport b. Should be told to the relatives of the patient. Glycogen is not formed in the muscle due to a. Most common site of ectopic pregnancy? a. Rupture in 0. a. Compression of S1 nerve root results in: a. Pelvic diaphragm b. A femal presented with history of car accident. Tracheo-esophageal fistula 135. Hypertrophy c. a. a.isovolumic contraction c. ASO titre b. Rectal incontinence b. TB b. Helminth b. Infection 150. Sagital suture d. Not a carcinogen. Bulk effect c. Genital tubrcle form. Granulosa cells d. Vitamin B12 147. 10% Formaldehyde is used for sending Biopsy specimens. IX. Epididymus 154. The cells forming the corona radiata are. Regarding surfactant inappropriate one a. a. Female at gestational age 34 having symphsio-fundal height of 28 weeks. Norepinephrine c. Uterine body c. Sarcoidosis 116. After rapid loss of 2 liter of fluid person drinks plain water it will lead to. PKa d. Stratum Functionalis includes a. Increase in the above mentioned 127. Ureter b. Tocoplasmosis is. A and b 143. Jejunum c. a. Normocytic Hypochromic 151. Alpha feto protein is not raised in. Glucagon b. To denature proteins e. Ileum 130. Sterilisation b.on laprotomy large haematoma of broad ligament formed by. Crisp clear information should be provided to the patient according to his needs c. Aldosterone release increased in a. Fatty acid oxidation helped by. Condition with increased ESR. a. Swell in hypotonic solution 123. Young lady with MCV 76m MCH 25 the type of anemia. Lateral 2/3 of thalamus 145. Urine for bile pigment 145. 3rd ventrical ki boundaries important hain. Gram -ve infection cause septic shock in which of the foll0wing tracts. Posterior wall is strengthened in its medial third by inguinal falx (conjoint tendon) b. Clioris b. Juandice in pregnany confirmed by. Glucose 6 phosphatase is absent in the muscles 124 . a. a. Esopagial atresia c. Brogenital diaphragm c. Labia majora 152. Fallopian tubes 116. a. The diagnostic lab test for the infection by streptococuss beta haemolyticus is. medial aspect of foot 142. T4 dec c. Alcohol b.112. Wet gangrene 158. Metaplasia 141. Glioma b. To prevent autolysis c. a. Pelvic nerves 138. Medial quadrant of breast drains into. occipitoanterior position . Ovarian artery c. L1 b.Thalamusd. ligamentum falvum d. Colon c. Type of reaction which occurs after diphtheria vaccine. Dopamine 172. loss of polarity and increasd nuclear to cytoplasmic ratio. a. Patient taking OCP and ATD still got pregnant. 3 % one drugg n other 2 % ANESTHTIC CORSES placenta dua to. In neonates spinal cord ends at? a. Toxic shock c. Adrenal androgens c. D. Which structure is likely to be damaged A. Sebacous glands are absent in. Stool culture onl e. Type 1 c. a. a. Ureter b. 10% increase risk of fetal talipes c. Aldosterone b. Condition with increased ESR. Should be breached if the patient authorizes 188. B. a. polycythemia 191. Right and left traigular ligament Falciform ligament . The cause of death in gas gangrene is due to . oxytocin. 1% Decrease chance of miscarriage b. The drug responsible. Type 4 179. Corner of lips b. Uterus b. Langerhans giant cells found in. Bacteremia b. a. Condyloma 185. Hydrocortisone 199. Baroreceptors b. a. Isoniazid 189. Golgi tendons b. Gastriesophageal fistula 183. Premalignant lesion on vulva a. Adrenal estrogen d. Facillliated 193. a. Supra spinous ligament 164. Rifampicinb.energy source. has extravasation of urine interna liliac artery ligated tostop bleeding which organ ruptured? a. Active transport d. Shouldn't inform the patient and the family c. Regarding the extent of Uterine tube a. Cervix c. Muscle spindles 177. Amniocentesis vs transcervical chorionic villous sympalling befre 14 weeks. a. Anterior spinous ligament b. The secret regarding the disease of a patient.Brocas areab. During a surgery a surgeon nick hepatoduodenal ligament. a. Pneumonia 196. AV node delay b. Right renal vein 169. Oxygen dissociation curve is shifted to the left when. ACTH effectively controls ? a. Sarcoidosis c. Should inform the patient and not the family b. Blood c. 1. a. Pneumonia 202. Compact bone is characterized by. Parasympathetics reduce heart rate by. Secondary cartiligineous joint. B. Myocarditis b . Portal vein Bile duct IVC Hepatic Duct Qs 3. Bacteremi b. a. Squamous cell CA b. Heinz bodies are found in A. Wegners granulomatosis d. lipid solubility c. Lacunae containing osteocytes 201. Smal bowel d.Cerebelluc. Sarum n coccyx b.she gives History of fever for 10 days ? What's the Diagnosis. Acidosis c. Renal agenesis b. Paramedian approach in spinal anesthesia must pierce on of ligaments. Decreasing action potential by Blocking Na+ channels 182. prolactin. Bare area of liver is limited by A. a. Vulvar nevib. a. Paget’s disease 186. interspinous ligament e. AFI index <2 possibily due to. Lateral attachment of urogenital diaphragm. Norepinephrine c. a.Bone marrow 165. a. prolactin and cortisol 175. a. Uterine artery b. Ant interventricular artery ke along jo vien drain karti uska pucha tha. Chemoreceptiors 181. rete ridges. Preganglionic sympathetic nerve fibers release . Type 2 d. Anal canal RADIOLOGY PAPER-B FEB 2014 Qs. verrucous CA 168. a. Uterine tube d. a. HPVb.Hypothalamus 192. Liver glycogen d.a. 198. A female presented with shock with history of abortion by dai . a.Pericarditis c. Descnding colon 180. A femal presented with history of car accident. a.Pleuritis 187.Sensory motor cortex e. An mcq about great cardiac vein. Urinary tract 163. a. Dysarthria is due to a lesion in. Pubic symphysis 169. Concentric lamelle b. a. In concentration b. Plasma cells 174. The structure responsible is. a. great cardiac vein tha ans 170. Ovaries 190. Alkalosis 200. Muscle glycogen b. Wegners granulomatosis d. The reason may be. Palms and soles c. Superficial perinepouchb. C. Which one of them is responsible when a muscle is stretched it leads to the relaxation of muscle. a. It extends from lateral end of uterus to medial end of ovary 201. Rupture of placenta 171.a. After 48 hours starvation. IVC b. Tuberculosis b. B. a. Obturator internus membrane c. C. IUCD 194.on laprotomy large haematoma of broad ligament. Cardiac ischemic response c. Potasium eflux through SA Node. Right kidney is not related to. Ischial tuberosity and pubic ra 178. During 2nd week which is appropriate test for Typhoid fever. Syphlis 176. Sarcoidosis c. L4 e. Biliray tract b. Slide showing simple columnar epithelium with indistinct smooth muscles and clefts. A woman with chest pain. ACTH b. Posterior logitudnal ligament c.worse on lying down . Should inform the relatives and not the family d. a. a. a. Syphlis 173. Antibodies are formed from. Fastest control of BP control is by. Oxytocin. Urinary bladder 198. Histopathology showing pleomorphism. Toxic shock c. A patient who is from a middle class family and has 3 children. Local anesthetic mechanism of action. a. Type 3 e. G6PD deficiency HS Sickle cell disease Qs 2. Temperature increases b. ACH b. The following combination of hormones regulate lactation. Langerhans giant cells found in. a. Tuberculosi b. Gall Bladder A. L3 d. a. culture only d. She has cervical dysplasia. Anaphylaxis b. Widal+Blood cultureb. T12 166. a. a. Ovarian artery c. Right Adrenal gland drains into. Deep perineal pouchc. The cause of death in gas gangrene is due to . C. Glans of penis 167. B. L2 c. Sphyincter urethrae is the content of. Vulval premalgnant lesion is. a. Muscle protein c. Triglycerides 195. Misscarriage rate is 1/2000 d. a. Produces bile Submucosa is present Mucosa is thrown in extensive folds Qs 4. Ovarian venous plexus 197. Main supply of head of humerus A. B. Mesonephric duct functional remnant or unit is A. Head of pancrease related anteriorly to all except . B. Capsule of left kidney prevents the spread of infection to A. B. E. Costocervical trunk Subscapular artery Aorta 1st part? Int thoracic artery? Qs 13. B. Intercostal spaces A.whats her peripheral blood picture right now A. B. Howeljowel bodies Thrombocytopenia Qs 16. Ant circumflex artery Post circumflex artery Arteries around rotator cuff Axillary artery Subscapular artery Qs 12. So better to give this topic a read from SNELLs or RJ Last Qs 8. E. E. Deep ring is A. C. B. B. B. D. A young 1yr old has hydrocele A. C. C. uterine artery (answer?) ovarian artery ? some nerves? Qs 21. 22. A pt got spleen rupture and she is going to spleenectomy. C. C. C. B. D. UMN lesion LMN lesion Hypothyroidsm Parkinson Hyptonia ( don’t rem if this option was given) Qs 7. C. Fascia transversalis Aponeurosis of transvers abdominus Aponuerosis of ext oblique Qs. B. T11 T7 L1 L3 Qs 18. Pendular knee jerk A. Infants spinal cord ends at A. B. During hysterectomy which structure is damaged A. D. D. Deep cervical artery is a branch of A. Portal HTN A. B. C. Duodenal ulcer which artery is involved A. C. B. Gastroduodenal artery Splenic artery Gastroepoploic artery Qs 23. Lies deep In ribs Lied on upper border of ribs? Lower border of ribs Qs 9. Other kidney Lesser sac Upper part of greater sac Lower part of greater sac Deep pelvis Qs 14. Proximal part of CBD is supplied by A. Metastatic Calcification of kidney most commonly due to A. C. B. C. D. C. Medial cord (answer)? Lateral cord Musculocutaneous nerve Median nerve? (don’t rem) Qs 6. B. Inf mesenteric artery Sup mesenteric artery Cystic artery Right hepatic art Left hep artery Qs 15. Hypervitaminosis D Hyperparathyroidism Hypercalcemia Qs 11. Epididymis Ductus deferens Vagina Uterus Qs 10. B. Scrotal swelling Peritoneal fluid accumulation Testis inflammation Qs 20.. E. Have three msucles in between Neurovascular bundle present Options were lengthy and tough. Btw L1 and L2 Lower border of L3 L2 Qs 17. D. C. B. Left colic (answer) Inf mesenteric Infepigastric Qs 19.C. D. C. D. D. Ligament teres Ligament venosum Qs 5. Neurovascular bundle A. B. Medial side of arm is supplied by A. C. C. C. Not from any MCQ book. Esophagogastric junction is at A. Lat ligament attached to lower epicondyle (ans?) Plantaris muscle?? Qs 27. D. Rectum is A. Related post to psoas bursa Has obtext inferiorly Related to femoral nerve? Qs 26. C. Rt suprarenal gland (ans) Left suprarenal gland Duodenum? Qs 24. C. B. C. Prim cartilaginous joint Sec cartilaginous joint Qs 42. Most imp anatomic functional lobe or largest subdivision of prostate is A. B. Intrinsic factor is secreted from A. S1 lesion A. C. Esophagus constricted at A. Aortic Pulmonary Tricuspid Mitral Coronary septum?? Don’t rem the option and neither understand the qs A. Ext pollicis and abdpollicis Dnt rem options learn it from snell review Qs 37. Left colic nodes Inf mesenteric nodes (ans?) Sup mesenteric nodes Qs 25. C. Median lobe Peripheral lobe Transitional lobe (ans???) ? Qs 38. Female pelvis with short AP diameter and long transverse diameter is A. C. Post interventricular artery supplies A. Most medial nucleus of cerebellum Qs 43. C. Rectum nerve supply by A. B. Uterine tubes Ovaries Mesentry of abdomen Qs. Pain of vagina mediated medially due to A. Hip joint is A. B. Beta globulin (ans) Albumin Qs 45. D. Pectoral LN Qs 35. C. Infhypogastric Sup hypogastric Pelvic splanchinic Qs 41. D. Lateral side of breast drains into A. B. C. Fastgial Dentate Qs 33. B. Gastric fundus Gastric antrum Qs 32. Straight structure? Has haustrations circular folds? Post tp rectum is S2S4S5?? Qs 28. B. Defect in interventricular septum causes or affects which valve most A. B. Sickle cell disease is a defect in A. B. B. Obturator nerve (ans) Femoral nerve Sciatic nerve Qs 39. Femur bone A. B. Infepigastric artery & deep inguinal ring (qs was only this with no extra words or hint) A. B. AV bundle A. B. Where crossed by arch of aorta (answer) Post mediastinum + left ventricle Rt bronchus Qs 36. B. Sigmoid and descending colon drain into A. Symphysis pubis is A. Common peroneal nerve damage scenario in a footballer and it was asked where it is damaged . Plateploid Gynaecoid Anthropoid Qs 40. C. E. 44. Only neuro connection btw right atria &ventricle (ans?) Present in right atria Qs 31. Loss of ankle jerk Loss of sensation on medial side Qs 30. B. B. B. C. B. Both right and left ventricle SA node Rt atria Some other options abt its aneurysm? Qs 34. Site of radial artery pulse is A. Most common site of ectopic pregnancy A. C. B. Medial (ans?) Lateral Qs 29. B.A. B. Sup salivary nucleus Inf salivary nucleus Auriculotemp nerve Lesser petrosal Greater petrosal A. B. C. E. Post and inf spine?? Nerve leave sup??? A. Amykoidosis all except Qs 53. B. B. Ileocecal valve Ileocecal sphincter (ans) Qs 48. Left supra renal gland A. Vertebrae A. B. B. Superficial inguinal lymph nodes doesn’t drain A.A. C. C. B. C. Human gonadotropins hormone main function is to A. B. Facial nerve Accessory nerve Qs 55. C. C. Post curvature is lost in an infant. D. Lateral pterygoid Medial pterygoid Temporalis Buccinators Qs 60. Post belly of digastric is supplied by Qs 65. Child has dark urine Child has dark ears No failure to survive/failure to survive (ans??) Qs 63. B. B. D. B. Qs. B. Not present in heart (ans??) Nephrotic syndrome Qs 64. B. Parasympathetic supply to submandibular gland is from A. C. Maxillary nerve (ans) Supraorbital nerve Qs 58. Tyrosine (ans) Tyrmaine Qs 62. Right testis cancer scenario and asked abt its drainage A. Alkaptonuria all except: A. Reflux of cecum is prevented by A. Neck of fibular (ans) Head of fibular Ant to tibia Qs 46. Cervical part??? Don’t remember complete options. LGB Geniculocalcarine tract (ans?) . B. Right supra renal gland (ans) Left supra renal gland Rt kidney Left kidney A. B. B. Prematurity (ans) Infections Qs 54. D. Phenylalanine converts into A. B. B. But thisqs was NOT FROM ANY BCQ BOOK. Posteriorly lesser sac has all except? A. D. B. D. Int iliac nodes(ans) Para aortic nodes Qs 50. B. Annulus fibrosis A. Testis (ans) Scrotum? Skin of gonads Vagina Qs 51. Separated from left kidney by perirenal fascia Drains in left renal vein (ans?) Qs 52. Most common cause of PDA A. Kyphosis Scoliosis Qs 59. No of divisions of lower airways?? A. Atherosclerosis mainly compress or is present in A. This condition is called A. Attached med and lat ligaments/? No sensory supply?? Qs 61. B. Para-aortic lymph node (ans) Int iliac lymph nodes Qs 49. C. Which muscle is defected A. Abdominal aorta (don’t rem if it was given in options) Sup mesenteric artery (ans) Qs 57. B. Prostate some scenario was given drains in A. Which goes to cortex some visual pathway qs A. 17 21 (ans???) 23 26?? C. 56. Deep pain around orbit and nose Qs 47. A girl has problem in opening mouth. B. Maintain pregnancy (ans?) Enhance breast Ovulation Qs 66. B. Intrauterine midtrimester hematopoiesis occurs in A. Adenoma has Glandular cells ( answer) 7. St. Ovarian cancer marker…CA 125 10. Right renal vein A. A 40 year old teacher while writing on blackboard is suffering from shaking of his hands. More in male Lymphocyte depletion has some prognosis?? Qs 76. B. Superior thoracic aperture 6. Hodgkin disease A. His cousin has same problem…Pipe lead rigidity and other sign not reported…all other signs are unremarkable…Drug should be given is ROPINIROLE. B. HIV drug causes pacreatitis and peripheral neuropathy is  Didanosine (Answer) 5. D. Premalignant condition is caused by all except: A. Pagets disease Cirrhosis of liver AIDS Osteoporosis?? Some breast disease too 4. Parietal pleura?? A. Autosomal dominant is A. B.surgeon needs to go to endothoracic fascia by reaching A. B. 2. B. Homonymous hemianopia caused by Medicine Online 28th August. PCA MCA A. base of the nail 3.. B. E. Anterior SURFACE of the heart is formed by Right Ventrilcle (anwser) ( remember the question was about surface NOT about border…anterior border of heart is formed by Right atrium) 8. Optic tract Optic chiasma Qs 72. C. St. Pulmonary TB A. A scnerio of 5 hydroxyindoleacetic acid ……Carcinoid tumor 11. A young man having dysphagia. Corneum (ANSWER) B. ESR is raised in A. Basalis C. some relation options Drains in IVC directly (I chose that) Qs 80. horness of voice …. B. Left renal vein A girl with periods In infections (ans) Qs 68. Most in post upper lobe (ans)? Foul smelling sputum Qs 78.2014 with ANSWERS !! A. Stab wound injury in intercostal space.Qs 67. Liver (ans) Spleen Qs 73. Anaphylactic shock comparison with Hypovolumic shock has  Increase cardiac output (answer) Qs 75. A. blood becomes turbulent due to  Decrease blood viscosity (ANSWER) Qs 74. Area of visual field supplied by A. Adult derivative of umblical vein A. B.case of medullary cell carcinoma…marker is CALCITONIN . Pancrease related anteriorly to all except: (don’t rem if it had except or not) Qs 70. Central and peripheral chemoreceptors are sensitive to A. D. B. It was not abt venous drainage. C. D. C. Right atrium of heart. Which has got soft keratin with having no nucleus A. B. spinosum D. St. Don’t remember qs exactly Marked drains in IVC directly Qs 81. In anemia . A scenario of VMA found in yrine …Pheochrocytoma (ANSWER) 9. B. Lesser sac Spleen Splenic vein SMA Qs 71. Ligamentumvenosum (ans) Ligarteriosum Qs 82. no options remembered Ext intercostal and internal intercostal muscles Neurovascular bundle?? Some weird options not given in any MCQ book. Dec P02 Inc CO2 IncPh 1. Hairs follicle E. Abt structure of right atrium A. Marfan syndrome Qs 77. Sinus venosus rt atria (ans) b Coronry sinus drains in Qs 69. B. basalis & st. Qs 79. C. Myoglobon (Answer) B. Turner syndrome  16. A pateient having CVP line in subclavian vein & on parental nutrition. B Increased gastric secretion. E Triglycerides in fat cells. Fragile X chromosome ( Answer) C. B Tubocurarine. Muscle carrying capacity with hemoglobin is  A. B Partial AV heart block. D Significant muscle fasciculations during onset of block. Creatinine is important in assessing renal failure…it arises maily from A. True about cerebellum A. E Baclofen. Ectopic urethera C. Key: c Q3 Half life of a drug may be helpful to determine: A Dosage schedule of the drug. Bending the stereocilia away from the kinocilia C. a malignant tumor that can cause serious complications E. E Increased urinary frequency. Resting tremors occur after injury C. C Doubling the loading dose but maintaining the infusion rate. D Tripling the rate of infusion. C Poorly sustained tetanic tension. Cytochome P450 c. Rapid ascent in an elevator D. Endoderm (answer) ….2ND EDITIOIN) B. Key: e Q5 Which of the following drugs given preoperatively is used to prevent postoperative pain caused by succinylcholine? A Dantrolene. Connected to medulla through middle cerebellar peduncle B. Increase carbohydrate consumption will increase the demand of  A. Increase GFR due to Dilation of afferent arterioles 20.Rabbia ali BCQ page 140 27. Riboflavin C.Rabbia ali mcqs PAGE 443. Key: b Q7 A 3 year old child has been admitted to emergency with suspicious of atropine overdose as there are: A Abdominal cramps. Mesoderm C. Ectoderm C. E Stimulation of autonomic ganglia. Tropomyoglobin 19. D Triazolam.. Skeletal muscle ( answer) 18.are affected…Substantia Nigra 14. which of the following is the most important reason for this step? A Familial tremor. Thaimine (Answer) B. a tumor that can metastasize D. E Quadripling the rate of infusion. D Angina pectoris. Key: a Q10 Characteristics of non depolarizing neuromuscular blockade include which of the following? A Block of post tetanic potentiation. B Histamine blocking action. NADPH cytochome P450 reductase b. The drug of chaoice for this organism is  A. E Time to get the steady state.Blood CP shows cluster of gram +ve cocci.5 cm (answer) 17. Vancomycin C. A new born baby is having complain of passing urine from umbilicus to urachus… A. Primordial germ cell of yolk sac are derived from A. E Occur at the same rate in adults and the new born. Lateral plate 26. C Usually leads to inactivation of the drug. C Increased cardiac rate. The most common cause of mental retardation is A. Protooncogene is converted to oncogene by  PONT mutation 15.. He develop fever. C Diazepam. Key: e Q4 Epinephrine does not cause increase concentrations of: A Glucose in blood. Slow ascent in an elevator E. ectopic ureter B. Piperacillin + Tanzobactam 24. D Papillary constriction. Flare due to arteriolar dilation is due to AXONAL REFLEX 22. B Level of absorption. Melanocytes are derived from A. The length of right bronchus is  2. B Includes activity of cytochrom P450. Totipotent cells 21. Kidney D. Key: c . B Meprobamate. Endoderm B. E Supraventricular tachycardias. D Lidocaine. Brain C. Monoxygenases Key C Q2 Phase II reactions of a drug biotransformation: A Decreases its water solubility. Liver B. Neural crest cells ( Answer) D. Scnerio about tremors and rigidity…. B Doubling the rate of infusion only. C Mild hypertension. Lies below tentorium cerebellum (ANSWER) D. Key: b Q6 The steady state concentration of a drug can be double by: A Doubling the both rate of infusion and concentration of drug. Amoxicillin + Clavulanic acid (ANSWER…. mass of mature disorganized tissue indigenous to that organ (Answer) B.qs # 246…. Rotating the head to the right (ANSWER MOCK Test With Key Q1 All of the following result in detoxification of drugs except: a. Hamartoma is defined as ? A. D cAMP in heart muscle. D Does not include acetylation. Troponin C. Cri du chat syndrome B. E Zolpidum. Key: c Q8 A 50 year old patient is having propranolol due to his cardiac problems but his physician now wants to stop this drug. Imepenam E. Hyperpolarization of the hair cells B. Actin B. Edward syndrome  D. D Rate of absorption through GIT. Ceftazidim D. Key: b Q9 Which of the following drugs when used for prolonged period in the maintenance treatment of tonic clonic seizures can lead to increased metabolism of warfarin like drugs? A Phenobarbital. B Lactate in blood. C Chlordiazepoxide. Which of the following would produce maximum excitation of the hair cells in the right horizontal semicircular canal? A.12. persistent allantois (ANSWER) 23. Cytochome Oxidase d. ectopic rest of normal tissue C. Ectoderm B. C Distribution into different body systems.. Pharyngeal arch E. C Free fatty acids in blood. Niacin 13. Ataxia not occur after this lesion  25. E Infusion related adverse effects.25 dihydroxycholecalceferol D) erythropoeitin Answer B 35 ) enzyme not stable at acidic ph A) trypsin B) chymotrypsin C) pepsin D) carboxypeptidase Answer C 36)Iron is stored in the body in the following except A) Spleen B) reticuloendothelial system C) gall bladder D) bone marrow Answer C 37) absolute refractory period in the heart A) corresponds to the duration of relaxation B) last till half of cardiac contraction C) shorter than refractory periods in skeletal muscle D) last till cardiac contraction Answer D 38) increased vagal tone causes A) hypertension . excessive bronchial secretion C Miosis. Key: a Q12 Which of the following is a prophylactic antiasthmatic agent that stabilizes mast cells: A Ipratropium. C Having a short elimination half-life. D Not effective in lyme disease. E Vit. Key: d Q13 A 30 year old male suffering from cerebral edema will be best treated with: A Furosemide. qualitatively abnormal reaction to a drug D Quantitatively exaggerated response E A type of drug agonism Ans:C 31)papez circuit in limbic system involve A) anterior thalamic nuclei B) pulvinar nuclei C) anterior hypothalamic nuclei D) interlaminar nuclei Answer A 32) site of rbc formation in adult is A) long bones B) flat bone C) liver D) spleen Answer B 33) botulinum toxin involve all of the following except A) neuromuscular junction B) post ganglionic parasympathetic ganglia C) peripheral ganglia D) CNS Answer D 34) which of the following hormone is not secreted by kidney A) renin B) angiotensin 1 C) 1. B Metabolized by hepatic enzyme. D Sucrulfate. E Acetazolamide. an aminoglycoside is: A Least nephrotoxic. Key: c Q17 Doxycycline is: A Bactericidal. Key: e Q18 Neomycin. D Cyclophosphamide. B Tamoxifen. D Cromoyln. C Ethacrynic acid. bradycardia D Weakness of all skeletal muscles Key D Q22 Which of the following drugs is both a muscarinic and nicotinic blocker? A Atropine B Benztropine C Hexamethonium D Succinylcholine E Tubocuraine Key B Q23 Epinephrine is used in the treatment of all of the following disorders EXCEPT: A Bronchospasm B Anaphylactic shock C Cardiac arrhythmias D Open-angle glaucoma E All of above Key C Q24 Valproate is very effective against: A Absence seizures B Myoclonic seizures C Generalized tonic-clonic seizures D All of the above Key D Q25 The mechanism of both topiramate and felbamate action is: A Reduction of excitatory glutamatergic neurotransmission B Inhibition of voltage sensitive Na channels C Potentiation of GABAergic neuronal transmission D All of the above E none of above Key D Q26 The pathophysiologic basis for antiparkinsonism therapy is: A A selective loss of dopaminergic neurons B The loss of some cholinergic neurons C The loss of the GABAergic cells D The loss of glutamatergic neurons E all of above Key A Q27 A long-acting drug against both absence and myoclonic seizures is: A Primidone B Carbamazepine C Clonazepam D Phenytoin E Ethionamide Key C Q28 The rate of secretion of thyrotropin is controlled by: A The amount of iodine in the thyroid gland B The amount of thyroid hormones in the thyroid gland C The concentration of thyroid hormones in blood D The concentration of catecholamines in blood E The concentration of cortisol in blood Key C Q29 If an agonist can produce maximal effects and has high efficacy it’s called: A Partial agonist B Antagonist C Agonist-antagonist D Full agonist E non competitive antagonist Key D Q30 Idiosyncratic reaction of a drug is: A A type of hypersensitivity reaction B A type of drug antagonism C Unpredictable. C Doxorubicin. D Hypotension. C Terbutaline. C Used in hepatic coma. B Prednisone. D Plasminogen. Key: d Q16 The toxic effect of amphotericin B which definitely require reduction of dose is: A Myelosuppression.C Urokinase. B Not excreted in the feces. B Amiloride. temperature B Muscle spindles C Motor function D Touch. C Renal Toxicity. E Omeprazole. which is more lipidsoluble: A Bupivacaine B Lidocaine C Mepivacaine D Procaine E cocaine key: A Q20 Indicate the function.pylori. E An adjunct in treatment of tuberculosis. D Mannitol. Key: d Q14 A 45 year old patient developed severe thrombocytopenia due to unfractionated heparin so the best alternative anticoagulant will be: A Lepirudin. E Fluorouracil. which the last to be blocked in anesthesia: A Pain. diarrhea B Increased salivation. B Levodopa. B Abciximab. K Key: a Q15 Acute hemorrhage cystitis is a common toxic effect seen with: A Vincristine. inherent. B Hepatitis. E Aminophyllin. pressure E All block at same time Key: C Q21 The symptoms of excessive stimulation of muscarinic receptors include all of the following EXCEPT: A Abdominal cramps.Q11 A 35 year old lady who is on anticancer therapy is now suffering from severe nausea and vomiting which of the following drugs is having most effective antiemetic actionsin this case? A Dexamethasone. E Not as effective as tetracycline against H. Key: c Q19 Indicate the local anesthetic. D A drug of choice in Rockey Mountain Spotted fever. C Apomorphine. B) tachycardia C) bradycardia D) increase in cardiac out put Answer C 39) the extrapyramidal system is not concerned with A) stretch reflex B) righting reflex C) spasticity D) sensation of viscera Answer D 40) epiphyseal closure is regulated by A) calcitonin B) somatomedians C) 1,25 dihydroxycholecalceferol D) thyroxine Answer B 41)..A 43 years old Diabetic suffering from Stroke,His desire for drinking water is obtunded since he recoverd.Which of the following is appropriate in this patient. a.watershad Infarcts of Post Hypothalamus. b.Lesions involving posterior commuinating artery c.Lesions involving Middle ceebral artery d.Lesions involving anterior Communicating artery. e.0.9%NaCl Infusion therapy Intravenously (Answer is D………………lesion involving Anterior communicating artery Inhibits or Obtund the intake of drinking water since it supplies area concered with Thirst……Ganong Page no 310…..24th Ed) 42.All of the following events in Humans are initiated by Heat Except.a.Anorexia b.Increase in Respiration c.Cutaneous Vasodilatation d.Apathy e.Incesed levels of Epinephrine in Blood (Ans is E………….Referance Ganong Page No 319 24th Ed) Q No 43.The insensible loss of water in humans is a.10mL/h b.20mL/h c.30mL/h d.40mL/h e.50mL/h (Answer is E…………reference is Ganong Pg no 318 24th ed) Q No 44.Most Common metabolic finding in Humans with Hypothalmaic disease is. a.Anorexia b.Bulemia c.Diabetes insipidus d.Precocious puberty e.Hypogonadism (Ans is D……………Ref Ganong page no 316 24th Ed) Q No 45.A drug abuser of Heroin is presented with Hypertension,What is appropriate. a.His Serum TSH levels are normal to decreased. b.His serum TSH levels are normal to increased c.His serum T3 levels are decreased d.His serum T3 levels are increased e.He is clinically Euthyroid (Ans is E…………..Ref Ganong page No 344 24th Ed) 46.Delta waves in ECG are charecterstically found in a.AV junctional rhythum b.Wolf- Parkinson-white syndrome c.Lown-ganong-Levine syndrome d.Brugada Sybdrome e.Hyperkelemia (Answer is B……… Reference essential revision Notes Karla 3rd ed …) 47. Metabolic cause of Proarrythmic Abnormal repolarization (ST-T changes) is. a.Hpercalcemia b.Hypernatremia C.Hyperthermia d.Hyperkaelemia e.Hpothyroidism (Ans is E……………Reference essential revision Notes Karla page no 29 3rd ed) 48.Gastrin secretion is inhibited by all Except a.increased Serum Somatostatin concentration b.increased Luminal Somatostatin concentration c.Incresed Serum (VIP) vasoactive intestinal peptide concentration d.Increased Luminal gastric acid concentration. e.Increased serum concentration of Glucagon (Answer is A…………reference Ganong Page No 471 24th Ed) 49.Regrading Secretion of Renin,which is most appropriate. a.Intra renal Baroreceptors mechanism Increases it b.Its secretion is Directly proportional to Na and Cl entering DCT c.Its secretions is Inversely proportional to Na and Cl entering DCT. d.Thirst increases it e.Thirst decreases it. (Ans is C………………Reference Ganong page no 705 .24th Ed) 50.Regarding Pressures in blood vessles.which one is appropriate a.Pressure in carotid is higher compared with pressures in Brachials. b Gravity has no effect in Pressure dynamics in vessels. c.Presure in any vessel below heart is increased d.Standing decreases cerebral pressure. e.Pressures in any vessel below heart is Decreased. (Ans is C………….Page No 578 Ganong…….effect of gravity……24th Ed) 51.Paroxysmal Nocturnal Hemoglobinuria is best diagnosed by Which of the following. A.Blood Compelte picture B.HAM acidified serum Test C.Sugar –Water Test. D.Flow Cytometery E.Hypocomplementemia (Ans is D………….Flow cytometery………….Refernce Goljan page no 306 4th Ed) 52.Which of the following has Increase MCHC. a.Iron deficiency anemia B.Sickle cell disease c.Heridatery Spherocytosis d.Heriditary Eliptocytosis e.Immune Hemolytic anemia. (Ans is D……….Heriditary Spherocytosis……….Refrence Goljan Page no 304 4th Ed) 53.Regarding Hypersegmented neutrophils which is appropriate. A.Associated with Iron deficency anemia B.They are used charecterstically as marker of Pernicious anemia. C.Almost always present with pancytopenia. D.Used to describe when Neutrophils have more than five nuclear lobes. E.Commonly seen in acute inflammatory response. (Ans is D………Reference Goljan Pathology …..page no 299 4th Ed) 54.Reticulocytosis is Usually evident in acute Hemorrhage. a.3-4 hours after the acute event. b.5-10 hours after the acute event c.1-2 days after the acute event D.5-7 days after the acute event E.14-28 days after the acute event. (Ans is D…….Reference Goljan Pathology page no 301 4th Edition) 55.Aplastic Crisis of Sickle cell Anemia is characterized by a.Lymphocytosis B.Lymphoctyopenia c.No change in Red blood cell count D.Reticulocytosis E.Reticulocytopenia (Ans is E……………Referenace Goljan page no 309 4th ed) 56.Intravascular Hemolysis is Best characterized by. a.Increased Unconjugated bilirubin levels in blood. b.Increased unconjugated bilirubin levels in Urine c.Normal Levels of Unconjugated bilirubin in blood d.No change in urinary execration of Hemoglobin. e.Decreased Unconjugated Bilirubin levels in blood (Ans is A…………..Reference Goljan 4th Ed) 57.Sterile Pyuria is usually diagnose by presence of which of the following in Urine A.Myoglobin b.hemoglobin c.urobilinogen d.Leukocyte esterase e.Bens jones proteins (Ans is D………………..Ref Goljan…………..page no 505 4th edition) 58.Hypoalbunemia in Nephrotic syndrome accelerates the synthesis of which of the following. a.Albumin itself as a replenishment measure. b.Antithrombin III c.Erythropoietin d.Cholestrol e.acute phase proteins (Ans is D……..reference Goljan Page No 512…….4th Edition) 59.A 7 years old girl is noticed by her mother for a palpable lesions on her Buttocks that does not blanch when pressure is applied, along with pain in both knees and small joints of hands. She had an episode of Sore throat 1 week earlier.What could be the Cause. a.A sequelae of staphylococcal Infection. b.Infective arthritis c.Churg-Strauss Syndrome d.Reiters Syndrome E.Henoch Schonlein Purpura. (Ans is E… Henoch Schonlein Purpura………..most common vasculitis in children Reference is Goljan Pathology Page Number 234 4th ed) 60.Regarding Euthyroid Sick syndrome which is most appropriate. a.it is a associated malignancy of Adrenal Gland.. B.serum TSH levels is specifically remain normal to Increased. C. serum TSH levels is specifically remain normal to Decreased. D.Symptoms usually require Thyroidectomy. e.Levothyroxine is drug of choice in the treatment options (Ans is C ……..Reference Goljan Pathology………page No 602 4th Edition) 61.Bithermal antibodies are found in which of the following conditions a.Immune hemolytic anemia b.Paroxsymal nocturnal hemoglobinuria c.Heriditary Eilptocytosis d.Sickle cell disease. e.Paroxysmal cold hemoglobinuria (Ans is E………Ref is Goljan Page No 307 4th Ed) PHARMACOLOGY BCQs 62.A patient of epilepsy is using Phenytoin for the last 1 year,His Blood complete picture shows Megaloblastic anemia.it is due to.fact that. a.it decreases the vit B 12 stores in body. B.it increases the osmotic volume of RBCs. C.it increases the folic acid consumption in the body. d.it inhibits the intestinal Conjugase e.it increases the activity of luminal Conjugases (Ans is D………………Reference Katzung review) 63.Regarding Sulfasalzine which of the following is appropriate. a.5-ASA is the active component which is absorbed to have systemic anti-inflamatory activity. b.can cause Ireversible Azoospermia. C.can safely be given in patients with Aspirin allergy. D.its metabolisim is impotant clinically for Acetylator status. E.Black females are at risk of Hemolysis. (Ans is D…..Ref is from Katzung and Davidson,Sulfasalazine is composed of 5-ASA and sulfapyridine which are cleaved by bacterial enzymes in the bowl.5-ASA is not absorbed but sulfapyridine is .5-ASA is active in IBD,whereas sulphapyridine is active in Rheumatoid disorders.Reversible Azoospermia is a side effect.in patient with aspirin allergy will react to sulfasalazine.Black males are at risk of Hemolysis owing to G6PD deficiency, which is X linked. Acetylator status is important Clinically in the metabolism.) 64.Which of the following Drug increases the formation of Hb F. a.Busulfan b.Cyclophosphamide C.Hydroxyurea D.Tyrosine kinase inhibitors E.5-flourouracil. (Ans is C…………………reference is Goljan) 65.Pneumonia in Patients with sickle cell disease can be prevented by Prophylactic use of which of the following agent. a.Amoxicillin calvulnate. B.Gentamicin c.Cefotaxime d.Penicillin e.Cefuroxime (Ans is D…………Katzung review,Goljan pathology ) 66.Non Ionized drugs are a.lipid soluble more than water soluble b.water soluble more than lipid soluble c.Neither soluble in water Nor in Lipids d.are strong acids e.are strong bases (Ans is A…………..reference Kaplan Pharma) 67. Chronic,accelerated and blast phases of CML are treated by which of the following agent. a.Dasatinib b.Nilotiib c.Eculizumab d.Bosutinib E.Imitinib (Ans is E…………Reference is Medscape) Tyrosine kinase inhibitors used in treatment of CML • Imatinib mesylate (Gleevec): For chronic, accelerated, and blastic phases; standard treatment of choice • Dasatinib (Sprycel): For chronic phase • Nilotinib (Tasigna): For chronic phase • Bosutinib (Bosulif): For chronic, accelerated, and blast phases • Ponatinib (Iclusig): For chronic or blast phase T315I positive cases, or in appropriate patients in whom no other TKI therapy is tolerated or indicated 67) All are motor nerves except: A : Accessory B : Abducent C : Trochlear D : Trigeminal Ans: D 68) Middle meningeal artery is transmitted through: A : Foramen rotundum B : Foramen ovale C : Foramen spinosum D : Foramen lacerum Ans: C 69) pleural tapping in mid axillary line, muscle not pierced is : A : Int. intercostal B : Ext. intercostal C : Innermost intercostal D : Transversus thoracis Ans: D 70|) In epidural Lumbar puncture, structures pierced A/E : A : Post. longitudinal ligament B : Ligamentous flavum C : Interspinous Lig. D : Supraspinous Lig. Ans: A 71) Fibrous Joint A/E: A : Skull Sutures B : Tooth Socket C : Inf. tibiofibular D : Symphisis pubis Ans: D 72) SUBSTANCES THAT DO NOT CROSS THE PLACENTAL MEMBRANE ARE A/E : A : IgG B : IgA C : IgM D : IgE Ans: A Q.73 : Double barr body is seen in : A : Turners B : Klinefelters C : XXX D : Downs Ans: C Q.74 : Which of the following disappear in umbilical cord: A : Left art. B : Left vein C : Right art. D : Rt. vein Ans: D Q.75 : Implantation occurs at the stage of : A : Zygote B : Morula C : Blastocyst D : Primary villi Ans: C Q.76 : A midline cleft lip is due to failure of fusion of: A : Maxillary processes B : Medial nasal processes C : Medial & lateral nasal process D : Medial nasal & maxillary process Ans: B Q.77 : which of the following is a feature of Y chromosome ? A : Acrocentric B : Telocentric C : Submetacentric D : Metacentric Ans: A Q.78 : All of the foll. are pneumatic bones Except : A : Maxilla B : Parietal C : Ethmoid D : Mastoid Ans: B Q.79 : All of the following are composite muscle Except: A : Pectineus B : Rectus femoris C : Adductor magnus D : Biceps femoris Ans: B Q.80 : The skin overlying the region where a venous cut down is made to access the great saphenous vein is supplied by : A : Femoral N. B : Sural N. C : Tibial N. D : Superficial Peroneal N. Ans: A Q.81 : Porto systemic shunt is not seen in : A : Liver B : Spleen C : Anorectum D : Gastro esophageal Ans: B Q.82 : aortic opening transmits A/E : A : aorta B : thoracic duct C : vagus nerves D : azygos vein Ans: C Q.83 : The urogenital diaphragm is formed by the following structures except : A : Deep transverse perineal muscle B : Perineal membrane C : Sphincter urethrae D : Colles fascia Ans: D Q.84 : The following structures pass posterior to the flexor retinaculum except : A : Flexor digitorum superficialis tendons B : Median nerve C : Ulnar nerve D : Flexor pollicis longus tendon Ans: Cs 85)Pain of ovarian carcinoma is referred to a) Back of thigh b) Gluteal region c) Anterior surface of thigh d) Medial surface of thigh Ans: b 86)Causes of primary amenorrhea are A/E a) Rokitansky syndrome b) Sheehan syndrome c) Kallman syndrome d) Turner syndrome Ans: b 87) Which of the following treatments for menorrhagia is not supported by evidence? a) Tranexemic acid b) Ethamsylate c) Combined OCP d) Progesterone Ans:b 87)Lady undergoes total radical hysterectomy for Stage Ib Ca Cervix. It is found that Ca extends to lower part of body of uterus and upper part of cervix. Next step of management a) Chemotherapy b) Radiotherapy c) Chemoradiotherapy d) Follow-up Ans:c 88)Ideal contraceptive for lactating mothers a) Barrier method b) Combined OC c) Lactational amenorrhoea d) Progesterone only pills Ans:d 89) A pregnant lady had no complaints but mild cervical lymphadenopathy in 1st trimester. She was prescribed spiramycin but she was non compliant. Baby was born with hydrocephalus & intracerebral calcification. Which of these is likely cause a) Toxoplasmosis b) CMV c) Cryptococcu d) Rubella Ans:a 90) The presence of increased levels of which of the following in amniotic fluid is an indicator of open neural tube defect in the fetus? a) Phosphatidylesterase b) Acetylcholinesterase c) Pseudocholinesteras d) Butyrylcholinesterase Ans:b 91) Vaginal delivery can be allowed in A/E a) Monochorionic monoamniotic twins b) Mento-anterior c) Extended breech d) Dichorionic twins – First vertex, second breech Ans:a 92)A lady with placenta previa delivered a baby. She had excessive bleeding. After resuscitation most likely complication is a) Galactorrhoea b) Diabetes insipidus c) Loss of menstruation d) Cushing’s syndrome Ans:c 93) Which organ is the primary site of hematopoiesis in the fetus before midpregnancy ? A) Bone B) Liver C) Spleen D) Lung Ans:b 94) Choline esterase in fetus most specific for ? A) Open spina bifida B) Gastrochisis C) Exomphalos Ans:a 95) Perinatal transmission of Hepatitis B is maximum when infection in mother occurs in ? A) At implantation B) 1st trimester C) 2nd trimester D) 3rd trimester Ans:d 96) The shape of cervical canal leading to preterm delivery is ? A) T shaped shaped Ans:c B) Y shaped C) U 97) In Bishop's score all are included except ? Effacement of Cervix A) B) Dilatation of Cervix C) Station of head D) Interspinous diameter Ans:d 98) External cephalic version is contraindicated in ? A) Anemia B) Flexed Breech C) PIH Ans:c 99) An ovarian cyst is identified in the immediate postpartum period. What should be the timing of surgery ? A) 2 weeks B) 6 weeks C) 8 weeks D) Immediately Ans:d 100) The type of pelvis which is associated with increased incidence of 'face to pubis' delivery is ? A) Android pelvis Gynaecoid pelvi B) Anthropoid pelvis D) Platypelloid pelvis C) Ans:b M0ck test 2015 1.Lady with infertility with bilateral tubal block at cornua. Best method ofmanagement is : Laparoscopy & Hysteroscopy Hydrotubation IVF Tuboplasty Answer : A; Laparoscopy is done for proper assessment & to exclude active infection or TB. 2Women with postmenopausal bleeding need endometrial sampling if endometrial on US is thicker than 1mm 2mm 5mm 8mm 10mm Answer : C ; endometrial thickness in menopause > 5 mm is suspicious for hyperplasia 3. Which of the following change in puberty is influenced by the estrogen: Growth of the acinar buds of the breast Epiphyseal fusion Proliferatve phase All of the above None of the above Answer : D   : Estrogen is critical for epiphyseal fusion in both young men and women .Proliferative phase also continues under the effect of ovarian estrogen produced by thematurating follicles till ovulation occurs   . Estrogen also can affects both ductal andglandular “ acinar “ system of breast . 4. Sub urethral diverticula may occur as a sequelae to infection of: Bartholin’s gland Skene’s gland Clitoral gland Vulvovaginal gland Answer : B   ; Periurethral glands (Skene's glands) are tubuloalveolar structures along thedorsolateral aspect that drain into the distal two thirds of the urethra. Repeated infectionand obstruction of these glands lead to formation of suburethral cysts or abscesses thatcan rupture into the urethral lumen. 5.any loss or abnormality of psychological , physiological or anatomical structure or function is ? a. impairment b. disability c. handicap d. disease 6.a data is arranged as satisfied , very satisfied , dissatisfied . this is a ? D) O shaped a. nominal data b. ordinal data c. interval data d. ratio it is mostly due to subclinical infection c. Suicide 13.40 year old female came to the OPD with complaints of with depressed mood. Tumour calcification can be detected by an ultrasound Correct answer   : B. Vasodilatation C. Thalamus B.Muscles supplied by anterior division of mandibular nerve are all except? a. These symptoms appeared after loss in a business deal 1 year Macrophages Trigeminal ganglion 19. Antidepressants should be given based on the side B. due to? Staphylococcus aureus C. Individuals with sporadic retinoblastoma do not pass effect profile their genes to their children C. Temporalis 8. Anterior cortex in the gram stained specimen of the discharge.False about Argyll Robertson Pupil is? A. Visual evoked response Electrooculogram Correct answer   : D.Areas involved in posterior cerebral artery infarct are all complaints of pain and discharge from left ear.Arden index is used for interpretation of? ElectroretinogramVisual field charting D. Combination therapy with 2 or more antidepressants D. Which is correct regarding the management? 20. Bordetella pertussis monophosphate shunt Correct answer   : C. Maxilla Correct answer   : D.Draughtsman appearance is seen with colonies of:Pneumococcus Streptococcus Staphylococcus None of the above ago. Masseter positive Kernig’s sign. Nosocomial infection C.Most common cause of premature death in except? Retina Vitreous Sclera Uvea schizophrenia is? Drug toxicity Suicide B.All of the following are involved in endophthalmitis 12. spread of epidemic is influenced by it A. sharps c. Pain D. herd structure is constant b. Accommodation reflex normal . Antidepressants should be given scan based on the side effect profile The history is characteristic of depressive disorder. No treatment is required as it is due to loss in a business C.Which of the following does not have polysaccharide capsule related antigen antibody response? Haemophilus influenza B.Not an action of bradykinin? a. A. 21. insomnia.herd immunity all are true except ? 15. It is required for transamination reactions C. Homicide D. 22. loss of appetite and lack of interest in surroundings for the past 1 year.Which of the following is not a pneumatic bone? A.Streptococcus pneumonia B. The treatment should be based pigmentosa on the side effect profile of the drugs. Bronchodilation B. Anterior cortex 10. all can be incenirated except ? 14. Sphenoid C.Bitemporal hemianopia is a characteristic feature of? Antidepressant drugs are to be given. But there are no Glaucomab) Optic neuritis c) Pituitary tumour d) Retinitis ideal antidepressants. Frontal bone B. Choroid plexus D. can be aquired by immunization d. D. cytotoxic waste b. Neisseria meningitides C. It is a coenzyme for alpha ketoglutarate dehydrogenase and pyruvate dehydrogenase in citric acid cycle 17. On except? examination the patient was febrile with neck rigidity and A. Temporal lobe D. It is a coenzyme for alpha 18. Reese Ellsworth classification is used for predicting deal visual prognosis following radiotherapy D. Haemophilus influenza D.Latent varicella zoster infection is found in? ketoglutarate dehydrogenase and pyruvate T cells B cells dehydrogenase in citric acid cycle 11. It is a coenzyme for transketolase in hexose Streptococcus pneumonia D. Most likely Correct answer   : C.Decreased energy production in thiamine deficiency is pathogen is? a.Which of the following is false regarding intraocular A. Gram positive cocci were visualised C. Selective Serotonin Reuptake Inhibitors (SSRI’s) are the retinoblastoma? most effective drugs 94% cases are sporadic B. Mandible 23. anatomical waste infectious waste Medial pterygoid B. Increase in vascular permeability  16. Lateral pterygoid C.A seven year old boy presented to the OPD with 9. Mandible 24.7. It is a cofactor for oxidative reduction Staphylococcus epidermidis B. Glutamate GABA is the main inhibitory neurotransmitter. Ovarian cycle Estrous cycle Extradural haemorrhage – Torn meningeal artery C. asymmetric mononeuritis multiplex and eosinophilia are seen in? A.Rupture of Charcot Bouchard aneurysm usually causes? C. Aspartate Inhibitory neurotransmitters – Glycine. Direct pupillary reflex absent 33. Pulmonary fibrosis Correct answer   : A. Pain D.Cycle which is absent in humans is? Menstrual cycle B. HCV infection 35. Increasing the amplitude of action potentials in the motor neurons 32.The force of muscle contraction can be increased by all A. Intracerebral haemorrhage ETIOLOGY OF INTRACRANIAL HAEMORRHAGE 25.Triad of skin lesions. Behcet’s syndrome infection C.xx 42. Excitatory neurotransmitters – Glutamate.Pulmonary compliance is not decreased in? COPD B. Jejunum 34. Decreased surfactant production Pulmonary congestion C. associated with chronic hypertension. Glycine B. D. Bronchodilation  of the following except : Correct answer : D. Estrous cycle 26.46xy 46. Increase in vascular permeability  Correct answer   : A.Anti LKM (Liver Kidney Microsomal) antibodies are of 3 28. Vasodilatation C. Cryoglobulinemic vasculitis B.B. GABA (Gamma Amino Butyric Acid) A. Visual acuity normal Subarachnoid haemorrhage D. Bronchodilation B. Aspartate D. Jejunum C. D. Glutamate 36.Which of the following are vitamin K dependant clotting factors?II and IV (especially middle meningeal artery) due to trauma... D.Not an action of bradykinin? 37. Giant cell arteritis 39. Indirect pupillary reflex normal Correct answer   : D.69xxy c.Which among the following is the primary action of nitric oxide in the GIT? A.The most common genetic make up in Gastation trophoblastic neoplasm is ?a)69xxx b. Subdural haemorrhage C.Anti LKM antibodies are found in? Inflammatory myopathies B. Vasodilation Subdural haemorrhage – Rupture of bridging cortical veins due to trauma. Acetylcholine C.Which of the following inhibits thrombin activity? Heparin cofactor II Alpha 2 antitrypsin Chymotrypsin Alpha 2 macroglobulin Both a & d are correct but mostttt correct is D. Primary biliary cirrhosis Correct answer   : C. Relaxation of gastrointestinal smooth muscle C. Polyarteritis nodos C. Intracerebral haemorrhage – Rupture of Charcot Bouchard aneurysm. Relaxation of gastrointestinal smooth muscle 27. Contraction of gastrointestinal smooth muscle B.The main excitatory neurotransmitter in the CNS is: A. COPD types: Anti LKM 1 – Autoimmune hepatitis type II and Chronic hepatitis C Anti LKM 2 – Drug induced hepatitis Anti LKM 3 – Chronic Hepatitis D 29. Antibodies present in the other options: Inflammatory myopathies – Anti Jo1 antibody – Anti alpha enolase antibody Behcet’s syndrome Primary biliary cirrhosis – Anti mitochondrial antibody against E2 subunit of pyruvate dehydrogenase complex 30. Recruiting larger motor units Correct answer : C. Which of the following genetic mutation has been described in aortic stenosis? a) KCNH2 b) KCNQ1 c) NOTCH1 d) SCN5As 40. Churg Strauss Syndrome D. Extradural haemorrhage B. Indirect pupillary reflex normal A.Reabsorption of water in the GIT is maximum in? Duodenum B. IX and X III and V VI and VII 31.the most common cause of mastitis?? . Increasing the number of motor units activated C. Vasoconstriction Correct answer   : B. Correct answer: D. HCV D. Hormonal cycle D. Increasing the frequency of activation of motor units B. Increasing the amplitude of action potentials in the motor neurons D. Intracerebral haemorrhage Correct answer   : C. Colon Correct answer   : B.Mitral annular area in systole: a) Decreases by 10% b) Decreases by 25% c) Increases by 10% d) Increases by 25% 41. Subarachnoid haemorrhage – Rupture of berry aneurysm. Ileum D. most accurate test for ectopic pregnancy?? 1. except a. abd enlargement is b.fibroadenoma c.nicotinic acid Aids pt wd treatment side effect pancreatitis neuropsthy…didonosine . Resting membrane potential acieved by:A. C8C.isthmus c.all related to CCOP. IronD.thromboembolism b.sixe and shape Pallegra . Which of the folowing prolong use of drug will cause persistent inc in gastrin secretion:a. 6.hirsutisim b. 12. 9.PPI Compression effects on esophagus by which organ enlargement:A. Fragile X Syndrome (Downs wasnt in options) Rt optic tract damage 30 yr male newly diagnosed wth HTN.laproscopy 2. dentist referred back to physician saying there is no dental probe.krukenbergs d.serous cystadenocarcinoma 44.8B. 54. Right atrium E. 0.inc 51. Increase incidence with increasing mother age B.infertility c. serum cholesterol 200.is same for all types of cell Diagnostic tumor marker for ovarian CA:A. urinary beta hcg c.95C. Periaquedactal of gray of mid brain Stem about diphtheria vaccine reaction:A:type 1 B:type 2 C:type 3: d type 4 Which of the following increases lower eosophageal tone n speeds up gastric emptying time:A.. FBS 88 mg%. beta hcg b.ampulla d.ovaran tumor with psammoma bodies? a.coli d. Vit CC.ovaries b.stroke d. Downs C. 19.inflammatory carcinoma e. Serum Albumin(PT is to monitor Liver failure) Patient with Gastrectomy presents with Anemia after One and half year. 7. CEA B..cervix 52. most common complain of pco?? a. Left ventricle Normal FEV/FVC1 Ratio:A.obesity d. what will you give:A. Lesion of spinothalamic tract at T8 B. K 2.b blockers d. 21. bilateral normal reflexes: A. Diffusion of potassium outside the cell . a.Na K pump …C..enlarged clitoris 11. Matrix of nail C.CA125 inv in all ovarian cancers d. Na 140. Turner A pt presented to his dentist due to inability to put his dentures. Primary hyperaldosteronism (asim gynea page 328 ) Soft Keratin with no nucleus found in:A.prognosis is better if age of pt is less th 65 yrs 18 april medicine test mcqs…tried to remember all but few remain… 3. 43.B.coronary disease c. risk factr for ovarian ca . gold standard sign of virilization?? A. Jejunum B. CA 125 C.6 Prolong coma is caused by lesion of which nucleus:A. 17. ProprioceptionB. 28. Ileum Parasympathetic effect on SA node is due to 1 K 2 Na 3 Na and K 4 calcium 5 cal and k Mcqs on Gap junction Metaplasia…a change in function Dysplasia ..demetia in old women e.which vitamin def.antichollenergic c. Aorta B. Temperature(Pain and Temp is by Anterolateral System) Contralateral loss of pain and temp at T9 and ipsilateral loss of proprioception below T8. tubular carcinoma c.H 2 antagonist e.intraductal pappiloadenoma d.infiltrating ductal ca d.a.on spironolactone d. PainC. Cervical rib attached to transverse process of cevical rib will compress:A. Pulmonary artery D. 23.5 . Nucleus cerelous B. 24.cirrhotic pt b. T1 .ductal ca in situ 50.E. 0. Cortex of hair B.ultrasound d.staph aureus 4. b. cystic teratoma b.inc plasma volume c.endomatrial ca 45. examples of pathological gynecomastasia.hirsutism d.granulosa theca c.inc RBC mass d.CORTISOL 49.oligomennhorea 46. 22.ESTROGEN D.acne muscle mass c. a. except a.palpable ovaries in post menopausal women is ovarian cancer until proven otherwise. 13.The most common diagnosed breast cancer is??\ a. Left atrium C.res acidosis 53. Diabetic nephropathyB. except? most commn sign 5.. Klienfelter B.strep pyogen b. PTD. true for ovarian ca. Hemisection at T12D. 8.breast cancer with wrst prognosis?? a. Stratum corneum Monitoring of HEP B virus can be done via: A. Vit B12B. Serum Protein E. what is cause of his HTN: A.ductal ca in situ 55. except Mutation of BERCA SUPPRESSOR Turners syn peutz jegarhans synd histry of breast ca multiparity 15. 85 yrs old age man with no appearent disease.inc GFR b.best serum marker of menopause C.lobulr ca in situ e. 16. Metoclopramide Scanty bar bodies: A. where is the lesion: (STEM NOT PROPER)A. S. 14. except?? a.phyllods tumor b. Aluminium hydroxide B. Hemisection at L1 or 2 (asim surgery page 21 4th edition) Downs Syndrome:A.. SGPT (or ALT) C. urinary sugar ++ RFTs were normal. 18.inc thyroxin e. on ECG lt ventricular hypertrophy. Brocas B wernik Maximum absorption of salt and water:A.yolk sac e.features of normal change in pregnancy. 27. 0. substantia nigra Cause of Mental retardation:A. Blood transfusion Dorsal Column Medial Leminsical carries:A.antacids b. c.kelinfelters syn c. 25. common site of fertilization? a. 48.. Complete spinal cord sectionC..on digoxin for ccf e. pagets disease b. PheochromocytomaD.FSH 47. a.LH B. Extra chromosome Aphasia (Non-fluent) after stroke or Trauma which area of Brain is involved:A. AFP 20.C5B.pseudomonas C. Bilirubin B. 10. Essential hypertensionC. 26. trochealor nerve damage.presistent urcharus 78. vit acitvated in intstines –>vit k 73. Parasyphetic effect . Iv hep(ANS)Iv hep plus oral anticoag…Aspirin 88. eclmptc ldy  drug given to her before GA hydralazne sodm nitropruside 75. dif b/w anaphylctc shock nd hypovlmc shck 71..asim 51. Primary hyperparathyroidism…asim and shoab gynea mcq no 79 page 320. Young pt wd Atypical cell wd lymphadenopathy.parkinsons/basal ganglia invlvment 85.invest monospot 44. Poorly cross bbb. muscles for foot eversion. 55yr old Pt allergic to penicilin. Sup to inf in intercostal space Nerve artery vein 90. Pregnant women 1st trimester.. Prophylaxis Vanco plus genta…Vanco…Clinda…Ampicilin…Amoxil 89. no fvr options were chronic inflmtn… serpous effusion abscess.bowmane cappilaryy high pressure . B cell damage in mononucleases asim and shoab 43. neutrophils 98% lymphocytes 2% cloudy plerual effusion of an old man wd h/o cough. FFPs mutated factr 5 laden 49. Chorea.proprnolol 39.pudendal nerve 64..... bells palsy …lesion at cerebelopontine…lesion just befre stapidius 52.  all anti psychotics are likely in  anti ematic effect anti pyschtic potency in mili grams  price 87.get dental treatment. purulent effusion. cavrns sinus contains -> ICA and abducnt nerve 72.. gas cangrene is due to RAbia plus asim 54. Pt wd high bp…dec afferent arteriole resiistnce. 50. malaria scenario 77./….whch nrve is involved…Obturator nerve...phenobarbitol 59.. qstn abt SMA brnchs Middle colic artry 65. progestrne is precusr to whch hrmnes asim gynea 66. . treated wd naloxone not responding. chrons and UC defrntitng lesioins Perianal lesion...levodopa… dopamine …diazepam. B cell cd19 and cd 20 asim and shoab 41.hypothalamic sensitivity to estriol 37... fibrinoid inflmtn 68. CRF ->///low co2……… oral intake is limited to 0..left atrium. left adrenal vein drain 70.peroneus longus 48..pupillary muscle contraction. membranous coilitis is cuased by 55. pO2 will be equal to 100mmhg in left to rt shunt rt to left shnt fibrosis high altitude 61. S4 due to ventrilur fiilling ganong mcq… 40. Tremor of teacher tx... whch spreads via orofcal trnsmsn hep b c CMV hep e 62. in pregnent womn whch is incrsd TIBc 60. if 100 cm ileum is resectd wht will happenInc water loss / inc bile salts loss…..Inhibits both immmune plus serological response. testicular ca markers 53. Contration of smooth musle anp… aldosterone…. dilated pupils occulomotr nerve damage 56. axillary fascia is derived from deep cervical…prevertebrl 82. Activated t cell in mononucleases asim and shoab 42. Puberty…estrogen.. macroprolsctinemia asim gynea page 325 84. urine osmlarity 600mosm urea 15 Na 110 K 3. asim 46. qstn abt carcinoid tumors 86. Inguinal region below inguinal ligamn…medial to lat. drug causes acidosis ulcers salycylate 58..29.navel 91... 34. 57.phenytoin 30.dec intestinal motility. adh 33. asim 45.4 options: SIADH dehydration addisons 47. sjogrens is assocated wd RA (ans) 67.... musle spindle or golgi tendon asim surgery mcqs.changes its shape wd function 83.. following splnectmy immediate finding will be – > thrombocytosis…… Howel jolly bodies 69. qstn abt aziothioprine mech of action….. myxoma…....develops atrial fib.. Ligamentam arteriosum is between Arch and left pul artery 92. qstn abt secretin …pancreatic hco3 63. Insulin s stimulated by glucogan 36. Wrist injury… deep ulnar nerve injury (asim and shoab gynea) 31. inclusions smthing . 38.. Tachyphylaxix means… 35. smthng abt amyloid light chain 81.5 l/day 74.. 79. young female wd recurrent abortion and dvt that is nt corrected even aftr giving…. pain ovry reaching medial side of thigh . thyroid epithelium...very imp mcq… 32. puberty starts wd increase in gondotrophins gonadal hormones 76.prev h/o rhematoid heart disease.. bby wd urine frm umblcs ans. chemotactix c5a 80.what would u do. .invlved in blood clotting Tb…t cell macrophages type 4 reaction Sle…scenario Pernicious anemia. Hla…. 183.extensor of trunk Rapid adaptation…pascinian Infra orbital artery… maxillaery branch Memory area…operculum of insula…temporal lobe Olfactory cell…bipolar cells Virus causes cancer by changing protein synthesis Feco oral route.glucose lower than normal 113. Supralateral Floor of 4th ventricle frmded by medulla pons…inferior cerbllm peduncles 102. Wound contraction Primary hepatocellular carcunoma caused by  A.lithium tx 126. Pt wd recurenr throat infection .. 150.standing..vasculitis.focal necrotizing vasculitis 131.. 163.atria fib . Hla…other bcq ... 173. 184...pansystolic murmur 127.femoral vein 130.lat pterygoid dam Dobutamine action . Branch of whch artery is end artery…coronary…central retinal artery Remnnt of allontios…persi urachus Taste center…??? Ionizing radical cuases cancer ... Dead space unchanged. 149.. A scenario wd increase jvp.rabia 122.rabia 120.After GASTRECTOMY Staph aureus inf after thorn prick Apoptosis…cancer … Anti oncogens p53 Vibrio cholera…. Rennin release. 181..raw vegetable liver 97..diarheaa…selective igA defici 117. 178. Cerebellum…afferent from motor cortex 101. Folic acid source.. Pancreatitis fat necrosis 118... Duchene dystr.. 168. Dead space volume is not changed…standing… tracheastomy…deep breathing…do cheetara note on dead space 107.dx gram staining 94.... 145.jg cell Insulin sec stimulated by glugogon Succinyl. Pheochromocytoma vma 99. Aflatotoxin  B. Dvt. Dilated cardiomyopathy. 147. 142.purkinjee Dorsal rami. 148. Venous return in exercise. 175...pyruvate and acetyl coa 106. Inc gastric motility.pulse deficit Female cant open mouth. 155. MyeloperoxidaSE…. 158.grft rejection. 154. 182..allergy creatinine is formed in muscles… liver. Granulation tissue B.. 174.antimitochondrial antibody 109. 153.asim 115.. Brain area 321 postcentral gyrus…agranular cortex.. Cs f.deep inspiration Female wd silicon breast implant having inflamtory ...hep E Dvt. Multiple sclerosis…reactive gliosis demyelination 140.. Mean 110.psudocholinsterase Plasma fibrinogrn. epinephrine 104.. High level of estrogen.upper part of crista terminalis Respiratory segmnta tree…one atery for each segmnt Diabetic nephropathy…urine albumin Fastest conduction. Lp site. Red blood cell in csf.invest cpk 134.calcitonin .. Heptatis c C. Primary billiary cirrhosis.. Yeloow color pleural effusion. Metaplasia asim 123... Cortisol... 159.. 157.not necessary in liver transplant if abo compate 114.. 152. Psudohyphea and budding. Ankylosing spondy b27 119. 176. 165..supraoptic nuc ttypical case scnro of asprgilosis local edema. 166..primary somatosensory 100.gastrin 132. 160. Beta blockers…Av block 128. 177.plasma cell nuetrophils esinophils Deudenal ulcer. 161.NUETROPHILS Rta…ist vasoconstriction..immobalization Adh .. 143. Dysplasia…wd big nuclei of cervical cell asim 124.. Wegener granul scenario. Alpha feto protein testicular ca 125. 151. Uncoupling of oxidation reaction.hypo epiblast. Second week.thyroisds hormone… insulin…. 141. 172. Atps 38 total 105. 146.bet 1 agonist About skeletal muscle…aponuerosis is fibrous and membranous…belly is fleshy all along length…origins are proximal and mobile…insertions are distal and immobile... Bipolar disorder. 144. 180.marker. Aldosterone mainly act on .. 169.endometrial hyperplasia 116. 185.. 162.upper boerder od l4 98. Inc triglyceride…chylomicron remnnt 136.. Sulphur containg amino acid cystine 112.. 167.when moves head to left 95. 164.coxsackie virus 129. 186.93...subarachoind hemmrhage 138...DNA Dif btw primary abd secondary wound healing A. Mitral reg. Mcq about upper motor neuron lesion 139..gastroduedenal artery Sa node. Alpha 1 anti trypsin def Medical ethics pillars..neutrophils 90 percent…basophils 2 percnt serous effuse…purulent effusion 96..forgotten the stem but tukkaa lagaya 137. kidney amputation and then tumor of nerver…nueroma?? Fetal life po2 is 80 % in which vessel…umbilical vein Nasal scraping for leprocy Atrial fib. Gonococcus. 170.. Product btwn glycolysis and tca.contraction of leg muscles 108.proximal distal loop collcting 111. 156..which has diffrnt function in adispose tissue and liver in protein syntheses 103..candidiasis 121. Vitamin k def scenario 133..autonomy beneficience male efficeience justice Thyroid Medullary ca….. 179.. Thymus…no lumph nodules.forgotten options of invest 135.digo. 171. Ornithine transcarbamoylase def scenario..lung translucency etc…pulmonary aneurysm.treachostomy. Rt Horinzal semicircular canal.. mode ?? 49. stomach rotates  22. fisher man with bleeding gums and echimosys ? deficiency of ?? A.c low rbc folate level 190. injestion of drug ? NaHCO3 given as antidote ? A. severe diarrhea senerio ? a. 55% c. cholestrol  45. KERATIN b. X RAY b. PIE CHART  23.  choice of investigation ? A. transduction by retroviruses  44. Muscle length measured by ? A. LFT b. dna transcription d. proto oncogene becomes oncogene by ? a. Corticosteroid dec what ?  A. e-coli  33. TRACHEOSTOMY b. epidermis c. inc lh c. 2% bupivicain in 4ml ? A. Necrosis type in pancreatitis 28. factor 9 deficiency should be treated by ? A. lenght of right bronchus ? A.02%  25.. cyclic GMP which hormone included ? A.. Female with history of c-section and blood transfusion 10 years back now presented with edema and ascites. progesteron (if given gnrh agonist or gnrh itself then that would be ans ) i dont remember exactly !! 18. dead bacteria 31.alveoli . Trichomonas d. folate  34. 1 c. burn c. upper outer quadtrant  48. Low sodiam b. Ovarian hyperstimulation to INC FSH b. thyroxine or thyroid hormone increases what ? A.5cm b. ACUTE BRONCHOSPASM b.hypothermia d. Dec peripheral conversion  15. 2 b.187. cervical dysplasia caused by ? A. BREAST b. u/s 39. CHLAMYDIA b. Cardiogenic shock due to ?   A. CAPILLARIES  37. DEAD NEUTROPHILS b. brain c.. THIAMINE b. soft tissues c. AT C3 4. mri c. DEC VASOMOTOR TONE    3. one group of individual with vomitting and one without ? type of data ? A.. POST MI b.08% c. 2 sec insp and 2 sec exp b. 95% d. 0. AB+ve b. dec lh  41. gram negative sepsis  2 . PHENOBARBITONE  40. icf d. Diagnostic feature of folic acid def…a megaloblstic erythropoisis. Pubic bone joint type in last trimester…symphysis syndesmosis 188.6 bi phosphate 53. common pathway between gluconeogenesis and glycolysis ? A. FACILATED DIFFUSION b. 20% d. LEFT RENAL VEIN ANT TO AORTA AND LEFT RENAL ARTERY  b. Amenorrhea in lactating women by ? A. angiotensin 2  189. dead space decreased by ? A. gene over expression c. shallow breathing c. limb bud d. left shift ?  A. blood supply of uterus ? A. Lymphocytes  5. serum to urine osmolarity ratio  36. rra smaller than left  50. anticholinergic drug used in the treatment of ? A. 3 sec insp and 2 sec exp c. gonococci c. pectoral and axillary lymph node drainage ? A.pus contains ? A.. dec estrogen d. ab -ve 13. 70% b. standing d. 80% taking hormne replacement while 20% not ? A. FFP b. HEART b. VIT C b.. carcinoma contain which filament ? A. Pulmonary atery ends in . 9 min  52. menopause ? patient with lab result ? A. hypothermia  6.iucd b.PROLECTIN b.5cm  12. 0. largest total cross sectional area in ? A. dermis 20. universal recipient ? A. BONES b. Dopamine infusion steady state ? A.bronchioles bronchi 17 april gynea by remsha sheikh 1. co poisoning  7. allergic asthma 14. Pul artery dilation is due to . Calcitonin secreting tumor are present in ? A. THYROID b. Which hormone regulate prolectin ? A. hyoid bone in errect posture located at what level ?  A. Muscle spindle  10. NOMINAL DATA 26. 1 sec exp and 3 sec exp  11. FRUCTOSE 6 PHOSPHATE to FRUCTOSE 1. triglycerides c. left shift ?  A. 1. point mutation b. uterus  42.b. median .a atherosclerosis b. there was a Q on mean. hsv2 24. ANP  35. 2. renin inhibited by ? A. tamoxifen ? A. NEURAL CREST CELLS b. structure formed at the end of 5th week ? A. Transverse incision suprapubic structure injured? A. hemorrhage e. deep inspiration  43. senerio regarding how many bar bodies in XXX ? A. Gnrh antagonist c. cryoprecipitate 38. most common organism transmitted through STD ? A. co-relation is said perfect when ?  A. true about kidney ? A. total lenght of respiratory cycle ? A.low serum folate .. afferent arteriolar dilatation  27. mechanism of action of carbidopa and levidopa ? A. increase GFR ? A. simple diffusion  51. metabollic alkalosis rathr than metabollic acidosis 21. Mg stores abonduntly present in ? A. ecf  30. parathyroid  9. 80% b. brain c. lead sheet used in pregnant lady for ? A. high potassium c. DOPAMINE 46. +1 8. riboflavin  29. 0 d. neurogenic shock due to ?  A. 85% 17. multiparity c. desmin  32. glucose transort from mother to fetus ? A. 5 min c. FREE FATTY ACIDS b.anuerysm 191. melanocytes originates from ? A. ovarian and uterine artery  16. IgA percentage in genital secretion ? A. vit b12 c. 2min b. 3 47. person taking carbohyderate deficient diet ? which vitamin should be supplimented? A. INF EPIGESTRIC ARTERY  19. diabetic patient developed sensory abnormality of left leg and ulcer of big toe? . osmorecepror b. ESTROGEN and PROGESTERON both b.. left upper and right lower paralysis c. 40 d. inc ph d. ant frontanalle  67. pregnant lady with labur pains died on the way to hospital due to ? A. caseous necrosis b. radiation exposure injury mechanism ? A. inc po2 c. stroke volume  71. arterial supply to bone by ? a. ?? 70. cushing disease B. methyl melonic acid 101 . anterior spinal nerve give cutaneus branches by ? A. c. factor 9 deficiency  78. beta blocker ? A.history of tick bite ? a. maternal antibody titre c. sputem AFB b.. Autopsy shows blood in pericardiam ?  a. cervical metaplasia 103. inhibition of reflex by centre located in pons 77.4 tracheal ring  94. uterosacral ligament b. patient with multiple fractures of pelvis having loss of sensation in perianal area. patient treated for TB pericarditis now dies suddenly . christmas disease ? A. metaphisial and nutrient artery b. dec osteaclastic activity  93. lie at the junction of external carotid artry d. pt in sibbi balochistan . lyshminiasis 88. adrenocortical insufficiency  90. Estrogen ? A.c4. inscised wound ? factor aid healing ? A. no increase of growth hormone in pregnancy due to ? A. mesonephric duct  85.3. sympethatic stimulation? A. pupil affected  76. inguinal ligament ? A.c5 82. constrictive pericarditis 62. mammary bud or milk line 68. ilioinguinal nerve d. adrenaline 79. estrogen c. thyroid ithmus lies at ? A. pudendal nerve root ? A. embryionic conter part of mullarian tubercle ? A.. intestinal sphinter relax  75. coticospinal decussation ?? A. large trunk senerio ?? A. cardiogenic shock 1st drug ? A... AUTOSOMAL DOMINANT b. parasympethatic stimulation ? A. ILLIOHYPOGASTRIC NERVE b. baroreceptor c. response to dec Ppo2 55.. stress hormone ? A. closure of epiphyseal cartilage b. q on diagnostic criteria for TB ? a. cushing syndrome c. pudendal nerve c. iucd in place ? a. nephrotic syndrom b. angiopathy and neuropathy  54. Q on pulse pressure increases by ? A. ANENCEPHALY 91. SUTURES  100.. micturation reflex by ? A. right upper and right lower paralysis d. PYRAMIDS  95. angle man syndrom c. urogenital sinus devided in to upper and lower part by ? A. fetal hydrocephalus head at station -2 from where csf is drained ? A.. clonidine . podt cutaneous nerve of thigh  59. thyroxin  108. 14 week pregnant lady had fracture and died due to ? A. renal cause of edeme ? a. DOXAZOCIN b.. Q on physiological functions of growth hromone ? a. hypophysectomy and ovulation induction? A. down syndrome ( triasomy 21 ) 63. malaria d.. broad ligament c. invasive Ca of cervix 87. POST FRONTANALL b. round ligament  58. Q was also related to amniotic fluid embolism  66. 22 b. air embolism c. s2. mammary gland originate from? A. hypoglycemia 110. NADPH oxidase  106. no veggie diet ?? A. INDIRECT COOMBS test b. PARAMESONEPHRIC DUCT b. responds to dec PH e. 2. INC PCO2 b. Q on characterstics of TB? a. free radicles  107. progestron  92. xlinked recessive  73. dopamine b . mentally retarted senerio ?? A. ISHIAL SPINE  74.camping and sleeps on floor . fever with lymphadenopathy .A. ant sup iliac spine to pubic tubercle 97. alpha blocker ? A. muscles of the back supplied by ? A. AMNIOTIC FLUID EMBOLISM b. T1 99. right upper and left lower paralysis  b. epiphyseal and nutrient artery  102. oxygen Pp when leaving placenta ? A. placental somatotropin b. senerio on single crease ? A. 50 109. GB pain to shoulder ? A. fat embolism  65. move potassium inside cell  69. 30. DORSAL SPINAL RAMI 98.. adult female breast atrophy ? A. Hiv associated with ? A. hypertension 104. filariasis c. which nerve is spared ? A.s3. CORTISOL 89. down syndrom  72. epithiloid cells c. pt treated with myocardial infarction now developed edemea ?causes? options?? 105.. caseoes necrosis  61. eating alot. FAT EMBOLISM  64. Acth increased and pt presents with hypertension and hirsutism ? A. limb short . 12 year ols with multiple episodes of staph abcess n klebsella attack ? deficiency of? A. fetal head at station 0 ? land mark called as ? A. Q on thirst stimulated by ? 84. kalazar b.  60. at 26 week of gestation with history of hemolytic disease in previous child ? which investigation should be done ? A. c3. cardiac temponade b.. caroted bodies response in stimulation to ? A. pregnant female . hmg and hcg  80. nerve fibre to uterus travel through ? A. pre central gyrus and left cerebral cortex damage will cause ?? a. pco2 in csf  56. pincipal action of insulin ? A. SEMINAL COLLICULUS 86.s4 83. mechanorecepter in bladder wall b. autosomal recessive c. plaider villi syndrom b . 15 year boy with megaloblastic anemia which investigations should be done ? a. metaplasia and chronic cervicites 81. left upper and lower paralysis 96. uterine prolapse ? A. pelvis and lower limb paralysis e. fetal blood for Rh sampling  57. carotid boies ?? a.. stuctures not passed through greater sciatic foramen ? A. glandular cells  113. scalene anterior  144. conter part of labia majora in males ? A. gastric Ca ? A. HEMODILUTION b. confidence interval can be find out by ? A. android d. sperms c. pharma mcq action of anesthetic require ? A. attachment of hepatic veins to ivc 152. most common cause of anemia in pregnancy ? A. 3rd c. liquifictive necrosis 114. sacral promontry un reachable . DENOVILLERS FASCIA c. more viscocity in artries e. 2inch from clitoris  156. body of sphenoid sinus b. scarlet fever toxin ? A. whip like cilia ? A. how rubella infection can be prevented ? A. s3 . bladder supply ? A. plasmodiam Malaria b. spleen squestration of rbcs and old rbcs phagocytosis in which part ? a. middle rectal artery is a branch of ? A. essential fatty acid ? A. labia majora drainage ? a. caudate 128. pudendal nerve b. more absorption in non ionised state  c. pre malignant condition of skin ? A. female in labour . walder fascia  134. education .BLOOD . android  136. arch less triangular . falciperem  150. 5th  162. RIGHT VENTRICLE  124. sub acute bacterial endocarditis caused by ? A. muscle tension  135. DEC VISCOSITY  142.111. cardiac abnormality  145. INTERNAL ILLIAC ARTERY  121. reticulocytosis 137. round ligament  166. post rectovaginal fascia  131.inc HCO3 in veins b. mosaicsm d. 122. oxytocin  115. in chronic intavascular hemolysis ? A. ventroanterior thalmus nucleus b. primordial germ cells derived from ? A. iron deficiency c. enterotoxin c. anthropoid b. Ig G  154. female with bi cornute uterus anomly and with that what other anomalies can be present ? A. MATERNAL ABDOMINAL MUSCLE CONTRACTION b. plateplloid c. functional layer of endometrium ? A. testes c. epididymud 165. most abundant immunoglobin in body is ? A. body of maxillary sinus 155. phrenic nerve lies on anterior surface of ? A. oxytocin d. post fornix 130. lipid solubility  159. sup inguinal lymph nodes 120. vaccination to babies  140. UROGENITAL ANOMILIES b. golgi tendon organ involved in ? a. two different genetic lines derived from a single zygote ? a. reagarding pelvic inlet increased transverse diametre and decreased anteroposterior diameter ?typ eof pelvis ? a. small size rbc in vein d. pelvic splancnic and inferior hypogastric plexes  158. exotoxin A b. vagina ? a. longest incubation period of which malarial parasite ? A. inc cholride ions in veins c. part of brain receive excitatory input from brain but doesnot send back axon ? a. limb deformity e. global aphasia  160. scarpa fasia d. anti rubella vaccine in pregnancy c. post 1/3 of interventricle groove supplied by ? A. s4 b. s. a child with dry cough . liver is connected to anterior abdomial wall by ? A. damage to brocas and wernickes will cause ? A. illioinguinal nerve  167 . L1 and L2 133. blood supply to uterus pass by which ligament ? A.regarding bupivican ? a. something related to articular surface  118. 6th b. posteriorly aorta B. SUPERIORLY CAUDATE LOBE c. and pain in joints . pudendal nerve  141. MEAN AND STANDARD ERROR OF MEAN  123. folate deficiency  138.nigra d. brain ischemia ? A. avoid contact d. body of ethmoid sinus c. antigens resplonsible fro graft rejection belong to ? A. red pulp sinosoids and marginal sinosides  129. which investigation will you do ?A. compound nevus b . zygote anomly 163. organ of corti  153. milk ejection ? A. prostaglandin 139. in anemia blood becomes turbulent due to ? A. bronchopulmonary  126. tertiary bronchus/segmental bronchus supplies part of lung ? A. fallopian tubes b.  127. female urethra not true ? A. medial group of superficial inguinal lymph nodes 119. vlt c. anal triangle lymph drainage ? a. bladder control ? A. menopause ? A. adenoma ? A. compactum and spongiosm  157. DEC HEPTAGLOBIN b. which type of pelvis is this ? a . linolenic acid  149. gastrosplenic ligament on the right side 132. simple navus c. ovale c. zygote acromegaly c. ENDODERM  143. hemoglobinemia c. globus palidus e. ANTI RUBELLA VACCINE IN PUBERTY b. spinal reflexes c. ishial spines inverted . mental retardation d. more oxygen in veins 117. braod ligament b. epiploic foramen ? a. during surgery from where can surgeon reach to pouch of douglas by one nick ? a. plasma cells  148. bucks fascia B.4th d. s2. Q about superior relation of pitutary gland ?? a. epiglottis d. HLA  146. TENDON OF OBTURATOR INTERNUS b. after prostectamy rectourethral fistual can be prevented by ? a. gynecoid b. VIRIDANS 147. loss of sensation over perianal area ? A. cardinal ligament c. pinna c. renal abnormality c. sternocostal and anterior surface of heart formed by ? A. station 2 on pv . low grade fever . menisci b. vivax d. hemoglobinuria d. most imp factor in 2nd stage of labour ? A. seboric keratitis d. difference of rbc in veins and artries ? A. hyline cartilage ? a. psoriasis 161. anthropoid  164. genomic imprinting b. h pylori  112. piriformis c. block Na channel in ionised state  b. SCROTUM b. dec response from ovary  116. cavernous sinus contains which cranial nerve ? A. FACIFORM LIGAMENT b. RIGHT CORONARY ARTERY  125. gamma globulins formed in ? A. . Polydactyly. p53. Metaplasia? A. Whole Wheat contain Vitamin? A. . Golgi Apparatus B. Albuminuria. 31. not sure) C. Functional change of Epithelium (Answer) . 16. Pulmonary Embolism. Costochondritis (Answer) D. Caseous Necrosis. Pleurisy . Chronic cervical adenopathy. Fragile X Syndrome? A. . Apoptosis. artery fat and suprarenal gland  FCPS Part-1 Exam Sunday 19th April. pateint have half/one side headache. Fatty Liver. Red meat (Answer. White meat B. . both d. Troponin-T. 13. In Acute Inflammation Fluid present is? A. 33. Yellow Colour E. B. 42. . Aneurysm of Aorta C. Mallory Bodies . Ans. T10 B. Self Replicative Organelles. Riboflavin . Abscess on upper outer quadrant of Rt. 21. Collagen? A. regarding kidney ?A. 25. Lightly Stain with Eosin 5. regarding blood supply of bones ? a. Highest Cholesterol content? A. artery  b. Young patient on corticosteroids therapy. Trisomy 13. cause? Ans. . 32. Giant Cell Arteritis. Increase number and size of cell B. One Child O and one child AB.. 27. . cause? Ans. . . cause? Ans.. Characteristic/diagnosis of Asthma? Ans. 36. 23. not melagnant? Ans. Myocardial Infarction due to block of LCA. Hemangiopericytoma. what occur? Ans. 14. Tumor invading Nerve Sheath? Ans. bones of hand by . Alcoholic Hepatitis. biopsy of Artery show granuloma formation? Ans. Gonadal Agenisis and Hypogonadism. Deep Venous Thrombosis of Leg. . B. CK and Trop-T normal. diagnosis by? Ans. Aortic Aneurysm at opening. contain Oxidase developed from? A. A B. Boat shaped cysts are seen? Ans. . vertebral level? A. female with Oligomenorrhea. . Mitochondria D. Nucleus C. nutrient foramen by.. . Protein more than 3gram (Answer) . Breast Atrophy caused by? Ans. decrease formation of collagen C. Chron's disease different from Ulcerative Colitis? Ans. Diaphragmatic opening. Defect in X chromosome. Abundant in Ligament Flavum B.. 12.. Atorvastatin (Answer. Diabetic nephropathy diagnosed by? Ans. . artery  c . karyotype XO/XX. 10. 39. First Biological marker of Myocardial Infarction? Ans..CULTURE b. Hallmark of HIV? A. 2015. Tumor Suppressor Gene. 17. cause? Ans. Scenario. cause? A. Filariasis . is known as Transudate C. Peroxisomes.. . Councilmans bodies are present? Ans. Re-epithelialization defective . Diuretics C. O C. Ans. Prolonged APTT. Perianal Lesions. 4. 29. Antinuclear Antibody. AB D. T8 (Answer) D.. FEV1/FVC <75% . Ans. Milk D. cleft Lip and Palate. Egg (some says Red Meat and some says Egg) . Esophageal Stage of Swallowing effected by? Ans. Scenario. 35. 30. C3b. Marjolin Ulcer. . B E. decreased neutrophil infiltration (Answer) B. 38. 40. Cell survival. Squamous Cell Carcinoma associated with? Ans. Female have Butterfly Rash. 37. . Morphine D. 8. . Tumor. 41. Specific Gravity is low than 1002. 28. no Uterus. 11. Drug which stable plaque in Acute Coronary Syndrome? A. Lack of Estrogen and Progesterone. no retrosternal burning. 26. after one month wound not healed. .. . Scenario. Ascites+Pleural Effusion. tuberculous suspected. T9 C. MI B. Thiamine (Answer) B. Tip of scapula. Thoracic Duct and Azygous Vein. Textile Industries. first test performed? Ans. Christmas disease. Specific Gravity of Fluid is 1010 B. CCF. . Immunodeficiency. . Pneumonia not respond to Antibiotics. Arm excised. O/E testes present. Lumbago Patient have chest pain and Tenderness at Posterior 2-4 Intercostal Space. Mother Blood Group A. if mutated. Scenario. 22. Father have? A. Trinucleotide repeat (Answer) B. Ans. 1. Aortic Aneurysm. T6 . 6. compress which structure? Ans. S ER (Answer) E. Byssinosis? Ans. Proliferation of virus inside T lymphocyte. Helmintic disease by caused by Mosquito bite? Ans. . scapula inf angle . . 19.. caused by? Ans.sputem culture c. female short Vagina. 24. . Two childs have two fathers. Pleural Effusion present. Teratoma. 7. . 18. Scenario of Alcoholic Patient. R ER 20. Scenario of Patau Syndrome. Elastic C. Pneumocystis carinii. 34. Opsonization. Testicular Feminizing Syndrome.. cause? A. Scleroderma. Nitrates . finding? Ans. 2. not Sure) B. which site of heart damaged? Ans.P normal. 9. Tumor with 3 germ Layers? Ans. Adenoma. afb  168. . renal fascia covers kidney/perirenal fat and suprarenal gland  169 . Microcephaly. Left Atrium and Left Ventricle. Medial Necrosis. 15. freely rejoin D. scenario. 43. 3. Adult Polycystic Disease. when surface exposed to Temparature is? A. 74. Isotonic contraction is different by? A. Horse shoe shaped kidney C. 64. (another Question also related with ACTH/Adrenocortical condition. Isotonic and Isometric contraction.44. Autonomic Nervous System? Ans. Gastrin. multiple Fracture. Osteoporosis B. Increase in Renal Blood Flow and GFR? Ans. after giving Thyrotropin TSH is Increased. . . Vitamin D deficiency C. 57. Have more energy of recovery D. Ans. first docor should ask "what Your parents have told you?" . Brain. i don't remeber exactly) . Dilatation of afferent Arteriole. . 35 B. CO poisoning (Answer) . Difference of Pulomanary and Systemic Circulation? Ans. norepinephrine C. . 59. Side. Chemical Synapse. Parasympathetic Increase GIT Motility. . Malignancy. Bicarbonate (Answer) B. Patient. 61. CCK causes? Ans. Problem in Thyroid (Answer) C. Scenario of Berry Aneurysm/hemorrhage. Patient came to clinic with complaints of Polyuria and poldipsia. Vasodilation cause by which Adrenergic Receptors? Ans. Rapid Ascent to high Altitude? Ans. 91. Neutrophills . at terminal illness. Bladder exstrophy B. 53. 70. 76. . 62. Pateint suffered Head Trauma. 45 (Answer. Ans. disease of? Ans. Steatorrhea. edema caused by? Ans. Organ have no change of Blood Supply during Exercise? Ans. Aldosterone causes? Ans. Multiple Fractures occur in which disease? A. Meningitis. 54. Epinephrine D. 79. Tricuspid valve Auscultation. Hemoglobin C. Hypothalamus. . Extreme left shift of Curve? A. hirsutism. Cushing Synrome . Lower end of Sternum on Rt. central Obesity. action with thephylline D. causes? Ans. 89. 47. . most likely? Ans. 4 calories. 51. 55. . Methemoglobin. . B. Anterior PItuatry Histology Features. Hypo ventilation. Problem in Pituatry B. Gluscose 40 (less) Lymphocytes. . Corticotropes constitute more than 50 percent of Ant: Pitutary. . Help patients themselves. 67. . A Girl of 8 years. Duplication of bladder D. Patient with tachycardia and Palpitations. Digoxin Toxicity by? Ans. 60. Chorea and Athetosis. Gastric Motility increased by? Ans. Pulmonary have low Resistance. Lipase. Ileum. 88. Patient transfused with 2 weeks old Bood. Major Stress harmone? A. 63. Hyperparathyroidism D. Counselling is? Ans. . association? Ans. RBC (Answer) B. ADH and Oxytocin produced in? Ans. Urothelial Carcinoma associated with? A. Blood contain mainly? A. . Less work done C. 58. Inhibit SA and AV node . HbF B. Pulse 120 b/min? Ans. Chromophobes are of smallest cell type. 81. Closure of Semilunar valves. 2nd heart Sound? Ans. . . Platelets C. 90. . 65. A. 72. Half life 30 seconds B. 82. Injection of ADH given but symptoms persists? Ans. cause is? A. . 80. Central Diabetes Insipidus. Inhibition of Gastric Emptying. Hypkalemia. Protein D. . Testes in Abdomen/ Cryptorchidism associated with? Ans. . cause? Ans. . 86. Heat production . Low Affinity of Oxygen? Ans. Adenosine? A. Major Blood Buffer? A. ACTH Incresed. 85. Nephrogenic Diabetes Insipidus. . Polycystic Kidney . Humans start burning sensation of skin. Insulin inhibits? Ans. deficincy of Pencreatic enzyme? Ans.P 90/60. . CSF shows. TSH low. . . Increase Hydrostatic Pessure. 77. Basal Ganglia. Transient Tachycardia C. Fluid Replacement. Random Sample B. not sure) . sub-endothelial deposits of IgG. Osteogenesis Imperfecta? Ans. . 83. Hypoparathyroidism E. Tuberculous meningitis. Systemic Sample (Answer) . Pulmonary Edema. Mean Systemic Filling Pressure? Ans. Incresed Thirst and Thirst not relieved by drinking water. ADH . Glycogenolysis. Loop Diuretics. B. 56. Thyroid Adenoma . . Collagen Disorder. One gram of Protein gives? Ans. Cushing Syndrome. . Monocytes D. 73. Increased Ph. 40 C. 45. 68. . Bile Salts absorbed in? Ans. Phosphate . cause? Ans. 87. Regarding Epithelium and Glands? Ans. 84. 49. and now Patient is asking "am i going to die?" Doctor response should be? Ans. Beta 2. 75. 78. ACTH (Answer) B. . parents told to doctor that dont anything about disease to our Child. symptoms Polydipsia and Polyuria. 46. In CCF. Affects Mean Arterial Pressure. 50. Accident. urine Osmolarity decrased. 71. . Ca uptake and release of ACh into cleft. Goodpasture syndrome? Ans. 69. 52. Urine Osmolarity decresed. Consume more phosphate bonds B. 52. 48. . Simple Squamous occurs at sites of Gaseous Exchange. Individuals have the independent and Same vhance of being selected? A. . . 119. 131. Route by which first pass Metabolism occur? Ans. Fainting may Occur during ventricle unable to pump Blood. Person Sitting Calm and Eyes closed. Ca Cervix. . . Appendicitis. CT scan D. . Ultrasound C. Antidote is? Ans. . 138. . 112. . which pattern on EEG? Ans. DIabetic Old Lady. 109. Nerve accamponies Superficail Temporal artery? Ans. Respiratory Symptoms? Ans. Ascariasis. 134. . . cause is? Ans. II and III . misdirection of which nerve? A. 117. B. . muscle effected? A. 103. not sure) . Parotid Duct open? Ans. 98. Tenderness in moving thigh medially and medial rotation. Scapula attached to Thoracic wall by muscle? A. Lower Motor neuron Lesion. Vertical Cut of Optic Chiasma. . Scenario of Weakness of Vision? Ans. Muscles of back supplied by? Ans. pap smear +ve. Oxidation. example? Ans. Liver biopsy B. 129. Regarding blood supply in the brain? A. Loop diuretics (Answer. Area of the Brain most activated before performing skilled movements? Ans. . 122.92. Virus Transfer information of one cell to another? Ans. 125. . 99. Oral. 102. Hepatic Artery. 126. 133. damage to? Ans. Alpha. Thickening of laraynx B. . Head tilt to left and Right Eye upward moved. Thorn prick form Abscess. Ans. superior ophthalmic (Answer) c. 110. Antagonizes factor Xa and Thrombin (not sure about answer) . Regarding Low molecular weight heparin? Ans. Bitemporal hemianopia. Staphylococcus Aureus. 114. hyperparathyroidism. . 96. Charecteristic Finding in Clostridium botulinum? Ans. 97. Blood Culture. Ans. Glossopharyngeal . Ans. Upper Motor neuron Lesion. Typhoid in week 2. Lattismus Dorsi C. 101. Induces overlying Ectoderm to form Neural plate. Rhomboids Major D. . . Cerebellar Lesion. 104. . . Drug which causes release of Histamine? Ans. Facial B. bacteria? Ans. Nerves involve in Light Reflex. 121. Scenario of organophophorus poisoning. Pelvic. diagnosed by? Ans. Arteries don't anastomose after entering the brain substance D. 105. . 100. Artery in True Pelvis? Ans. . 132. Medial Leminscus formed by? Ans. Deposits of Fat on Medial side of thigh . In complete Heart block? Ans. . (don't remember answer) . Cortical association area. diagnosed by? Ans. . Serology E. Erythropoietin. not sure) B. Darkening of areola C. Scenario. Protrusion of Mandible by Muscle? Ans. Ans. herpes Infection. 142. . Superior Oblique (Answer. Patient during eating. Cataract. Anti-Tuberculous Drug cause Hyperuricemia? Ans. 123. Blood culture+Widal Test. Flaccid paralysis. 93. Notochord. 127. Atropine. 120. 130. 94. . Dorsal Ramus. Ans. appendix position? Ans. Arteries don't anastomose on the surfac of the brain C. 135. 111. shows 2 P waves to 1 QRS complex? Ans. Miosis+ Salivation. . Anmeia. 115. . Rt. Eye swelling. . Typhoid at day 6. Drug for MRSA? A. Finding of Falciparum Malaria? Ans. 106. 118. Phenylbutazone. Superior Oblique C. . Tip of Nose. Internal Arcuate Fibers decussate. cause Thrombocytopenia in Child after Infection (don't remember Options) . Drug will be given? Ans. Ophthalmic division of Trigeminal. Infraorbital. High calcium in serum. Difficulty in moving foot medially/Inversion. Chronic Renal Failure. . Quadratus lumborum B. Loss of Function of Tibialis Anterior and Tibialis Posterior. Black water fever. Rt. ECG. Serratus Anterior (Answer. . . Drug causing hypoglycemia when given with sulphonylureas? Ans. Upper 2nd Molar between gum and Cheek and lips. after 2 years? Ans. Gastric Artery is branch of? Ans. 124. Vagus C. Morphine. 137. 136. Subclavian Vessels groove over? Ans. 108. Gentamycin. Carotid Sheath? Ans. 116. Transformation. block to Vein? A. diagnosed by? A. Internal capsule is supplied by anterior cerebral artery . Cavernous Sinus Thrombosis. Aurico-temporal Nerve. . First Rib. Middle cerebral artery supplies foot area (Answer) B. 140. Pyrazinamide. . . 141. . drug will b given? A. . Dysmetria. Wasting. Patient with Hemoptysis. . Lateral Pterygoid. Bisphosphonates B. 128. Testosterone function? A. 95. not sure) . 139. Co-trimaxazole (Answer) B. Increased lacrimation. Thickening of Investing layers of fascia of neck. Nerve? Ans. Stool Antigen Testing . Mobitz Type II. . Amoebic Liver Abscess. . CoAmoxiclav . Lt. Inferior Ophthalmic . Phase I Biotransformation. 107. Drug contraindicated in Chronic Renal Failure? Ans. Middle Rectal Artery. Bronchial Arteries. 113. Inferior Rectus . Scenario. Supplied by Internal Carotid artery . 158. 173. Dyspnea on lying down. Recurrent laryngeal nerve on isthmus D. Muscle contraction of leg. Contain Lymphocytes (Answer. Peroxisomes originate from? A. Radioactive Strontium causes? A. Heart. Osteosarcoma (Answer. Blood C/S . Performed after 14 weeks.G Apparatus. . 186. Ribosmal RNA formation C. X-Ray. End product of Purine Metabolism in Urine? Ans. not sure) C. Neutrophils. R ER C. Golgi B. 182. not sure) D. 167. caused by (related question. . . 145. 147. 146. DIC. . Infection. X linked Agamaglobulinemia. 165. XXY (Answer. . Rt. Heart failure (Answer. Hypotonic Saline . (don't remember exactly) . . Ondansetron. Upper End of Fibula D. Cause of Hypernatremia? A. defect in? Ans. Papillary Carcinoma (Answer) B. . Gamma Efferent fibers supply? A. Medullary Carcinoma D. DIC occurs due to decrease in platelet count. XX C. Most common bone fractured? Ans. Lady with decreased levels of plasma cells and recurrent infections? Ans. 153. Follicular Carcinoma C. Question of Biostatistics. Not given in pregnancy. Discharge from muscle spindles decreases when? A. Contain Euchromatin during replication B. Alveoli. Drugs causing Neuroleptic malignant sydrome? Ans. Primary Center of Ossification at Epiphysis of Long bone at Birth? A. Upper End of Tibia . 156. 175.P 220/110. Lower End of Humerus C. . XY B. M Leprae in Nasal Scrapings B. Complement deficiency. Central portion of muscle fibers . Nucleolus? A. 1st line cell in inflammation? Ans. Estrogen is produced by? Ans. not sure) B. R ER . 181. cause? Ans. 180. 172. not sure) C. 185. Ans. . Plasma volume increases alot by giving?? A. 162. B. 155. S ER D. Negative Intrapleural pressure. Patient having raised PO decreased calcium? Ans. not sure) C. 166. Carbohydrates and proteins combined and secreted from? A. Mediator of pain? Ans. Small cell Ca of Lung B. 189. . 159. Distilled Water B. muscle contracts C. Thymus? A. A child with recurrent infections decreased levels of all Immunoglobulins? Ans. Mitrochondria . . 177. Developed from 4th pouch C. Venous Return increased by? Ans. S ER C. when the muscle spindle is stretched . 178. 188. Glandular structure . Increase in salivary gland secretion. 190. Factor IIV d. M Leprae in sputum (Answer. not sure) C. . Haloperidol. . 161. Tibia. 183. not sure) B. Shortened PR Interval? Ans. End of intrafusal muscle fibers (Answer) B. Local Cause of delayed Wound Healing. . . Thrombocytopenia appropriate? Ans. 176. True Hermaphrodite genotype? A. excessive ADH . Bradykinin. Golgi? B. . Contains DNA and nonhistone proteins . Ans. cause? Ans. 184. . Retrosternal Goiter. Rt. Organ with high arteriovenous oxygen difference at rest? Ans. Hypertonic Saline (Answer. Lower end of Femur B. Ovary. XY/XX . Acute Renal Failure.143. end of the intrafusal muscle fiber contracts B. 164. 163. 157. Factor X C. Which is a function of Parasympathetic Nervous System? Ans. . 151. 150. 154. Eosinophils. J. Answer was Variability. Factor that maintains the lungs not to collapse? Ans. . . Scenario. . . Injury to spinal cord does not effect Respiration? Ans. WPW Syndrome. below C5. 169. Superior Vena Cava. Anaplastic Carcinoma . LDL. Factor IX . Lysosomes E. 152. Regarding Amniocentesis? Ans. Question about Immunodeficiency. . Smoking associated? Ans. 144. i dont remember exact) A. Thromboplastin B. Nucleus Ambigus (not sure. 149. 187. Lady with dypnea at rest. Lipoprotein with highest cholesterol content? Ans. . IgE interacts with? Ans. 160. not remember exact question) . Antiemetics during chemotherapy. 179. 170. Vagus Nerve Nucleus? Ans. Pulmonary Artery supplies? Ans. . Thyroid drain to deep cervical (Answer. . Most likely feature of thyroid? A. Uric Acid   . Bronchogenic Carcinoma. Slow growing Thyroid Tumor? A. T3 to 6th Chostochondral junction. 148. . Diagnosis of Leprosy at initial stages by? A. Oblique Fissure? Ans. . 171. . Pulmonary Embolism. Goiter moves on deglutation bcoz attach to prevertebral fascia B. Regarding Thiazides? Ans. Kaposi sarcoma . border of Heart shadow formed by? Ans. . 168. 174. Warfarin? Ans. Cause Hypokalemia.
Copyright © 2024 DOKUMEN.SITE Inc.